You are on page 1of 182

Practice Question:

1. Terminology

2. Synopsis on CR

3. Important articles on CR
- Mapping
- CR Strategy
- Important Links on CR

4. Strengthen and Weaken the argument


 Strengthen
-
 Weaken
- Insect infestations in certain cotton-growing regions of the world have caused [GP1]
- In Kravonia, the average salary for jobs requiring a college degree [GP]
- Although fullerenes--spherical molecules made entirely of carbon--were first found [GP]
- The country of Ertland has never imported apples in any significant quantity [GP]
- The Highway Traffic Safety Institute reports that the PZ 1000 has the fewest injuries [GP , Verbal 2nd]
- Tiger sharks are common in the waters surrounding Tenare Island. [GP] 600

5. Evidence Family
 Inference Questions
-
 Explain a Discrepancy/ Resolve the paradox
- City Controller: 63% of our residents voted to approve the developer's request [MCAT4]
-
 EXCEPT Questions

6. Structured based family


 Describe the role / Bold Face
- Discussion on Bold face
- In countries where automobile insurance includes compensation for whiplash injuries [GP] 700
- One of the limiting factors in human physical performance is [GP] 650
-
 Describe the Argument
-
7. Complete the Argument
- A certain tropical island received food donations in the form of powdered milk [GP]
8. Assumption Family
 Find the Assumption END
-
 Evaluate the Argument

 Flaw Questions

=======================

CR Terminology

=======================

Downtown is a term primarily used in North America by English speakers to refer to a city's core (or center) or central
business district (CBD), often in a geographical, commercial, or communal sense. The term is not generally used
in British English, whose speakers instead use the term city centre.

A suburb is a residential area or a mixed use area, either existing as part of a city or urban area or as a separate
residential community within commuting distance of a city. In most English-speaking regions, suburban areas are
defined in contrast to central or inner city areas, but in Australian English and South African English, "suburb" has
become largely synonymous with what is called a "neighborhood" in other countries and the term extends to inner city
areas. In some areas, such as Australia, China, New Zealand, the United Kingdom, and a few U.S. states, new suburbs
are routinely annexed by adjacent cities. In others, such as Arabia, Canada, France, and much of the United States, many
suburbs remain separate municipalities or are governed as part of a larger local government area such as a county.

1a : the process of readying a transport vehicle for departure after its arrival; also : the time spent in this process
<a quick turnaround between flights>b : the action of receiving, processing, and returning something <24-
hour turnaround time on most orders> 2 : a space permitting the turning around of a vehicle.

Airplane turnaround time is defined as the time required to unload an airplane after its arrival at the gate and to
prepare it for departure again.

Testament : something that serves as a sign or evidence of a specified fact, event, or quality.

Condominium - a building or complex of buildings containing a number of individually owned apartments or houses.
Overcome : v. to destroy, demolish, vanquish, expel

‘To overcome a problem’ means to the answer option will eliminate the problem. i.e. there will be concrete initiative in
addressing the problem. [OG13 CR 29]

-----------------------------------

---------------

Frequently used Terminology :

Main Conclusion—a summary of the argument’s primary position;

Intermediate Conclusion—a position utilized by the argument as a stepping stone in order to advance toward
the main conclusion;

Premise—a theory or proposition upon which an argument is based or from which a conclusion is drawn;

Fact—information generally believed to be true OR known to be true—usually advanced as evidence to


support a premise;

Evidence—specific type of fact offered in support of a theory or premise;


Context—a frame of reference of value in the interpretation of aspects of an argument or the argument’s
components;

Consideration—a factor (fact) to be taken into account in forming a judgment or decision;

Position—a point of view or attitude about an issue or question;

Assumption—a position or belief that is taken to be true, without proof;

Principle—a basic or essential truth (stronger and broader than a fact).

Judgment—an opinion formed from a consideration of the facts.

---------------------------------

DmitryFarber

Manhattan Prep Instructor

Let's start with "premise," "fact," and "evidence." These are all different words for a statement that we accept as true.
For the most part, these terms are interchangeable, but "premise" and "evidence" imply that the information is being
used to support an argument, while "fact" might in some cases refer to information that is not used to support one side or
the other.

"Context" is going to be more neutral. If something is context, it is providing the background for the argument, but
doesn't itself constitute an argument. Typically, context will be factual, but some kind of initial position or disagreement
could also serve as context (this is less likely).

A "consideration" is something we need to consider in making a decision. This will generally also be a premise, but it is
likely to be something new that is introduced after an initial argument. For instance, I might say that because John was
late to his interview, we shouldn’t hire him. If you point out that John was hit by a car on his way to the interview, that
would be a consideration introduced to counter my conclusion.

A “principle” is a different sort of premise—it’s not a fact, but rather an initial idea upon which we build. For instance, I
might say “No one who has committed a felony should be allowed to run for governor. Candidate X robbed a bank, so
she should not be allowed to run for governor.” In this case, “Candidate X robbed a bank” is a factual premise, while
“No one who has committed a felony should be allowed to run for governor” is a principle. For the purposes of the
argument, we would grant both of them as true. Therefore, the only assumption in this argument is that robbing the bank
was a felony.

“Judgment” is tricky. It sounds like a conclusion, but it may refer to a sub-conclusion or principle. For instance, in #123
in OG 13, it is used to refer to a premise. We are basically told that “If people are right to believe X, then Y is also true.”

“Position” means conclusion. It may refer either to the author’s conclusion or an opposing position that the author is
refuting.

--------------------------------------------
GMAT Critical Reasoning: Words You Need to Know
BY CHRIS LELE ON FEBRUARY 14, 2012

For many, one advantage of taking the GMAT over the Revised GRE is that they do not have to cram such polysyllabic
franken-words as juxtaposition, tendentiousness, and didacticism. That doesn’t mean that a shaky grasp of vocabulary
will hold you in good stead come time for the GMAT. There are certain words you must know, words that pertain to the
Critical Reasoning section on the GMAT.

If you’ve done any prepping on the GMAT critical reasoning, you may have noticed that certain vocabulary pops up
frequently. Most of these words relate to weakening or strengthening an argument. As long as you have a strong grasp of
these words, you will not get tripped up by the jargon and will instead be able to focus your energy and effort in
dissecting the argument.

Below is a list of vocabulary words that you have to know for the GMAT Critical Reasoning section.

Undermine

To undermine is to weaken over time. In argument-speak to undermine an argument is to weaken it. Undermine should
not be confused with underscore, which means to provide support.

“All of the following serve to undermine the editorial’s contention that…”

Maintain

Maintain is known as a homonym – a word that has multiple meanings. The definition we are concerned with here is to
state strongly, assert.

“Proponents of the accumulation theory maintain that…


The network, however, maintains that negative reactions to the report had nothing to do with its loss of viewers. [OG13 CR 19]

Posit

To posit is to make an argument on something that is unknown. For example, many astronomers posit that we are not
alone in the universe. We of course don’t know for sure whether little green men are out there.

Important synonyms for posit include postulate and advance, the latter of which is a homonym.

“, some researchers into sunspot activity have posited that…


Corroborate

To corroborate is to lend support to. The word corroborate is typically accompanied by the word evidence.
Corroborating evidence strengthens a claim.

“Which of the following, if true, best corroborates…”

Paradox

Many test prep programs classify an argument type as a paradox. Specifically, if there is some unaccounted for
discrepancy in an argument – and we of course have to figure out what that discrepancy is – we are dealing with a
paradox.

More broadly speaking, a paradox is a statement that, on the surface may seem absurd or untrue, may actually be true.
For instance, the French are known to have very rich diets yet remain thin. The Americans, on the other hand, ever
obsessed with the latest diet fad, are….well not so French. Who would have thought? (A sentiment that embodies the
paradox).

“Which of the following could best account for the paradox noted above…”

---------------------------------

A small note on bold face questions-- The cue words that can expose the relationship

- Daagh

The correct interpretation of the action words largely brings out the relation between the two bold face statements. It is important to understand the
basic tenets of CR passages such as premises, assumption, the conclusion etc; we must also be thorough with what the argument seeks to establish,
or seeks to oppose or in effect what direction is the argument is heading on. The primary purpose of the argument or the focus of the passage
should be clear in the mind before we get down considering the choices.

The broad classification of the descriptive words is

1) those that are premises or evidence that lead to a conclusion; those that support or oppose a conclusion, unstated premises such as assumptions.
These are facts and mostly derive from the present or past. The cue words for this category are since, because, although etc.

those that are conclusions, interim conclusions or final or main conclusions; those that are speculations, judgments, opinions, stance, stands,
feelings; These may not be facts and are more in the nature futuristic events and liable to become wrong.

3) words like claim, consideration, position, strategy, policy ,objection undermining, drawback, define, describe which are subjective expressions.

1. Premises words

a finding – a discovery , a revelation


an assumption: -- a non-existing factor, to be drawn to lead to a conclusion, invisible factor; take for granted
Consideration - A factor to be considered in forming a judgment or decision
Evidence - grounds in support of that consideration
Phenomenon – an occurrence, a thing of past or present
the circumstance- context, setting, the atmosphere, a situation
Observation—seeing. the past or the present
Platform – a ground from which something is derived
2. Conclusion words :

a hunch – a premonition that something is likely to occur


a judgment – personal assessment
a prediction, guess, premonition, intuition, instinct
a proposal – a futuristic offer
a recommendation – a futuristic advice
conclusion – What the argument seeks to establish
contrary conclusion – opposite decision
Generalization- a general idea developed from smaller premises, a principle, statement, or idea having general application. Can be both part of a
conclusion or an observation
interim conclusion, a minor conclusion leading to the main conclusion
Judgment opinion - thinking , view, estimation
main conclusion - the final conclusion
Opinion, - personal view
Proposal – Proposition, Suggestion
Speculation, -- uncertain, like to happen not a fact

3) descriptive words

a claim – an assertion – averment - insistence


a drawback – a reverse
a policy - guideline, rule
a strategy - modus operandi , a method to reach an aim
an objection – opposition, criticism , undermining argument :

The argument The main point of the topic the text the passage seeks to establish. - the point of the passage supported by the argument
calls that conclusion into question.- doubts, undermines , weakens , disagrees , disputes
challenges (verb) – questions, negates,
to claim - to demand, to maintain
Contends: to controvert or debate; dispute.
Context- platform, opportunity, situation. Scenario
Contrary --Opposed; contradictory; repugnant; inconsistent criticism
Define: give the meaning of the word, like dictionaries do.
Demonstrate – prove
Describe: give some characteristics of the item defined.
Explain: give reasons why the item is what it is or does what it does; provide more complete information
Evaluate - To ascertain or fix the value or worth of; to examine and judge; weigh carefully; appraise
explanation – giving reasons
identify - recognize, locate,
Illustrate -, explain with examples, to buttress the author’s point
Imply – mean without mentioning
Implications – after effects; side effects
Infer- deduce something from stated facts
In part --- based on one of the premises
in support - to endorse;
in support of the overall position – to agree with the argument
Policy – principle, rule, guideline
potential objection – possible, or implied objection
Presents - to offer for observation, To introduce
presents a situation – provides a platform
provides further evidence – fortifies, reinforces with additional facts
provides support– goes in tandem with the reasoning
questioning – doubting, criticizing, undermining,
supplies a context - provides a ground
support an opposing position – to oppose
that the argument disputes - the argument disapproves, or disputes
the content – the actual wordings or expression
the course - the path
the position A stand taken, one’s thinking ,
to challenge – to question, to criticize .
to support the position – to support some stand
to undermine- weaken, call in to question, raise doubt
an appeal to authority – an automatic acceptance of an expert’s or celebrity’s opinion

While the above is a general idea of the usage of these words in boldface questions, they have to be understood in the special context in which they
are presented. Some of them may also be on borderline; So you must use your contextual judgment in such cases.
_________________

you can know a lot about something and not really understand it."-- a quote
No one knows this better than a GMAT student does.
Narendran +9198845 44509

----------------------------------------------------

Strategy and Tips for Bold Face [The Final Frontier]

Folks,

Got hold of this one from Charu's notes. I thought this could be useful -

Bold Face Questions:

The fine folks at ETS (“Creating Access to Graduate Business Education”) bring you the final frontier in verbal testing: the GMAT bold-faced
critical reasoning question—the last hurdle between you and the 700+ score you so richly deserve. But these questions seem to cause a lot of
anxiety among test-takers and test-takers-to-be. Why? I think that it is because these questions are strange and uncharted. Approaching the BF
question is a bit like trying to read Dostoevsky, in Russian, while stumbling around in the dark, in a room full of holes. It’s disorienting and
confusing and generally unpleasant. There are no clear references and no decent guides. We fear the unknown. We try to avoid what we do not
understand. But given our common goal, let’s get to know the BF question. Let’s come to understand it. If we know it and understand it, then we
can kill it.

When you see one of these questions in your actual GMAT, the first thing you should do is congratulate yourself on having done well enough to
have brought the BF challenge upon yourself in the first place. In the final analysis NOT getting a BF question or two during the exam is definitely
WORSE than getting them. So, you’re already doing something right. The key is to use what you know to split the answer choices. Here’s my
approach:

(1) Read the argument. Read it quickly, as you ask yourself, “What’s the point here?”

(2) Identify the Main Conclusion. You’ve got to identify the main conclusion to proceed—the main conclusion is your “port of entry” into the BF
question. So, find main conclusion as quickly as possible and note whether it is one of the bolded phrases.

(3) Go directly to the answer choices. Do not, I repeat DO NOT, spend any time trying to figure out what roles the bolded phrases play within the
argument without some idea of the terms that are being offered in the answer choices. It’s a waste of precious time.

IF the main conclusion IS one of the bolded phrases, then find the answer choices that offer that option for the respective bolded phrase (first or
second). A significant number of BF questions can be answered correctly with this information ALONE. If there is only one choice that matches up
with the bolded main conclusion then you’re done. Mark it and move on.

Otherwise…

-Have a quick look through the choices to discover what the terms in play (see below).

(4) Return to the argument and determine the relationship between each bolded phrase and the argument’s main conclusion. Do they basically
agree with the conclusion of the argument? Does one but not the other? Neither? What other relationships occur to you?

(5) Return to the answer choices and use these relationships to discard at least two and probably three choices. Let’s have a review of key terms:

* Main Conclusion—a summary of the argument’s primary position;


* Intermediate Conclusion—a position utilized by the argument as a stepping stone in order to advance toward the main conclusion;
* Premise—a theory or proposition upon which an argument is based or from which a conclusion is drawn;
* Fact—information generally believed to be true OR known to be true—usually advanced as evidence to support a premise;
* Evidence—specific type of fact offered in support of a theory or premise;
* Context—a frame of reference of value in the interpretation of aspects of an argument or the argument’s components;
* Consideration—a factor (fact) to be taken into account in forming a judgment or decision;
* Position—a point of view or attitude about an issue or question;
* Assumption—a position or belief that is taken to be true, without proof;
* Principle—a basic or essential truth (stronger and broader than a fact).
* Judgment—an opinion formed from a consideration of the facts.

(6) Now, take each of the remaining choices one by one, matching similar parts of each answer choice to their respective BF phrase, then
discriminate between the dissimilar parts of each answer choice and their respective BF phrase. That should take you the rest of the way home.

Lets’s face a typical BF CR question:


“Environmental organizations want to preserve the land surrounding the Wilgrinn Wilderness Area from residential development. They
plan to do this by purchasing that land from the farmers who own it. That plan is ill-conceived: if the farmers did sell their land, they would sell it
to the highest bidder, and developers would outbid any other bidders. On the other hand, these farmers will never actually sell any of the land,
provided that farming it remains viable. But farming will not remain viable if the farms are left unmodernized, and most of the farmers lack
the financial resources modernization requires. And that is exactly why a more sensible preservation strategy would be to assist the farmers to
modernize their farms to the extent needed to maintain viability.”

In the argument as a whole, the two boldface proportions play which of the following roles?

A. The first presents a goal that the argument rejects as ill-conceived; the second is evidence that is presented as grounds for that rejection.
B. The first presents a goal that the argument concludes cannot be attained; the second is a reason offered in support of that conclusion.
C. The first presents a goal that the argument concludes can be attained; the second is a judgment disputing that conclusion.
D. The first presents a goal, strategies for achieving which are being evaluated in the argument; the second is a judgment providing a basis for the
argument’s advocacy of a particular strategy.
E. The first presents a goal that the argument endorses; the second presents a situation that the argument contends must be changed if that goal is to
be met in the foreseeable future.

Main Conclusion: “A more sensible preservation strategy would be to assist the farmers to modernize their farms to the extent needed to maintain
viability.” So, we didn’t luck out and get the answer just by having identified the main conclusion. No problem.

Terms in play:

A. (1) A goal (that the argument rejects)


(2) Evidence (as grounds for the rejection)
B. (1) A goal (that the argument judges as unattainable)
(2) Grounds (support for that judgment)
C. (1) A goal (that the argument judges as attainable)
(2) A refutation (of that judgment)
D. (1) A goal (with the strategies for attainment in question in the argument)
(2) Reasoning (for supporting ONE of the noted strategies)
E. (1) A goal (endorsed by the argument)
(2) A factor (effecting the timely attainment of that goal)

Back to the Bold-Faced phrases to determine their relationship to the main conclusion:

BF 1: Preservation of the Wilgrinn land (that’s a goal) => The plan is ill-conceived (not the goal, but the plan) => So, the goal is preservation of the
land (First BF) and that’s also part of the main conclusion => First BF is a goal with which the argument basically agrees => A and B are gone.

BF 2: Doesn’t really relate to the main conclusion, so

Return to the answer choices and consider second BF description in remaining choices—C, D, and E => E implies that the argument suggests
changing the approach to a goal; it says nothing about changing a situation of any sort => E is gone. That leaves us with C and D.

Read Choice C in its entirety:

(1) Says the first BF presents a goal that the argument concludes can be attanined. Not exactly—the argument actually concludes that a different
strategy is needed to attain the goal, not simply that the goal is attainable (so this part of this choice doesn’t match the argument).
(2) Says the second BF in the argument is a judgment disputing whether the goal can be attained. NO, definitely not—the second BF in the
argument suggests a reason why one strategy won’t succeed, but has nothing to do with whether the goal can be attained (this part of this choice
doesn’t match the argument, at all) => C is gone.

Now look at the remaining choice, to make sure it fits:

(1) Goal, strategies for achieving under consideration (YES).

(2) basis (judgement) for supporting an alternative to the earlier plan aimed at achieving the same goal—preservation of Wilgrinn land (YES).

Done—mark it and move on. That’s it.

Let’ know some basic terms to ace the BF CRs of GMAT:

Principle: something fundamental that we do not question. This would be somewhat stronger than a fact because it is not specific to a limited
number of cases but instead, apply to a broader range of scenarios (and often deeper in meaning). For instance, you will not talk about the principle
that crime is increasing in large cities. Instead, it is a fact which applies to large cities. However, you will talk about the principles of Physics or the
fundamental principles of Human Rights. Principles convey a stronger connotation than mere facts.

Fact: something taken as true at face value (stats, historical events)

Evidence: what is used to support a conclusion (examples, stats, historical events). Although these may include facts, it is usually stronger than
facts because they are direct elements needed for the conclusion to stand whereas facts are not necessary for the latter to stand

Pre-evidence: This is a bit of a stretch. It will not often be on the test but it seems very similar to "background" information as described below.

Background: Elements needed to put the evidence into context but which, as stand alone pieces of information, might not constitute what is called
an evidence necessary to arrive at a conclusion. For instance, blood tests performed on one thousand persons may reveal that 35% of those
persons were HIV infected. However, the background information could be that the test was performed in more under-informed regions of
the world where AIDS knowledge is at a minimum. As you can see, the fact that the test was performed in more under-informed regions is not in
and of itself an evidence because it does not allow us to come to a conclusion. Instead, the 35% stats, as a stand-alone piece of info, is what will
lead us to the conclusion we want. However, the background info is also crucial and cannot be omitted; it is required background info.

Consideration: Something which was taken into account or given some thought before arriving at the conclusion.

Premise: For GMAT purpose, Premise and Evidence are the same.
Assumption: Unstated information which will link the argument to a logical conclusion. Without this, the argument falls apart.

Inference: Something that might not be explicitly stated or proved. For instance, you may say that 95% of GMAT test-takers have over 340. We
can reasonably infer that Anthony will get more than 340 on his GMAT based on the fact given.

Argument: Central to every CR question is the argument. An argument is an ordered line of reasoning composed of premises, assumptions, and a
conclusion. Understanding the elements of an argument is essential to performing well in this section.

Conclusion:
The conclusion is the endpoint of the line of reasoning of an argument. Think of it as the result of the argument. The line of reasoning leading to a
conclusion is often where errors in logic are made. Conclusion can be defined as the last deduction or claim

One of the new question types in GMAT CAT is the bolded question in Critical Reasoning Section. In such argument, one or two sentences in
stimulus are bolded. A sample question looks like this.
The question following requires you to identify the logical relationship between the boldfaced sentences, or how it relates to a particular position
(the author agree or disagree).
Boldface question is totally new on computer-based test, but is easy to tackle. All you have to do is to understand the argument: identify the
conclusion, evidence, and the reasoning from evidence to conclusion. Sometime, you are required to critique the validity of the argument.

BF CRs Wrap-Up:
1. Identify the conclusion. Ask yourself what the conclusion is, what the author trying to prove, or what the author’s main point is.
2. Look for the evidence that the author uses to support or argue against a position.
3. Search for argument indicator to determine the relationship between evidence and conclusion.
Conclusion Indicators
So thus therefore as a result
consequently accordingly hence imply
conclude that follows that means that infer that

Premise Indicators
because since for as
If assume suppose evidence
on the basis of the reason is that may be derived from in that

Counter-evidence Indicators
actually despite admittedly except
even though nonetheless nevertheless although
however In spite of do may

Regards,
Cumic

=================================

Synopsis on CR

=================================

Notes by other

https://gmatclub.com/forum/my-notes-critical-reasoning-276903.html

CR

- Note taking / abbreviated terms


- GMAT sense of some common words [by empowerGMAT]
- Language Shift
- One specific example to generalization
- Time frame mismatch
- Out of Scope (OOS)
- Convoluted Options
- Different Question Stems
- Phrases make meaning twisted.
- Real world sense
- CR Strategy
- Eliminating bad answer options
- Using ‘cognitive dissonance’ in ‘Plan – Goal’ scenarios
- Introducing alternative plan to achieve goal
- Wrongly doubt the plan
- Comparing two groups
- Tackling Assumption question
- Tackling weaken question
- Tackling Evaluate question
- Tackling inference question
- Inference VS must be true question
- Tackling Fill in the blanks / Complete the passage / argument questions
- Prefer VS Purchase
- Justification for closing an unprofitable business unit
- Decision on ‘Profit’ , ‘Revenue’ etc
- Impact of inflation
- Pre-thinking on some scenario

Note taking / Abbreviated terms

IRC – irrelevant comparison


180
Many / several / more // less / increase
Wrap
Alt – alternative solution / plan
OOF – outside the framework discussed in prompt
Alt p– alternative plan
Amb – ambiguous
TF – Time Frame

GMAT sense of some common words [by empowerGMAT]

- Increase / Decrease / More / Less:


In GMAT, when we see the word ‘increase’, we should not assume it’s significantly increase. Increase is a
relative word. We don’t know how significantly increase it is. It could be lowest possible point increase
(.001%) that could still trigger the word increase. GMAC try to prey to our presumptions about the scale of
increases and decreases. We must be SUPER SKIPTICAL about ambiguous comparative term ‘increased’.

- Several
Max : Several is relative comparison word. Ambiguous. Is it just one, two or five? I don’t know whether there
is any consequence first. [empower Weaken Quiz Q4]

- Most
Max : statistically most means greater than 50 percent.

- Some

- Change
Belal : Change means increase / decrease – even it could be slightly / significantly.
Anonymous from BTG
All = 100
Most = 51 to 100 (“a majority”)
Some are not = 0 to 99 (also “Not All”)
Most are not = 0 to 49
Some = 1 to 100 (“at least one”)
None = 0

Stuart Kovinsky | Kaplan GMAT Faculty


"All" means 100%, but is conditional.
"Most" means more than 50%.
"Many" is an unspecified % and could just mean 1.
"Some" means "at least one".

http://lsatblog.blogspot.com/2011/02/lsat-numbers-all-most-several-many-some.html

Language Shift

- Prompt says, fish consumption in Colton is extremely low. [Empower CR Weaken Quiz Q1]
Option b says, Heavy consumption of O3 fatty acid results in serious side effects like this this & this.
Max : prompt says population in general / at large. Not the heavy consumers of O3. We don’t even know what
percentage of consumer O3 heavy consumers refer to. It might be literally just one guy. May be he just a fish
nut – who knows. This info ( heavy consumer) doesn’t matter, because we are not talking about these people /
segment.
- Prevention of alien species VS total alleviation of species’ harm [empower Strengthen Quiz Q1]
- Premise says that Malvernia is converting heating system from oil to natural gas. [OG13#72]

If domestic oil supply is going up and demand is going down, it seems almost inevitable that their reliance on imported
oil will decrease. However, note the limited scope of the first premise: they aren't switching ALL oil usage to natural gas,
only the fuel used in their heating systems. We actually don't know anything about the overall demand for oil. That is the
gap in the author's reasoning. Choice D exploits this gap: if the amount of oil used for non-heating purposes (generating
electricity and transportation) is increasing, then the overall demand for oil might still be increasing too fast for domestic
oil production to keep pace (even though production is increasing as well).

One specific example to generalization

- OE : The argument moves from a particular example, that is, the percentage of vehicles ticketed for exceeding
the speed limit that were equipped with radar detectors, to a generalization about the regular driving behaviors
of all drivers who have radar detectors in their vehicles. The reasoning links the example to the generalization
with an assumption. What can the assumption be? Only if the drivers ticketed in this instance are assumed to
make a regular habit of exceeding the speed limit can the conclusion be drawn that drivers with radar detectors
are more likely to do so regularly than drivers who are not ticketed. [OG13 CR 77]

Time frame mismatch

- The prompt propose to create a world class station that will put customer first. However, most of the citizen
are currently pleased with their system. So current satisfaction level is of little implication to this argument.
[empower Strengthen Quiz Q3]
- OG13 CR 77
Premise : According to survey report, proportionately greater percentage of ticketed drivers have radar
Conclusion : Drivers (having radar) are more likely to exceed limit regularly.
Galib: shifting of time frame in conclusion. Focus on the adverb of frequency.
Ron : passage equates being ticketed for speeding = with speeding regularly.

Out of scope (OOS) options

[b] Most children have other opportunities, in addition to after-school hours, in which they can choose
whether to play computer games or to interact with other people. [OG13 CR 48]
OE : The argument is limited to after-school hours or spare time.

[d] Formal instruction contributes little or nothing to children’s acquisition of communication skills.
OE : This could be false and the argument could still be sound; perhaps

[e] The mental skills developed through playing computer games do not contribute significantly to children’s
intellectual development.
OE : This could be false and the argument could still be sound as long as the intellectual development the
games contribute to does not contribute to the development of communication skills.

Lucero, Manhattan : Answer choices (B) & (D) give us reasons why students might or might not get
experience in interpersonal communication outside of their spare time, but they are both irrelevant to the main
conclusion. If I said Monday Joe was more productive than Saturday Joe, you wouldn’t be able to refute this
by saying there are five other days of the week that Joe could be productive. You should also notice that both
of these wrong answer choices do not differentiate between the two groups we are comparing. If both groups
are receiving formal instruction that contributes little or nothing to communication skills, there is an even
playing field for these two groups and the communication skills gained during children’s spare time would be
even more essential.

Our other goal in this comparison is to focus on the interpersonal communication abilities of children. Answer
choice (E) brings in a consideration other than communication skills. The argument is not concerned with
saying that computer games have no beneficial aspects, only whether the computer games will result in lower
interpersonal communication skills. If I said Monday Joe was more productive than Saturday Joe, you
couldn’t refute this by saying that Saturday Joe was happier than Monday Joe.

Convoluted options (intentionally making the sentence awkward for making the meaning passing difficult)

- (A) The frame was made from wood local to the region where the picture was painted. [OG16 CR 8]
- [e] Quarantining all tulu plants from horticultural companies at which any case of fungal rot has been detected
until those tulu plants can be tested for fungal rot [OG13 CR 28]
OE : This will not help overcome the problem. Such a quarantine program might lead horticultural companies
to start selling tulu plants only if they are less than 24 weeks old, thereby minimizing the chance of quarantine
by minimizing the chance of detection.
Galib: our focul point is NOT ensuring testing, BUT testing in a way so that fungus is detected reliably.
- (B) It has been many years since the restriction led to the hunting season for snow geese being closed earlier than the scheduled
date. [EP1 OA]
Chetan : It tells us that restriction came into play long time back. Scheduled date here means WINTER.
Karishma : option B tells you that the restrictions are ancient and are meaningful today. They haven't come into effect for many years.
This means that for many years, the 5% limit has not been reached.
Fluke : "B" tells us that the hunters are not able to hunt even 5% of the snow geese population before the hunting season.
Thus, lowering the restriction, or in other words, increasing the percent decrease to any value greater than 5 won't help at
all.

[Ryan Jacobs, mgmat] The reason I bring this up is because when I teach GMAT Critical Reasoning in my classes, and I
ask my students why they eliminated an answer choice that they thought was incorrect, the single most common reason
they give me is some variation of the following:
“They didn’t mention that in the argument.”
“That wasn’t something the author of the argument brought up.”
“That seems random/like it came out of nowhere.”
Yet when you think about it, all those things are true of the correct answer to the question above! If, right now, any of
these are the reason you’re eliminating answer choices, then you’re often eliminating the correct answer.
Here is my super-quick fix. Stop saying “that answer choice wasn’t mentioned, so it’s wrong.” Start saying “that answer
choice doesn’t affect the argument’s conclusion, so it’s wrong.” That’s it! You’ll improve your GMAT Critical
Reasoning accuracy overnight.

Phrases make meaning twisted.

- Gortland has long been narrowly self‐sufficient in both grain and meat.

Different Question Stem

Which of the following, if true, would cast the most doubt on the effectiveness of the authority's plan to finance the
proposed improvements by increasing bridge tolls? [Q. Pack 1 CR 1]

OE : Any financing plan is based on estimates of costs and revenues, and any factor that significantly increases costs or lowers revenues threatens
the effectiveness of that plan.

-----------

The argument is vulnerable to criticism on the grounds that it gives reason to believe that it is likely that … [GP2 CR 17]

Stuart Kovinsky | Kaplan GMAT Faculty

"vulnerable to criticism" in the stem indicates that we have a logical flaw question. Flaw questions are fairly rare on the
GMAT, but they do show up occasionally. However, since flaw questions are very closely related to weakening
questions, the most common CR question type, there's no need to panic when you see one.
The main difference between flaw and weakening questions is in the types of answers that we see. For flaw, the answers
are general statements about the logic of the argument, one of which accurately characterizes the problem; for
weakening, the answers are true facts about the world, one of which makes us doubt that the conclusion is true.

----------------

OG13 CR 29

Which of the following, if performed by the government ministry, could logically be expected to overcome the
problem with their plan to test for the fungal rot?

GP

The objection implied above to the productivity measure described is based on doubts about the truth of which of the
following statements?

Which of the following conclusions about Country Z’s adversely affected export-dependent industries is best supported
by the passage? [OG13 CR 66] must be true Q. (inference by rich)

Real world Sense:

- (D) When tolls were last increased on the two bridges in question, almost 20 percent of the regular commuter traffic switched to a
slightly longer alternative route that has since been improved. [Q.Pack 1 CR 1]

Q. Because choice D cited a past fact about how people responded to the strategy, it has no relevance to the current
situation.

Ron: ... but you're looking at essentially the same situation again, so, yes, it's relevant.

remember, you have to use real-world sense on these problems.


if you saw this situation on the TV news, and your friend said, "Hey, remember what happened last time there was a toll hike?" ...
you would clearly not say "that's irrelevant". because, in this situation, it's not.

there's not going to be a "rule" for whether this sort of thing is relevant; you're going to have to judge each individual situation
independently.
...which is the whole point of these problems in the first place.

- If fee / tax imposed,


a) People will avoid the subject matter (e.g will avoid the road itself)
b) people will try alternatives to by-pass it. (eg. Will follow a different route)
c) Will forward / share the fees to other body to whom the fees is insignificant. (superstore will transfer to poly
cost / penalty to customers who may find the access cost insignificant.
Q.Pack 1 CR#1, (tristate authority plans to impose toll to finance railway bridge
- Overutilization hampers the normal trend.
OR, After implementation of a plan, if the plan stops working (start reverse working) after a certain period of
time, we can assume that after a certain time, the plan may start backfiring or trigger a destroyer effect.
Q.P.1 CR 05

- Financial incentive is more effective incentive to make desired outcome. [OG13 CR 46]
(D) The beauty of county parkland is an important issue for most of the county’s residents.
OE : The argument need not assume this, and nothing in the argument indicates that it does. Financial incentives
could be enough to make the desired outcome happen even if residents are indifferent to the parkland’s beauty
- Does managerial EXPERIENCE a necessary matter in regard to QUALITY of management? No. (max)

- When the prompt concludes that county’s office will be able to retain best trained analysts by paying more
competitive salaries, it (argument) assumes that the county’s office has the capability in its annual budget to
accommodate more competitive salaries. [empower strengthen Quiz Q4]
- Given that the caterpillars eat only milkweed leaves dusted with insecticides, but we do not know when, in the
course of their development, they do so. Monarch butterflies would be at risk only if the caterpillars were to eat
the milkweed leaves when those leaves had maize pollen on them. [OG13 CR 46]

- Deer ticks are not deer. They are tiny insects that live on the blood of deer and other animals. The question
assumes that you know that a tick must feed on a person in order to infect the person. [marty murray while
explaining a CR from EP2]

- Galib: the reason for not completing antibiotic course that patient recover from illness after taking this medicine
before completion course of medicine. Then they think that as they got recovered, no need to take further
medicine. [EP2]

- Different kinds of crops may react differently to the same climate conditions.
Neighboring x and y country has experienced the same climate condition. If two countries grow different kinds
of crops that may react differently to the same climate conditions. [OG13#73]

- In most of the cases, in case of comparison, the comparison is logical, but the compared issues are not
comparable. For example, in this case, [OG13#73], the crops are different and thus may react differently in same
climate condition.

- Consumption = Production + Import


- [OG13#73]To the stimulus. On first reading, it seems like a pretty reasonable argument. The Malvernians have relied
on imported oil in the past, but they may be able to turn that around, for two reasons:

1. They're switching from oil to natural gas, of which they have a plentiful supply.
2. Domestic oil production is increasing.

If domestic oil supply is going up and demand is going down, it seems almost inevitable that their reliance on
imported oil will decrease. However, note the limited scope of the first premise: they aren't switching ALL oil usage to
natural gas, only the fuel used in their heating systems. We actually don't know anything about the overall demand
for oil. That is the gap in the author's reasoning. Choice D exploits this gap: if the amount of oil used for non-heating
purposes (generating electricity and transportation) is increasing, then the overall demand for oil might still be
increasing too fast for domestic oil production to keep pace (even though production is increasing as well).

Manhattan CR Strategy :

Ready? Before we dive into the GRW process for CR, you might have already learned the 4-step process we use for CR questions:

Step 1: Identify the Question


Step 2: Deconstruct the Argument
Step 3: State the Goal
Step 4: Work from Wrong to Right

If you’re now wondering why I’m introducing a new process … I’m not! Those 4 steps actually go right along with the GRW model; we just didn’t
make that model explicit to you before.

Okay, where do you think you should Glance first on CR?

RON on how to crack CR

My best two answers here are...


... Name the speaker,
... Imagine the argument as an actual conversation with that speaker.

If you can successfully formulate this problem as a conversation, then it will be obvious that "diners on stools typically do xxxx" is a
generalization -- because you'll understand why the conversation is happening. The conversation is happening because the Hollywood
doesn't have stools at all right now, and Joe is arguing that some should be installed.

(Framing arguments as conversations will also do away with the useless (and counterproductive) habit of looking for "rules" that
will solve CR problems. After all, no one looks for rules when they have conversations! You just think about what the other person
is saying.)

-------------

most importantly—

• if this problem still confuses you after a while, just walk away from it. then come back to it after a couple of weeks.

• at that point you'll have 'new eyes', and you might find that you suddenly have a better intuition for what's going on here.

• if the problem still confuses you at that point, then just...


...smile,
and
...ignore it.
this single problem is, quite clearly, of little importance overall. (i think it's the only GMAC problem ever in this particular format.)
do not let it distract you from more fundamental concerns.[source: GP2 CR 17]

------------

also remember—you should NEVER have to "plug in" NUMERICAL VALUES for a VERBAL question!
really... don't do this.

there's an entire section of this exam where you may actually want to do things like that. that's the QUANT section!

in CR, if numbers are involved, you should ALWAYS just think in very general terms about the numbers/statistics involved and how they
interact.

for instance, in choice D here, you just need to realize that "less expensive meals" don't mean lesser profits—because the customers are
leaving earlier!
you just need to think about the fact that people spending less individually could still lead to greater revenues overall, IF they are coming
and going faster. "plugging in numbers" is not going to help you understand this. (in fact, you'd have to understand the concept ALREADY
in order to "plug" the relevant numbers!)

……

Lucero : Let me digress for a moment and say that the difference between an easy and difficult Critical Reasoning
problem isn’t in the topic or question type, but how obvious some of the wrong answer choices are. On easier questions,
there tend to be 3-4 bad answer choices, on more difficult problems there aren’t as many of these.

-----------------------

Eliminating bad answer option

Lucero : If I said Monday Joe was more productive than Saturday Joe, you wouldn’t be able to refute this by saying
there are five other days of the week that Joe could be productive.

Our other goal in this comparison is to focus on the interpersonal communication abilities of children. Answer choice
(E) brings in a consideration other than communication skills. The argument is not concerned with saying that computer
games have no beneficial aspects, only whether the computer games will result in lower interpersonal communication
skills. If I said Monday Joe was more productive than Saturday Joe, you couldn’t refute this by saying that Saturday Joe
was happier than Monday Joe. [OG13 CR 47]

--------------

Using cognitive dissonance in Plan – Goal scenarios

Patrick Tyrrell, MGMAT instructor, [apr’18] https://goo.gl/xhnAqV

When we read arguments and plans in Assumption, Strengthen, and Weaken questions, there usually isn’t any inherent
tension. In an argument, the author is leading you into her Conclusion. When we read a Plan, it’s designed to lead to a
certain Goal.

It’s up to us to create the tension. It’s gonna take all the stubbornness we’ve got. Are we ready for it? Of course we
are. After all, how many times have we acquiesced to an opponent during a political debate on Facebook or Twitter?

When we read Arguments, we frame the tension by thinking:


GIVEN [this evidence], HOW CAN I ARGUE [the opposite of the conclusion]?
When we read Plans, we frame the tension by thinking:
GIVEN [this plan], HOW CAN I ARGUE [we won’t achieve the goal]?

Every correct answer I’ve seen to questions based on Plans is addressing one of these four concerns:

1. Will the Plan backfire in some way that will thwart the original Goal?
2. Can we assume the skill (ability/feasibility) and will (motivation/incentive) of those involved in the Plan?
3. Does this Plan address a significant source of the problem it’s meant to solve?

Try some of these GMAT Critical Reasoning questions about Plans.

1. Find the Plan and the Goal.


2. Try to frame the tension of “GIVEN [this plan], HOW CAN I ARGUE [we won’t achieve the goal]?” before
you see answers. Jot down or type up any ideas you have.
3. Examine the choices and select your answer.
4. Determine whether the correct answer dealt with Backfire / Skill & Will / Significant Source of Problem.

2018 OG: diag66, diag 77, diag 80, 546, 554, 559, 563, 566, 575, 576, 577, 578, 585, 587, 589, 590, 592, 597, 600, 604,
605, 610, 617, 620, 643, 646

------------------------------

Wrongly doubt the plan

When asks about the effectiveness / result / intended effected of the plan, the wrong options create doubts on the
implementation of the plan.

Q. Pack 1- CR 1

(A) Before the authority increases tolls on any of the area bridges, it is required by law to hold public hearings at which objections to the
proposed increase can be raised.
(E) The chairman of the authority is a member of the Tristate Automobile Club that has registered strong opposition to the proposed toll
increase.

-----------------------------
Alternative plan to achieve goal

If Plan – Goal strengthen question, an alternative solution, even if it’s preferable, is considered Out of Focus.

---------

----------------------------

Comparing two groups

- In case of comparing two groups / issues, if conclusion is drawn on one particular issue / group, then other issue
/ group will play the expected role (as per conclusion assumed). This is assumption. [OG13 CR 47]

In conclusion, it’s said that ‘children who play computer games have less communication skills than other
children’. That means it’s assumed in conclusion that the other children have better communication skill.
- Another question GP2.1 file ; possibly within no. 15 to 17. Almost same context.

---------

Applying combination of two methods

- Before deciding effectiveness of a particular method ( / solution / option/ etc) between two, we need to consider
the combination of two method – i.e. whether combination might give the better result than choosing any single
option. [OG13 CR 51]
How to tackle Assumption Questions

- Best discussion on Aristotle prep guide Pg – 40


- OG13 CR 46, 77
- An assumption is a statement that needs to be added to the premises for the conclusion to be true. [Stacey]
- as always, you can think of an assumption as something that not only strengthens or reinforces the conclusion, but also is necessary to
draw that conclusion. [Ninja]
- two big assumption :
capacity i.e. outcome is even possible and
timing i.e. outcome will happen at the on time. [OG13 CR 47]
In case of comparing two groups / issues, if conclusion is drawn on one particular issue / group, then other issue
/ group will play the expected role (as per conclusion assumed). This is assumption. [OG13 CR 47]

In conclusion, it’s said that children who play computer games have less communication skills than other
children. That means it’s assumed in conclusion that the other children have better communication skill.
-

----------------

How to tackle Weaken Questions

- Best discussion on Aristotle prep guide Pg – 59


- Causality [X causes Y]

We can weaken the causality by any of the following :


i) Z causes Y [2nd cause]
ii) No X  Y happened
iii) X  No Y
iv) W causes both X and Y

- Study Report
i) Lack of validity / authenticity of study report
ii) Target / focused situation is NOT representative of the scenario discussed under Study Report.

- Weaken scenario
Based on examples / trend, a decision or suggestion is taken on a different object. Here the assumption is that
scenario of examples / trend is representative of the discussed object. [Empower CR Quiz Q3]

- Trap Answer option to weaken questions:


A classic trap is to provide a better solution / an alternative solution. There could be a thousand of alternative
proposal. How can that weaken a GIVEN proposal?

How to tackle Evaluate Questions

- Best discussion on Aristotle prep guide Pg – 48


- Evaluate the Argument question indirectly asking you to identify the assumptions. So try to predict the
assumptions. (Aristotle prep)
- Sample Question Stems :
Which of the following consideration will be most useful to assess the likelihood of success of the
entrepreneur’s plan?
How to tackle Inference Questions

- Best discussion on Aristotle prep guide Pg – 71


- Don’t assume anything and don’t try to draw any real-world conclusions!
- Trap Option: ones that might be true or are even pretty likely to be true but don’t have to be true. You’re not
looking for a “reasonable” conclusion. You’re looking for a “must be true” conclusion.
- The most common trap on this type is something that could be true and might even be likely to be true but does
not absolutely have to be true.
- Be careful about one thing: Inference questions can sometimes include the words ‘strongly supports’ and this
language also shows up in Strengthen questions.
- Our goal is to find an answer that must be true given some or all of the information presented in the argument.
Note that we do not need to use all of the information.
- On Inference questions, the goal is to find the answer that must follow from the given information in the
argument. The most common trap on this question type is an answer that goes too far—what I call a real-world
conclusion. A real-world conclusion might be true, and even might be likely to be true, but it doesn’t have to be
true…so it’s not the right answer on the GMAT.
- Don’t try to predict the answer. [aristotleprep]
- Max : We need to find an option that is 100% provable only from the information from the prompt. Look to
eliminate classic GMAT inference trap : a reasonable but unsupported option.

- ‘Should’ in option of inference question.


On inference question, the word ‘should’ is always universally incorrect. Why? It requires so much more proof
than the prompt offer to justify an option that should be taken. Just be skeptical if any option in inference
question use that word ‘should’. Just think about it. It is extremely hard to absolutely proof that someone should
do something.

- Sample Question Step


The statements above, if true, most support which of the following ?
If the facts stated in the passage above are true, a proper test of a country’s ability to be competitive is its ability
to [OG13 CR 60]

Inference VS must be true question


- [GMATpill] Inference vs must be true are quite similar concepts. But "must be true" is something that definitely cannot
have a false side to it - otherwise it violates "must" be true.

Inference - this is a statement (usually among the answer choices) that can be supported by information in the passage.
Thus, somewhere in the passage, we have a supporting statement or phrase that would support the "Inferred" statement.
Because it is supported by something in the passage, it is called an "inference".

An inference is not necessarily something that *must* be true. Here, you deal with something that can be supported by
information in the passage, but might not be a true statement ALL the time. So the key point to pay attention to is that
*must* be true has absolutely no exceptions - whereas an inference is more flexible and *only* requires some support from
somewhere in the passage.
- [Dimitry Farber] If a CR or RC question asks you what "must be true" or wants you to "draw a conclusion," it's asking for an
inference. This is anythingthat must be true based on the text in question. It does not need to be a "main point" or tie
together all of the statements in argument/passage.
If a question asks you for the conclusion of an argument or the main point of a passage, it wants you to identify what the
author's message is. This is a totally different task. In CR, the conclusion will be explicitly stated in the argument. In an
RC passage, the main point/conclusion may not be stated precisely, but should be a distillation/summary of the author's
message across the passage.

In short, you need to distinguish between questions that want you to draw a conclusion (conclude something yourself) and
questions that ask you to identify the conclusion (simply identify what the author is getting at). In the former case, your
conclusion should be logically valid and require no assumptions. In the latter case, you're simply describing the author's
opinion, and this opinion does not have to be valid. Reading the question carefully up-front and making sure you
understand what you're being asked to find will do wonders for your accuracy and efficiency.

Fill in the blanks / Complete the Passage / argument Questions


- Stacey : What kind of question type is it? Actually these fall into one of the regular types that you already know:
Strengthen, Weaken, Find the Assumption, Inference, and so on. The trick is that it’s a bit harder to tell which
type.

The majority of Complete the Passage questions are Strengthen. The second most common category is Find the
Assumption. Typically, if you see the word because or since right before the underline (as in this problem), then
you probably have a Strengthen question. The word because (or since) indicates that the correct answer will add
a piece of evidence to support some statement. You’ll need to read the argument to be sure, but you can have a
pretty strong hunch.

Prefer VS purchase
- Preference doesn’t say us much regarding purchase behavior. You can prefer flying a private jet, but mostly you
have to fly with a commercial airline. So the iris could prefer a laptop with faster chips, but might buy some
other laptops which have a slightly slower processor but a much longer battery life or which cost much less. [ref:
Aristotle prep CR guide : Page 55]
Galib: purchase decision doesn’t depend on a single attractive feature rather depends on a bag of mixed features
depending on the product. Common examples of such features are cost, quality, longevity, maintenance,
promotion, guarantee etc.
- Experts don’t like something… who cares this in GMAT?

Justification for closing an unprofitable business unit


- DavidG@veritasprep : In the argument about closing unprofitable roads, the logic comes down to considering the effects of
one part of an economic system on another. For example, if a video game manufacturer was taking a loss on consoles, but
turning a large profit on the sale of games, it would obviously be flawed to claim that the manufacturer should phase out
production of consoles, as the unprofitable portion of the business is necessary for the profitable portion of the business to
function. How profitable the business is overall, is irrelevant.
Same Scenario : Verbal Review 2016 CR03,

Decision on Profit / [comparing profit of two entities]


- Profit = Revenue – Cost
In general, either revenue should be higher OR Cost should be lower and vice versa to make profit larger.
In real world, there are multiple factors that affect Profit, Revenue and Cost. Consider other factors in play in the
argument.
[source : Galib. Q : OG13 CR 27]

- The profit margin itself does’t shed any light on the overall profitability.
- For comparing profit margin of two company, it requires to assume that both are equally capable of generating a
comparable profit. See below example :

Impact of inflation

Inflation is calculated by comparing CPI of two different points.

Inflation doesn’t impact similarly to all people.

Inflation can be misleading by usual fluctuation of price which is not typical in the whole year. E.g. charismas / eid
period doesn’t reflects the normal shopping habit. In that case, inflation doesn’t reflect abnormal comsumption.

Question: Does the inflation rate calculated by CPI correctly reflect the my / a certain group’s inflation rate, based on the
specific goods I / a particular group regularly consume?

Well, that depends on my market basket of goods. If u don’t spend any money on housing, then its not a true reflection of your
inflation rate. It might either overstate / understate my inflation rate.

Different groups can have different inflation rates.

https://www.economicshelp.org/macroeconomics/inflation/problems-measuring/

Problems in measuring inflation :


Inflation is a measure of changes in the cost of living. It is calculated by using statistics such as Consumer Price index CPI, retail
price index RPI. The process for measuring inflation is broadly

 Creating a weighted basket of goods – depending on frequently goods are bought


 Measuring monthly changes in prices.
 Creating an index from the price change multiplied by the weighting of the good.

Difficulties in measuring inflation include

1. Changes in the quality of goods. Changes in the quality of goods mean that price rises may not reflect inflation, but just the
fact it is an improved good. For example, computers have many more features than 10 years ago, so it is difficult to compare
prices because they are effectively different goods.
2. Shrinkflation. It is also possible goods can deteriorate in quality and size. For example, the price of vegetables may stay the
same, but if the size decreases, the price per gram effectively rises. Shrinkflation has often been a response to rising cost-
push inflation – firms reduce the size of chocolate bars rather than increase the price. Inflation measures may not pick up on
this marginal decrease in size.
3. One-off shocks may give a misleading impression. For example, a rise in oil prices will lead to higher inflation. But, this
rise in prices may just be temporary. Tax changes have a similar effect.
4. Different groups can have different inflation rates. Rising electricity and gas prices may affect old people more than
young people. Therefore, old people could have a higher inflation rate than the national average. This is important if
pensions are index linked because their cost of living may rise more than prices causing a decrease in living standards.
5. Basket of goods can become outdated. In a fast-changing economy, goods people are buying is frequently changing.
Trends may cause people to be buying new technology or in different places – and the traditional basket of goods can fail to
keep up. For example, if there is a rise in internet shopping, inflation measures should give a higher weighting to online
prices, but it takes time to update the basket of goods and which prices should be counted.

-------------------------------

Pre-thinking on some scenario

[source : OG13 CR 24]

Govt plans to increase 20% govt provided pension to elder persons win 65 years old to make them financially better off.. The
attainment of the goal may be hampered by
- Increase of annual rate of inflation since pension increase
- For some reasons, increased pension has not been duly received by the eligible elder persons.

------------------------------------------
[source : OG13 CR 27]

If we need to compare the profit of two automobile insurance companies of two cities, the following factors need to be considered :
- Similar cars / vehicles
- Comparable drivers in terms of skills and experience
- Average number of accidents of two cities.
- Cost of repairing collision damage.
- Condition of roads and streets
- Profitability issues of two companies – tax, efficiency, any other low cost factor etc.

------------------------------------------
[source : OG13 CR 00]
=================================

Important Articles on CR

=================================

Mapping for CR Questions :

Part 1: Question-by-Question Classification – 13th Edition


This section contains a reverse lookup of the The Official Guide for GMAT Review (13th Edition) questions referenced according to the
topic/chapter in PowerScore Critical Reasoning Bible. You can use the reference to work on specific types of questions that you find challenging.
For example, if you wish to work on Must Be True questions, do questions 26, 54, 60, etc. from the Official GuideCritical Reasoning section.
---------------------------------------
Must Be True Questions: 26, 54, 60, 66, 74, 103, 104, 105
Main Point: 38, 91
Assumption Questions: 12, 21, 31, 33, 35, 39, 41, 45, 46, 48, 55, 59, 65, 69, 75, 77, 81, 83, 93, 101, 106, 109, 113
Strengthen Questions: 1, 5, 11, 14, 16, 17, 19, 22, 23, 29, 30, 40, 50, 52, 53, 56, 64, 67, 80, 95, 96, 102, 108, 111, 118, 120, 121
Resolve the Paradox Questions: 3, 6, 9, 13, 24, 44, 49, 57, 61, 86, 92, 94, 99
Weaken Questions: 4, 7, 25, 32, 37, 43, 58, 62, 71, 73, 79, 82, 87, 88, 90, 97, 107, 112, 115, 117, 119, 122
Method of Reasoning/Flaw in the Reasoning Questions: 2, 8, 18, 20, 28, 34, 51, 63, 76, 78, 84, 85, 89, 98, 100, 116, 123
Evaluate the Argument Questions: 10, 15, 27, 36, 42, 47, 68, 70, 72, 110, 114, 124

Part 2: Question Type Frequency


---------------------------------------
Type Number Percentage of whole
---------------------------------------
Must 8 6.45%
MP 2 1.61%
Assumption 23 18.55%
Strengthen 27 21.77%
Resolve 13 10.48%
Weaken 23 18.55%
Method 12 9.68%
Flaw 4 3.23%
Evaluate 12 9.68%
FIB 8 11.29%
Except (X) 2 2.42%
_________________

Thanks,
Prashant Ponde

----------------------------------------

OG13 CR 19 : strengthen. Shadow ‘Cause and effect’

OG13 CR 23 : Correlation VS Causation


OG13 CR 24 : Resolve the paradox

OG13 CR 26 : Inference / Conclusion

OG13 CR 46 : Defending assumption

----------------------------------------------------

CR strategy :

Ron : i personally don't like diagrams in CR because, in my experience, "diagramming" an argument makes it much more difficult to think
in simple real-life terms ("have a conversation" / "insert myself into the situation" / "explain to a 9-year-old").

Regarding how to review :

Tim : One approach would be to just let some of those super-tough problems go. But if you want to learn how to be more successful at
them, the key is in the review process. After you're done answering a set of questions, set aside 3-5 minutes for every question to review,
whether you got the question right or wrong. Then focus on really understanding what makes each wrong answer wrong. The more
thoroughly you analyze the wrong answers and figure out what the GMAT does to create wrong answers, the easier it will be to avoid
picking wrong answers on the test.

Ron : for the problems you truly find challenging, '3-5 minutes' is not nearly enough.

when you review a CR problem that you found truly challenging, you should do ALL of the following:

• Make an analogy for the problem.


if you've read any decent number of my posts on here, you'll notice that i very rarely discuss CR problems by talking about the actual
problems themselves—i almost always create analogies to facilitate understanding. if a CR problem is hard, the difficulty ALWAYS
comes from the specific content/words/topic, and NEVER comes from 'logic' or 'structure'.
if you spend the time to come up with a GOOD analogy—something that may take quite a while—you may find that it becomes trivially
easy to understand how the original problem works.

• Specify exactly why the wrong answers are wrong.


remember, ALL of the wrong answers are COMPLETELY wrong. if you think that one of the wrong answers is just 'worse than the correct
answer', then, nope—more review is in order.

• if you think you've discovered some sort of pattern, explore other problems for it.
e.g., i wrote this post an hour or two ago, about 'if':
https://www.manhattanprep.com/gmat/foru ... ml#p117071
the student who asked this question—and YOU, if you find the same issue challenging—should now look for the 'if' pattern in OTHER
problems.
by doing so, you'll be able to...
...see whether it's a real pattern; (if the 'pattern' is broken by other problems then it's not a patern, and you're back to the drawing board)
...see how frequently it (or similar stuff) is tested;
...cement your understanding.

this process could easily take over an hour for a single problem. but, remember, the name of the game is quality, not quantity.

---------------------------------------------
Important links on CR

Sampling in Official GMAT CR Questions


https://gmatclub.com/forum/article-representative-sample-a-concept-tested-in-gmat-cr-158832.html?fl=similar

GMAT Club Verbal Focus: The CR Hack For 75% of CR Questions


https://gmatclub.com/forum/gmat-club-verbal-focus-the-cr-hack-for-75-of-cr-questions-229247.html

-----------------------------------------

Guideline for critical reasoning

https://www.manhattanprep.com/gmat/blog/2013/06/19/the-master-resource-list-for-critical-reasoning/

The Master Resource List for Critical Reasoning


by Stacey Koprince Jun 19, 2013

Which Critical Reasoning question type drives you crazy? Boldface? Find the
Assumption? Inference?

I’ve put together what I’m calling the Master Resource List for Critical Reasoning. A
couple of disclaimers. First, this list includes only free resources, no paid ones. There
are a lot of good resources out there that cost some money—they’re just not on this
list!

Second, this list for Critical Reasoning is limited to my own articles. I’m not trying to
claim that only my articles are good enough to make such a list—far from it. I’m most
familiar with my own articles, so that’s what I’m using. (And, okay, I will admit that I think the Manhattan Prep CR

process is the best one out there. But I’m biased. )

The Critical Reasoning Process


Before you dive into individual question types, it’s critical to know the overall CR process. A few key notes:

 There are 4 major and 5 minor question sub-types* and each one has its own particular technique details.
 Your job is to learn the overall process/strategy for CR as well as the techniques specific to each question sub-
type.

*Every now and then, a question pops up that doesn’t quite fit one of the 9 main categories. There are exceptions to
every rule in the universe.
In order to master Critical Reasoning, you should be able to answer the following questions about each question type:

1. How do I recognize this question type?


2. What kind of information should I expect to find in the argument, based on this question type? What kind of
information is going to be the most important?
3. What is the goal for this question type? What characteristics must the correct answer have?
4. What kinds of traps will be set for me? What are the common wrong answer types for this question type?

The Assumption Family


Assumption Family questions always contain a conclusion. This group consists of five sub-types:

Find the Assumption: What does the other assume is true when drawing the conclusion? Want to try another?

Flaw: The flip• of Find the Assumption. The author assumes something, but that thing might not be true. What is
the flaw in the author’s reasoning?

Evaluate the Argument: What information would help to determine whether the conclusion is more or less likely to be
valid?

Strengthen the Conclusion: What new information would help to make the conclusion a little more likely to be true?

Weaken the Conclusion: What new information would help to make the conclusion a little less likely to be true?

The Evidence Family


Evidence Family questions really don’t have conclusions (never big conclusions, like the Assumption arguments, and
usually no conclusions at all). This group consists of two sub-types:

Inference: Given the information in the argument, which answer choice must be true?

Explain a Discrepancy: The argument contains some surprising information or outcome. Which answer choice
provides some new information that clears up this surprising situation?

The Structure Family


Like Assumption questions, Structure questions do contain conclusions. The answer choices are usually in more
abstract• form, discussing characteristics of pieces of the argument.

Describe the Role*: aka boldface. The boldface portion(s) plays what kind of role in the overall argument?

Describe the Argument: These are a variant of the boldface question and they’re so rare that I don’t have an article for
you. If you’re really worried about these, you can take a look at our CR Strategy guide—but my best advice for you is

not to worry about these.

*Note: this Describe the Role article is old enough that it doesn’t use our new• CR process, introduced nearly 2 years
ago. I’ll do another in an upcoming article!

What Do I Really Need to Study?


The four major Critical Reasoning types are Find the Assumption, Strengthen, Weaken, and Inference. The majority of
your CR questions will be in one of those four categories. If you’re going for up to about 75th percentile on Verbal,
concentrate just on those.

Of the minor types, the most common are Discrepancy, Describe the Role, and Evaluate. If you want to break the
75th percentile on Verbal, then also take a look at those three minor types, but spend more time on the major types. If CR
is your weakest Verbal area, you can also skip whichever of those three minor types is hardest for you—some people
really hate boldface questions and others think Evaluate questions are the worst.

If you’re looking to break the 90th percentile on Verbal, then you have to study everything. You can still pick one minor
type as your I’ll guess/bail quickly if I have to• question type, but you have to try it first.

Great, I’ve Mastered Critical Reasoning!


Let’s test that theory, shall we? After you’ve studied all of the above and you feel pretty comfortable with CR, try this
problem out. I’m not even going to tell you which type it is (in fact, that is one of the things that makes this problem
very hard—what is it, in the first place?).

If you struggle with it, don’t get discouraged. It is a very challenging problem. Instead, use it as an opportunity to get

even better!

Ask Dr. David: Critical Reasoning in the Eyes of an Expert

https://www.veritasprep.com/blog/2013/02/ask-dr-david-critical-reasoning-in-the-eyes-of-an-expert/

1) Narrow your focus. Critical reasoning (and, in fact, the whole verbal section) is about narrowing your focus. You are given
lots of information but not all of it is at the same level of importance.
The Veritas Prep Critical Reasoning book demonstrates how to break the argument down into context, evidence, and
conclusion.

Clearly your attention should be focused on the evidence and the conclusion. This is where the path to the correct answer lies.
But what role does the context play? Why is the context there if it is not something to focus on?

There are actually two roles that the context plays:


 Fairness – the context is there to ensure that the critical reasoning section is not a vocabulary test. It ensures cultural
and linguistic fairness so that everyone has a chance to answer correctly if they apply the right logic.
 Distraction – the context is also there for a not so generous reason, many of the incorrect answer choices are based on
focusing too much on the context.
By narrowing your focus, you can avoid most of the trap answers that catch so many test takers. Obviously you should not
bring in outside information, but you also should not give too much of your focus to the background information either. It is the
evidence that must lead to the conclusion.
2) Stop and gather yourself and notice anything that is not quite what was expected. Many people just push through a
critical reasoning stimulus and fail to really take advantage of what is there. An expert gathers himself after each phrase or each
sentence just the way you should on Reading Comprehension after each paragraph!
An argument generally is constructed with the context first, then the evidence and the conclusion is often the last part of the
argument. By stopping and gathering yourself you are able to see the argument form in front of you.

There are two big things to look for as you read, each of these will be easier to spot if you pause and gather yourself.

 Shifts in language. You will often hear experts talking about shifts in language. These can be from one noun to
another, for example, from “physical growth” to “maturity.” Clearly these are not the same thing – a person can be
fully grown physically without yet being mature – and you can see that the Flaw, Assumption, Weakness, or
Strengthen answer would likely be something to do with the difference between “being fully grown physically” and
“being mature.”
 Unnecessary and unexpected wording. This often involves adjectives and adverbs. For example, maybe the
argument is doing a fine job of telling you that one cause of a recession is that wealth is too concentrated in a society.
Yet the conclusion in the argument says “the ONLY cause of recession is that wealth is too concentrated…” Now you
know that the Flaw or Assumption or Weakness, etc. has to do with this use of “only” – a use which is not supported
by the evidence.
If you can internalize these things you will be doing most of your work on Critical Reasoning as you read the argument itself
and as you analyze it. You will know what to focus on and may even have a very strong idea of what the answer must be before
you even move to the answer choices. That is seeing critical reasoning the way an expert does.

-----------------------------------

Inference Vs Assumption – GMAT Critical Reasoning


by Rajat Sadana on January 14, 2015

The purpose of this article is to clearly bring out the difference between the Inference and Assumption so that any source
of confusion can be eliminated.

Some students are confused between inference and Assumption because the phrase “Must be true” is used in both
question types. We have also seen that some students do face problems in identifying one from the other. Let’s begin
with the definitions of both the terms: Inference and Assumption.

What is inference in Critical Reasoning


An inference is a piece of information which can be logically deducted from the given set of statements. In GMAT, an
inference could be a logical deduction from a single statement or from a combination from two or more statements.

Let’s take an example:

Everyone who reads this article will be more informed about Inference and Assumption. Rahul read this article.

Inference: Rahul is more informed about Inference and Assumption


This is an example of a combination inference where two or more statements are combined to produce the correct
answer choice. In this specific case, the first statement lays out a general rule of the form: If X, then Y i.e. If one reads
this article, then he will be more informed about Inference and Assumption.

The next statement about Rahul states that ‘X’ statement is satisfied in case of Rahul. Therefore, combining these
statements, we can make an inference that Rahul is more informed about Inference and Assumption.

What is assumption in Critical Reasoning

An assumption is a hidden or an unstated premise. There are two keywords in this definition: “hidden” and “premise”.

 Being a premise means that the assumption must be true for the conclusion to hold true
 “Hidden” means it cannot be logically derived from existing information i.e. it must present some new information. This
is because there is no need to assume a thing, which can be logically derived from existing information.
Everyone who reads this article will be more informed about Inference and Assumption. Rahul received this article in
his email. Hence, Rahul is more informed about Inference and Assumption.
Assumption: Rahul has read every mail that he has received so far.
Using the same terminology we used in the previous example: In this case, we are given “If X, then Y” and that Rahul
received this article in his email. These two statements are, then, taken to conclude that Y holds for Rahul.

But within the context of the passage, we know that to conclude Y, X must hold. Therefore, Rahul must have read the
article. However, we are only given that Rahul received this article in his email.

Therefore, the underlying assumption is that Rahul has read every mail that he has received so far.

Inference vs Assumption: point of confusion

We can see from the definitions above that there are some critical differences between an inference and an assumption:

Inference Assumption

Question stem Structure – If the Which of the following is an


statements above are true, which of assumption on which the argument
the following must be true? depends?

Inference is a statement which must Assumption is a statement which must


be true, if the given information is be true, for the given information to be
true true; in other words, for the conclusion
to hold true.

An inference can always be logically An assumption can never be logically


deducted from the given information deducted from the given information –
It contains some new information

It’s very rare for an inference question An assumption question necessarily


to have a conclusion in the passage or needs to have a conclusion in the
the question stem passage or the question stem

If we focus on the second point of difference above, we can see that an inference can never be same as an assumption. In
other words, they are mutually exclusive sets. There is no overlap.

-----------------------------------------------------------------

Why Am I Not Doing Better on GMAT Critical Reasoning?


by Patrick Tyrrell Apr 19, 2018
GIVEN THAT I’m awesome, HOW COME I’m not awesome at GMAT Critical Reasoning?

Do you know what cognitive dissonance is? It’s a concept that usually only comes up when someone is smugly implying
that you’re a hypocrite: “So you believe that abortion is wrong because we don’t have the right to take away someone
else’s right to life …but you believe in the death penalty?”

Wipe that look of “checkmate” off your face, bub:

There’s a crucial distinction between abortion and the death penalty, in terms of whose life is being taken!

The secret to being good at (90% of) GMAT Critical Reasoning is understanding how we can use cognitive dissonance
to our benefit. If, after reading the Critical Reasoning paragraph, we frame the right tension in our brains, then the
correct answer will feel like a relief as it swoops in to resolve the tension, like a major third rescuing us from a
suspended fourth (I see you, music nerds).

The 7 Main Question Types in GMAT Critical Reasoning


– Role of the Bold (is this bolded claim the Main Conclusion, something Supporting, something Opposing, or
something Neutral?)
– Inference (using only these statements, which answer choice can you [basically] prove?)
– Explain Discrepancy (which answer most helps to explain the surprise?)
– Assumption (which answer, if negated, would most hurt the argument/plan?)
– Weaken (which answer, if true, would most hurt the argument/plan?)
– Evaluate (which answer poses a question that could be answered in a way that would hurt the argument/plan?)
– Strengthen (which answer, if true, most helps the argument/plan?)

Role of the Bold has nothing to do with cognitive dissonance …or with the rest of GMAT Critical Reasoning …or with
business school …why, Role of the Bold, why?

Inference is about trying to derive conclusions by combining two or more facts, and frequently the available inference is
framed by some paradox tension:

The Bears won the game, but the Lions scored more points in the 2nd half. (infer: Bears scored more in the 1st half)
My rent just increased, however, it still represents the same percentage of my income. (infer: my income also just
increased)

In trying to resolve this mathematical tension, we find the available inference being tested.

“Given that the Lions scored more in the 2nd half, how did the Bears win the game?” Oh, they must have outplayed the
Lions in the 1st half.
“Given that my rent went up, how could it still be the same % of my income?” Oh, I guess I might have recently gotten a
raise.

Explain Discrepancy built its reputation on cognitive dissonance, Holmes! The whole premise behind these questions is
that they will present us with cognitive dissonance, and our correct answer will help us to resolve that confusion.

Ayuki has been studying for the GMAT for several months now. She has done almost every problem in the Official Guide
and has taken six practice tests. Nevertheless, her score has failed to significantly increase.

Which of the following, if true, most helps to explain this apparent discrepancy?
We always prephrase these paragraphs in this format:

GIVEN [the background fact(s)], HOW CAN WE MAKE SENSE OF [the surprising fact]?

In this case, GIVEN that Ayuki has done tons of problems and tests over the past few months, HOW CAN WE MAKE
SENSE OF her lack of improvement?

Correct answers might include cheap plugs for previous blog posts, such as:

(A) Ayuki does not have a sufficiently robust method for reviewing her problems.

(B) Ayuki approaches every problem with an overly narrow desire to solve algebraically and doesn’t consider alternative
solution paths.

(C) Ayuki is convinced that the test is measuring her Math/English skills rather than a constellation of scrappy, real-
world skills.

Beware that there are many Explain Discrepancy questions masquerading as Strengthen questions. Our books and
materials don’t even always catch the difference (and the difference isn’t of world-shaking importance). But pretend I
had presented you with the same paragraph, slightly tweaked and with a different question stem.

Ayuki has been studying for the GMAT for several months now. She has done almost every problem in the Official Guide
and has taken six practice tests. Nevertheless, Patrick doesn’t expect that her score will have improved significantly
from where she started.

Which of the following, if true, most helps to support Patrick’s expectation?

Because of the “which answer most supports” wording, we’d be tempted to see this as Strengthen, and in the sense that
we are trying to strengthen Patrick’s claim, it is a Strengthen question. But since Patrick has not yet presented any
rationale for his expectation, it’s not like we’ve read an argument. We’re not thinking about what assumptions Patrick
might be making. We have no reasons to think Ayuki would be at the same score, and we have multiple reasons to think
Ayuki would be at a higher score. So it’s more appropriate to interpret this as Explain Discrepancy, since we’re trying
to explain a surprising claim.

The easy way to tell we’re reading a Paradox, not an Argument, is that we’re trying to strengthen a claim that comes
after the word nevertheless.
If the question wants me to support a claim that’s prefaced by but, yet, however, nevertheless, then I know I’m really
trying to Explain a surprise.

We’ve established that Inference questions often use cognitive dissonance as a way of leading us towards an inference
and that Explain Discrepancy questions always use cognitive dissonance—it’s the name of their game.

When we read arguments and plans in Assumption, Strengthen, and Weaken questions, there usually isn’t any inherent
tension. In an argument, the author is leading you into her Conclusion. When we read a Plan, it’s designed to lead to a
certain Goal.

It’s up to us to create the tension. It’s gonna take all the stubbornness we’ve got. Are we ready for it? Of course we
are. After all, how many times have we acquiesced to an opponent during a political debate on Facebook or Twitter?

When we read Arguments, we frame the tension by thinking:


GIVEN [this evidence], HOW CAN I ARGUE [the opposite of the conclusion]?
When we read Plans, we frame the tension by thinking:
GIVEN [this plan], HOW CAN I ARGUE [we won’t achieve the goal]?

Every correct answer I’ve seen to questions based on Plans is addressing one of these four concerns:

4. Will the Plan backfire in some way that will thwart the original Goal?
5. Can we assume the skill (ability/feasibility) and will (motivation/incentive) of those involved in the Plan?
6. Does this Plan address a significant source of the problem it’s meant to solve?

Try some of these GMAT Critical Reasoning questions about Plans.

5. Find the Plan and the Goal.


6. Try to frame the tension of “GIVEN [this plan], HOW CAN I ARGUE [we won’t achieve the goal]?” before
you see answers. Jot down or type up any ideas you have.
7. Examine the choices and select your answer.
8. Determine whether the correct answer dealt with Backfire / Skill & Will / Significant Source of Problem.

2018 OG: diag66, diag 77, diag 80, 546, 554, 559, 563, 566, 575, 576, 577, 578, 585, 587, 589, 590, 592, 597, 600, 604,
605, 610, 617, 620, 643, 646

When it comes to Arguments, there are three main patterns:


– Causal Interpretations
– Predictions
– Everything Else

The juiciest and most important category is Causal Interpretations. In these arguments, the author presents some form
of Curious Fact and then concludes some possible Causal Interpretation of what that fact tells us.

EXAMPLES of Causal Interpretation arguments:

Jake is crying. Thus, he must be cutting onions.


Apple’s stock dropped 10% last quarter. Clearly, people don’t like the new iPhone.
People who read Patrick’s blogs score higher than those who don’t. This shows that his blog posts boost GMAT scores.

The Curious Fact is normally an unusual phenomenon (Paul crying), a statistical change (Apple’s stock went down), or
an interesting correlation (Patrick’s blog readers tend to be higher scorers).

Once we see the author’s conclusion, her Causal Interpretation, we need to ask ourselves two questions, but the first one
is like 8 times as important as the second one:

1. Is there some OTHER WAY to interpret this Curious Fact?


2. How PLAUSIBLE is the Author’s Interpretation?

When we read an argument and the conclusion is a Causal Interpretation, we’re still going to frame the tension in our
normal way: GIVEN [this evidence], HOW CAN I ARGUE [the opposite of the conclusion]?

I’m just letting you know that there are two very different ways to argue against the conclusion: you can supply
a different way to interpret the Curious Fact, or you can specifically undermine the author’s way of interpreting it.
Jake is crying. Thus, he must be cutting onions.
Different Way Objections: Jake’s dog died. Jake got fired. Jake is watching “This is Us.”
Author’s Way Objections: Jake does not have a knife. Jake is in the bathroom, not the kitchen.

Apple’s stock dropped 10% last quarter. Clearly, people don’t like the new iPhone.
Different Way Objections: The CEO of Apple was involved in a scandal. There’s an industry-wide recession.
Author’s Way Objections: Tech reviews of new iPhone were very positive. Consumer satisfaction surveys give new
iPhone high marks.

People who read Patrick’s blogs score higher than those who don’t. This shows that his blog posts boost GMAT scores.
Different Way Objections: Patrick only reveals URL of blog articles to high scorers. Patrick offers free tutoring to
anyone willing to read his blogs.
Author’s Way Objections: After reading one of Patrick’s articles, students usually perform worse on their next test.
Patrick’s blog is about fudge, not GMAT.

Try some of these GMAT Critical Reasoning questions about Causal Interpretations.

1. Find the Curious Fact (where the author thought “Why is this? What does this mean?”) and the Author’s
Interpretation (the author is saying “Here’s what it tells us.”).
2. Try to frame the tension of “GIVEN [this evidence], HOW CAN I ARGUE [the opposite of the conclusion]?”
before you see answers. Specifically think, “Is there a DIFFERENT WAY to explain the Curious Fact? Can I
undermine the plausibility of the AUTHOR’S WAY?”
3. Examine the choices and select your answer.
4. Determine whether the correct answer dealt with supplying/denying a DIFFERENT WAY to explain the curious
fact or whether it dealt with bolstering/undermining the plausibility of the AUTHOR’S WAY of explaining
things.

OG 2018: diag 68, diag 69, diag 70, diag 71, diag 78, diag 82, 549, 555, 560, 567, 569, 571, 574, 579, 583, 583, 584,
588, 595, 598, 602, 606, 607, 625, 635, 637, 638, 642, 644, 648, 649, 658, 661, 664, 667

The other pattern in Arguments, though not nearly as prevalent as Causal Interpretations, is arguments that end with the
author concluding some Prediction. The author is almost always examining some recent trends and forecasting
something on the basis of them. We need to push back against the author’s thinking by asking whether there’s any
important factor she’s failed to consider or whether she is failing to foresee a changing factor that could thwart the
prediction.

Try some of these GMAT Critical Reasoning questions about Predictions:

1. Find the Conclusion and the Evidence (the supporting ideas).


2. Try to frame the tension of “GIVEN [this evidence], HOW CAN I ARGUE [the opposite of the conclusion]?”
before you see answers. Specifically think, “Is the author OVERLOOKING AN IMPORTANT FACTOR or
failing to foresee a CHANGING FACTOR?”
3. Examine the choices and select your answer.
4. Determine whether the correct answer dealt with an important factor we originally left out of our calculation, a
changing factor that could keep the prediction from coming true, or both.

OG 2018 diag 76, 547, 593, 611, 614, 615, 622, 628, 632, 636, 654, 657, 663, 666

The catch-all category of Everything Else arguments is surprisingly smaller than you’d think. Most of these questions
are testing us on solving for the “Missing Puzzle Piece” in the author’s logic. We can practice our debating mindset by
framing that same tension of “GIVEN [the evidence], HOW CAN I ARGUE AGAINST [the conclusion]?” We might
also profit, pun intended, by thinking about what missing idea we need to feel better about our conclusion or to connect
the dots from our Premises to our Conclusion. Give these a try.
OG 2018 545, 580, 594, 608, 619, 624, 627, 629, 633, 645, 650, 655

For all of these question sets, don’t burn yourself out by trying the entire list in one sitting. You get more value out of
doing 3-5 of them and taking the time to review them (and then taking a break from them to let your brain actually
process/encode what you did).

To return to where we started, GIVEN THAT you’re awesome, HOW IS IT THAT you’re not awesome at GMAT
Critical Reasoning (yet)? One possible way to explain this discrepancy is that you haven’t yet fully learned how to

harness the power of cognitive dissonance.

Patrick Tyrrell is a Manhattan Prep instructor based in Los Angeles, California. He has a B.A. in philosophy, a 780
on the GMAT,

----------------------------------------------------------------------

Advanced Critical Reasoning Lesson: RTFQ


by Neil Thornton Jan 25, 2014
S o your Critical Reasoning (CR) score has moved a little, but not enough. Or each question is still taking you
3 minutes to answer. You’ve studied for months, read the Strategy Guides, taken every practice test, and
completed every Critical Reasoning question in the big Official Guide and the Verbal Review supplement so
many times you have them all memorized. What more can you do? Do more questions? You can probably
imagine, more questions will usually mean more of the same issues, and simply reinforce bad habits…

Chances are, despite all your hard work, you’re still using your intuition and “gut feeling” to answer CR
questions. Unfortunately, your gut feeling works some of the time, but not 100% of the time. Remember, the
test is designed so that the average person picking what “looks right” will get only 50% of the questions
correct.

So what to do? For now, stop doing more questions until you 1) learn the formal rules of logic behind how CR
works, and 2) deeply analyze all the questions you’ve done for repeating patterns: question types, patterns of
reasoning, logical flaws, right and wrong answer types, etc.

So that’s what the next few weeks will be about. Each week, I’ll post an article that goes absurdly in-depth
about one aspect of the logic behind CR, along with exercises to apply those lessons. These are the same
exercises I do with my tutoring students, who have found them very effective. I’m also interested in your
feedback: what worked for you? What didn’t? Questions and concepts you’re still struggling with? I’m open to
discussion and debate.

So let’s get started. I’ll start with the essentials and then really nerd out on formal logic, so keep reading to the
end.

LESSON ONE: RTFQ


In our classes, we teach a four-step process to answering CR questions:

1) Identify the question (Know what the question is asking and what kind of question it is)
2) Deconstruct the argument (Analyze each piece of the passage for what role it plays)
3) Pause and state the goal (Predict what the correct answer should do)
4) Work from wrong to right (Use process of elimination to get to the right–or “least wrong”–answer.)

Today’s focus: Step one, which I call RTFQ, as in “Read the F___ Question” (F as in Full! Read the Full
question. What were you thinking?)

The basics: The GMAT only asks a limited number of questions, with very rare variation. Each type of
question implies HOW you should deconstruct the argument and WHAT the right answer will do. If you don’t
identify the question properly, you won’t look for the right things, or you’ll waste time reading for things that
aren’t there. So…Right now, can you name them all? No really.

Exercise 1: Before you scroll down, get out your notebook and write down as many types of questions as you
can think of. Ready? Go.
.
.
.
.
.

How many did you come up with? 5? 6? Depending on how you break them down (what books you’ve read
and who taught you), there are anywhere from 10-13 common types of questions. Here’s a list of the 11 most
common that I use, grouped by category.

Structure based:
Identify the bolded part (role in the reasoning)
Identify the overall reasoning
Identify the conclusion
Mimic the reasoning (also known as parallel the reasoning)

Reasoning/assumption based:
Assumption
Strengthen
Weaken (and Flaw questions)
Evaluate
Fill in the Blank

Evidence or fact-based questions:


Inference (also known as “Draw a Conclusion” questions)
Resolve or Explain (a paradox or discrepancy)

I’ll explain more about the categories in future articles, but for now… Can you identify them when they show
up? One of the most common mistakes you can make on the GMAT is simply misidentifying the question (e.g.
mistaking strengthen for inference or strengthen for explain).

Exercise 2: Pick a dozen questions and name them! Take out your Official Guide for GMAT Review and get to
work. Let’s pick numbers 50 through 61. If you need help, skim the passage itself to (All questions excerpted
from The Official Guide for GMAT Review 13th Edition, by GMAC®)
50. Which of the following, if true, most strengthens the argument above?
51. The argument is most vulnerable to the objection that it fails to
52. Which of the following, if true, most strongly supports Summit’s explanation of its success in retaining
employees?
53. Which of the following strategies would be most likely to minimize company X’s losses on the policies?
54. If the statements above are true, which of the following must be true?
55. Which of the following most logically completes the argument given below?
56. The conclusion above would be more reasonably drawn if which of the following were inserted into the
argument as an additional premise?
57. Which of the following, if true, most helps to explain the surprising finding?
58. Which of the following, if true, most seriously weakens the conclusion above?
59. Which of the following most logically completes the passage?
60. If the facts stated in the passage above are true, a proper test of a country’s ability to be competitive is its
ability to
61. Which of the following, if true, does most to explain the contrast described above?

And for good measure, identify number 66.

66. Which of the following conclusions about Country Z’s adversely affected export-dependent industries is
best supported by the passage?

Write down what you think each one is.

Ready for your answers? Here’s how I identified them, with commentary.

50. Strengthen: Pretty straight up. The correct answer will strengthen the argument above.
51. Weaken: Or more specifically, identify the flaw in the reasoning. The words “it fails to” mean that the right
answer, when considered, will damage the argument.
52. Strengthen: Don’t let the word “explain” fool you. The explanation is already in the argument; in fact the
explanation may be the conclusion of the argument. Your job is to find an additional piece of evidence to
strengthen that explanation.
53. Resolve/Explain: This one was tough. The question implies that there’s a problem (losses) to be solved
(“minimize[d]”), which is what many resolve/explain questions do. Also, the argument itself describes a
pretty clear contradiction: how does X keep its prices low, but also make enough income to pay for claims?
The answer will resolve this. Feel free to argue with me in the comment section, though.
54. Inference (also known as “draw a conclusion”): Notice how “the statements above are true.” That mean
you WON’T be looking for premises and conclusions, just putting facts together to find out what else must be
true. More about this later in the section about “Deductive Reasoning.”
55. Fill in the Blank: note that the blank part starts with the word “because____” so you’ll be providing a
premise that helps the conclusion. So, in a way, you can look at this as a strengthen question, too.
56. Assumption: Yes, assumption, though if you named this as a strengthen question, you’ll probably get it
right. Technically, though, when the GMAT asks for an additional or unstated premise that makes the
argument “more reasonably drawn” or that is “required,” it’s asking you for the assumption. But it’s
interesting to note that assumption and strengthen questions both do the same thing: support the reasoning
of an argument.
57. Resolve/explain: NOT strengthen. Imagine walking into your house to find your favorite chair is broken.
Explaining WHY it’s broken is far different from Strengthening or fixing the chair with additional support.
58. Weaken: fair enough, easy to spot.
59. Fill in the blank: and with the word “since____” leading off the blank, it’s another strengthen.
60. Inference: Again, “if the statements above are true…” your reading for facts, not arguments.
61. Resolve/explain: again

aaaaand #66?
66. Inference: Yes. Inference. NOT STRENGTHEN! For more about how to differentiate between Inference
and Strengthen questions, see our Critical Reasoning Strategy Guide, chapter 6.

So, how’d you do? If you were less than 100%, spend some time with the strategy guide, focusing on how to
identify question types. Write down several examples of each question type and quiz yourself some more. You
can use the Official Guide Problem Sets in the back of the CR Strategy Guide to see whether you were right or
not. Keep working until you’re 100%.

NERDING OUT ON LOGIC

Critical reason is a test of LOGIC. So, with a big stack of logic books next to me, I’m going to discuss some of
the formal rules behind the what GMAT writes questions. Ready?

The GMAT uses the word conclusion in two different ways. Most of the time, the GMAT is referring to an
“inductive” conclusion, but occasionally, it’s asking about a “deductive” conclusion. Don’t confuse them!
So to explain: There are two kind of reasoning in the word: deductive reasoning and inductive reasoning.

Deductive reasoning is more concrete, more mathematical, more “true.”

Wikipedia’s definition of deductive reasoning: “Deductive reasoning links premises with conclusions. If all
premises are true, the terms are clear, and the rules of deductive logic are followed, then the conclusion
reached is necessarily true.”

In other words, if the premises are true, then the conclusion must be true. Does this sound like a common
question type? (Hint: it starts with an “I____”)

Here are some examples of deductively valid arguments.

Premise: Sally is taller than Frank.


Premise: Frank is taller than William
Conclusion: Sally must be taller than William.
(Other deductively valid conclusions: Frank is shorter than Sally. William is not the same height as sally.)

Premise: All cats are persnickety


Premise: Mr. Whiskers is a cat.
Conclusion: Mr. Whiskers is persnickety.
(Other deductively valid conclusions: Some persnickety things are cats. At least one cat is named Mr.
Whiskers.)

Inductive reasoning is a little softer, and much more common on the GMAT and in the real world. Science,
economics, medicine, and our justice system are largely based on induction.

Wikipedia’s definition again: “Inductive reasoning is reasoning in which the premises seek to supply strong
evidence for (not absolute proof of) the truth of the conclusion. While the conclusion of a deductive argument
is supposed to be certain, the truth of an inductive argument is supposed to be probable, based upon the
evidence given.”

In other words, if the premised are true, then the conclusion has a probability of being true, but also a
probability of being false.
I’m usually sleepy after 11:00pm.
It’s past midnight.
I must be sleepy.

3 out of 4 dentists recommend chewing OctiDent after meals.


You should chew Octident after every meal.

After I cut bacon out of my diet, I lost 5 pounds.


If you want to lose weight, you should cut bacon out of your diet.

Inductively valid arguments have a very high probability of being true, with little chance of contradictory
evidence (good scientific theories). Inductively invalid arguments have a high probability of being false
(horoscopes). The dividing line between valid and invalid arguments can be shady and can depend on context.
90% certainty would be a great bet at a casino, but a lousy bet on airplane guidance systems.

We’ll get more into how to evaluate inductive reasoning vs. deductive reasoning in later articles, but for now,
lets just learn to spot it.

Exercise: peruse the Official Guide questions 50-61 again. Decide whether the question and argument will be
based on induction or deduction (Hint: if the argument can be helped or hurt, it’s probably induction. In the
conclusion must be true, it’s deduction.)

50. Which of the following, if true, most strengthens the argument above?
51. The argument is most vulnerable to the objection that it fails to
52. Which of the following, if true, most strongly supports Summit’s explanation of its success in retaining
employees?
53. Which of the following strategies would be most likely to minimize company X’s losses on the policies?
54. If the statements above are true, which of the following must be true?
55. Which of the following most logically completes the argument given below?
56. The conclusion above would be more reasonably drawn if which of the following were inserted into the
argument as an additional premise?
57. Which of the following, if true, most helps to explain the surprising finding?
58. Which of the following, if true, most seriously weakens the conclusion above?
59. Which of the following most logically completes the passage?
60. If the facts stated in the passage above are true, a proper test of a country’s ability to be competitive is its
ability to
61. Which of the following, if true, does most to explain the contrast described above?

Answer Key:

50. Induction
51. Induction
52. Induction
53. Induction
54. DEDUCTION
55. Induction
56. Induction
57. Induction (The explanation will be inductively valid.)
58. Induction
59. Induction
60. DEDUCTION
61. Induction

What do you think about question 66? Discuss and debate it in the comments below!
Take some time looking up deductive reasoning vs. inductive reasoning on the web. Wikipedia is a good place
to start. Then, start analyzing other questions for the kind of reasoning tested on each. You may find that a lot
of the questions you got wrong were one type or the other.

For an advanced drill, dig up all the Inference questions you can find. (I’ll give you a few: 66, 91, 103, and 104)
Some of them are asking you for deductively valid conclusions, while others are asking for inductively valid
conclusions. Can you determine which is which? Again, post your results in the comments section below.

Get to work, and for now just focus on those questions! See you in future articles.

GMATprep 1 weaken_Q1

Insect infestations in certain cotton-growing regions of the world have caused dramatic increases in the price of cotton
on the world market. By contrast, the price of soybeans has long remained stable. Knowing that cotton plants mature
quickly, many soybean growers in Ortovia plan to cease growing soybeans and begin raising cotton instead, thereby
taking advantage of the high price of cotton to increase their income significantly,
at least over the next several years.

Which of the following, if true, most seriously weakens the plan's chances for success?

A. The cost of raising soybeans has increased significantly over the past several years and is expected to continue to
climb.
B. Tests of a newly developed, inexpensive pesticide have shown it to be both environmentally safe and effective against
the insects that have infested cotton crops.
C. In the past several years, there has been no sharp increase in the demand for cotton and for goods made out of cotton.
D. Few consumers would be willing to pay significantly higher prices for cotton goods than they are now paying.
E. The species of insect that has infested cotton plants has never been known to attack soybean plants.

In a weaken question one should always attack the conclusion, i.e. by growing cotton the farmers would be able to take
advantage of the increasing prices and hence significantly increase their income.

If there would be few customers to pay, farmers will not be able to increase their income.
Hence D.

In weaken question, the correct answer will not only contradict the conclusion but also undermine the possibility of
conclusion to be true, without affecting the premises or the facts in the argument.

----------

Shib, one of the keys to answering weaken/strengthen questions is to efficiently yet accurately identify the argument's
conclusion and then to understand the premises (facts) on which this conclusion rests. In the text at hand, the soybean
farmers have concluded that a switch to cotton will permit them to take advantage of the increased price of cotton on the
world markets. In GMAT "shorthand," my diagram would appear as follows

P: Insects -> cotton $^ on world market


P: Soybean $ stable
C: Ortavia farmers will increase income significantly if switch to cotton

To weaken this conclusion, the answer choice must demonstrate that the switch to cotton will NOT necessarily increase
the income of Ortavia's farmers. Use a strengthen/weaken slash chart as you utilize your process of elimination.

-- A) This has no bearing on the conclusion

W B) If a new pesticide were developed, farmers globally could again begin to grow cotton, reversing its increasing
price on world markets.

w C) A smaller weaken. According to the text, supply of cotton has decreased. Thus, we do not necessarily need to show
an increase in demand.

-- D) As long as customers are paying the high current price, there is no need for the price to further increase

-- E) Not relevant

The correct answer is B :)

GMATprep | Roles_Q1

In countries where automobile insurance includes compensation for whiplash injuries sustained in automobile accidents,
reports of having suffered such injuries are twice as frequent as they are in countries where whiplash is not covered.
Some commentators have argued, correctly, that since there is presently no objective test for whiplash, spurious
reports of whiplash injuries cannot be readily identified. These commentators are, however, wrong to draw the further
conclusion that in the countries with the higher rates of reported whiplash injuries, half of the reported cases are
spurious: clearly, in countries where automobile insurance does not include compensation for whiplash, people often
have little incentive to report whiplash injuries that they actually have suffered.

In the argument given, the two boldfaced portions play which of the following roles?

A) The first is evidence that has been used to support a conclusion that the argument criticizes; the second is that
conclusion.
B) The first is evidence that has been used to support a conclusion that the argument criticizes; the second is the position
that the argument defends.
C) The first is a claim that has been used to support a conclusion that the argument accepts; the second is the position
that the argument defends.
D) The first is an intermediate conclusion that has been used to support a conclusion that the argument defends; the
second is the position that the argument opposes.
E) The first presents a claim that is disputed in the argument; the second is a conclusion that has been drawn on the basis
of that claim.

By RON

well, first off, you can eliminate choice (d) pretty quickly, on the grounds that "there is presently no objective test for
whiplash" is a fact.
since this is a fact, it can't possibly be a "conclusion" of any kind; conclusions must be claims, not facts. (a fact can't be a
conclusion because facts don't require supporting arguments; you can just come out and state them, because they're,
well, facts.)

from this point, you don't even have to process the actual material of the argument -- you can just read the transitions,
which will give you a remarkably exact sense of what's going on in there.

Some commentators have argued, correctly, that, since (bold #1)...


--> the speaker says "correctly", so it's clear that the speaker accepts bold #1 as correct. on the other hand, that's not
significant, because #1 is factual, and the gmat doesn't contain stated facts that turn out to be false (only claims are
weakened/disproved on gmat problems).
what's significant is that the fact is used by "some commentators", i.e., other people.**

These commentators are, however, wrong to draw the further conclusion that (bold #2)
--> bold #2 is the other guys' conclusion, and our speaker thinks it's wrong.

so, we want an answer choice that says, "bold #1 is a fact that the other guys use in making their case; bold #2 is the
other guys' main point, which is wrong according to our narrator."
that's (a).

(b) is wrong because our narrator doesn't defend the other guys' conclusion; (s)he does the opposite.

(c) is wrong for the same reason as (b), but also because it mis-identifies a fact (bold #1) as a "claim".

(d) is wrong for two reasons: because it mistakenly labels a factual statement as a "conclusion" (as stated above), and
also because it says that the fact is used to support an argument with which the narrator agrees. the opposite is true; that
fact is used by the narrator's opponent.

(e) is wrong because it says that our narrator disagrees with #1, rather than with #2.

--

**in fact, as soon as you see "some people think...", "it is commonly thought that...", etc. - basically, any form of The
other guys think X - you can go ahead and assume that the narrator (= main argument) is going to oppose that point of
view.
after all, that's the only sensible rationale for introducing "the other guys" into the argument in the first place. (if the
narrator agreed with the other guys, then there would be no point in citing the other guys at all -- i.e., rather than saying
The other guys think X, and they're right, the narrator will just say X.)

Question is slightly changed by Kaplan as below:

Q26:
In countries where automobile insurance includes compensation for whiplash
injuries sustained in automobile accidents, reports of having suffered such injuries
are twice as frequent as they are in countries where whiplash is not covered. Some
commentators have argued, correctly, that since there is presently no objective test for
whiplash, spurious reports of whiplash injuries cannot be readily identified. These
commentators are, however, wrong to draw the further conclusion that in the countries
with the higher rates of reported whiplash injuries, half of the reported cases are spurious:
clearly, in countries where automobile insurance does not include compensation for
whiplash, people often have little incentive to report whiplash injuries that they actually
have suffered.

In the argument given, the two boldfaced portions play which of the following roles?

A. The first is a finding whose accuracy is evaluated in the argument; the second is
an intermediate conclusion drawn to support the judgment reached by the
argument on the accuracy of that finding.
B. The first is a finding whose accuracy is evaluated in the argument; the second is
evidence that has been used to challenge the accuracy of that finding.
C. The first is a finding whose implications are at issue in the argument; the second
is an intermediate conclusion that has been used to support a conclusion that the
argument criticizes.
D. The first is a claim that the argument disputes; the second is a narrower claim that
the argument accepts.
E. The first is a claim that has been used to support a conclusion that the argument
accepts; the second is that conclusion

The same question in OG 13 Ed. CR#78

In countries where automobile insurance includes compensation for whiplash injuries sustained in automobile
accidents, reports of having suffered such injuries are twice as frequent as they are in countries where whiplash is
not covered. Presently, no objective test for whiplash exists, so it is true that spurious reports of whiplash injuries
cannot be readily identified. Nevertheless, these facts do not warrant the conclusion drawn by some commentators that
in the countries with the higher rates of reported whiplash injuries, half of the reported cases are spurious. Clearly, in
countries where automobile insurance does not include compensation for whiplash, people often have little
incentive to report whiplash injuries that they actually have suffered.

In the argument given, the two boldfaced portions play which of the following roles?

(A) The first is a claim that the argument disputes; the second is a conclusion that has been based on that claim.
(B) The first is a claim that has been used to support a conclusion that the argument accepts; the second is that
conclusion.
(C) The first is evidence that has been used to support a conclusion for which the argument provides further evidence;
the second is the main conclusion of the argument.
(D) The first is a fi nding whose implications are at issue in the argument; the second is a claim presented in order to
argue against deriving certain implications from that finding.
(E) The first is a fi nding whose accuracy is evaluated in the argument; the second is evidence presented to establish that
the finding is accurate.

Argument Evaluation
Situation Reported whiplash injuries are twice as common in countries where car insurance
companies pay compensation for such injuries as they are in countries where insurance
companies do not. Although there is no objective test for whiplash, this does not mean,
as some suggest, that half of the reports of such injuries are fake. It could simply be that
where insurance will not pay for such injuries, people are less inclined to report them.

Reasoning What roles do the two boldfaced portions play in the argument?The first portion tells us about
the correlation between reported cases of whiplash in countries and the willingness of
insurance companies in those countries to compensate for whiplash injuries. The argument
next states that whiplash is difficult to objectively verify. The argument then asserts that
althoughthis last fact, taken together with the first boldfaced portion, has led some to infer
that over half of the reported cases in countries with the highest whiplash rates are spurious,
such an inference is unwarranted. The second boldfaced portion then helps to explain why
such an inference is not necessarily warranted by offering an alternative explanation.

A The claim made in the first boldfaced portion is never disputed in the argument; at dispute is how
to account for the fact that this claim is true. The second is not the argument’s conclusion.
B In a manner of speaking, perhaps, the argument uses the first portion to support its conclusion;
but there is no indication that it has been used elsewhere to do so. In any case, the second
boldfaced portion is not the argument’s conclusion.
C The first has been used to support a conclusion that the argument rejects; the second boldfaced
portion is not the argument’s conclusion.
D Correct. This option correctly identifies the roles played in the argument by the boldfaced portions.
E The accuracy of the first boldfaced portion is never questioned in the argument; nor is the second
intended to somehow help show that the first is accurate. Rather, the argument assumes that the
first portion is accurate.
The correct answer is D.

GMATprep | Weaken_Q2

In Kravonia, the average salary for jobs requiring a college degree has always been higher than the average salary for
jobs that do not require a degree. Over the last few years, the number of Kravonians enrolled in college has been
growing steadily. Therefore, the number of Kravonians entering the job market who have at least the qualification of a
college degree will eventually be significantly higher than it has been over the last few years.

Which of the following, if true, most seriously weakens the argument?

A. Kravonians with more than one college degree earn little more, on average, than do Kravonians with only one college
degree.
B. The average number of years Kravonian college students remain enrolled before completing a degree has been
increasing over the past several years.
C. Despite the increase in the number of Kravonians attending college, the percentage of the population attending
college is lower in Kravonia than in most other countries.
D. In recent years, employers have been requiring college degrees for workers in jobs that were previously performed
successfully by Kravonians who did not have college degrees.
E. For many years, employers in Kravonia have had difficulty finding enough college graduates to fill the high-paying
jobs that were available.

the problem with this argument is that it takes the observation that "the number of Kravonians enrolled in college has
been growing steadily", and takes that observation to mean that the number of graduates is going to increase in
proportion.

choice (b) attacks this connection -- if students are taking longer to graduate from college, then the increased enrollment
doesn't necessarily mean that there will be any more graduates! it's just a symptom of the fact that students are staying in
school for longer.

for instance, if students take 4 years to graduate from college and college graduates 1000 students per year, then (not
counting dropouts) that college's enrollment will be approximately 4000 students.
if students begin to take five years to graduate from college, then that same college will begin to have an enrollment of
5000 students -- even if it graduates the same number (1000) of students per year.

Rephrasing this examples


choice b weakens the argument by undermining the idea that there are going to be more college graduates in the first
place.
it's not a "timeline" thing. instead, if choice B is true, then it's quite possible that there will continue to be exactly the
same number of graduates per year as in years past.

again, think about numbers if necessary. let's say a 4-year college has 4000 students; then that means 1000 students are
graduating per year.
now, let's say the college's enrollment increases to 5000 students.
your first thought may be "hey, 25% more students = 25% more graduates". while this would be a perfectly valid
thought if everyone were still graduating in 4 years, it doesn't work anymore if people are now taking 5 years to
graduate; in that case, you're still looking at 1000 graduates per year. (the increased enrollment would just be an artifact
of the fact that there are now 5 classes of students enrolled, not just 4 anymore; each graduating class will be the same
size as always.)

The argument is about what will eventually happen.

Your statements here are, at best, a reason why certain strictly interpreted statistics won't change for a short time.
In other words, for some extremely short timeframe (a few years at most), you're saying that this choice doesn't
strengthen the argument.

Why B not correct by RON


remember that you're looking for REQUIRED ASSUMPTIONS.

here's a very useful criterion to use in these problems:


try REVERSING putative assumptions and see the effect on the argument.
if you REVERSE A REQUIRED ASSUMPTION, the ARGUMENT SHOULD BECOME INVALID.

let's try this with choice (d):


reverse the assumption: let's say the average salary for non-degree jobs will increase over the next 4 years.
this would actually STRENGTHEN the argument (!), because it would provide another reason that the average salary
would increase.
this is the complete opposite of the effect you're looking for; reversing the assumption should destroy the argument.
bad.
very bad.
wrong answer
-----

Two big problems there.

1/
"Doesn't strengthen" is not at all the same thing as "weaken".

2/
"Will eventually..." encompasses enough time for people to retire and for new graduates to take their place. With the
changes described, this choice definitely strengthens the argument in the long term.
-----
If (D) says "will not decrease" instead of "will not increase", can (D) become an assumption the stimulus depends on?

correct.

you can prove this to yourself by trying out the reversal method (described above) on your newly generated assumption;
if you reverse this new assumption, the validity of the passage is definitely destroyed.

-------

GMATprep | Roles_Q2

One of the limiting factors in human physical performance is the amount of oxygen that is absorbed by the muscles from
the bloodstream. Accordingly, entrepreneurs have begun selling at gymnasiums and health club bottles of drinking
water, labeled "SuperOXY," that has extra oxygen dissolved in the water. Such water would be useless in improving
physical performance, however, since the amount of oxygen in the blood of people who are exercising is already
more than the muscles can absorb.

Which of the following, if true, would serve the same function in the argument as the statement in boldface?

A) world-class athletes turn in record performances without such water


B) frequent physical exercise increases the body's ability to take in and use oxygen
C) the only way to get oxygen into the bloodstream so that it can be absorbed by the muscles is through the lungs
D) lack of oxygen is not the only factor limiting human physical performance
E) the water lost in exercising can be replaced with ordinary tap water

C is the best substitute.


the main function of the boldface is to highlight the fact that extra oxygen consumed through water would be useless in
improving performance.

(a) is irrelevant. this has nothing whatsoever to do with whether the water will improve performance.
analogy: world-class athletes also turn in record performances without using steroids. this fact clearly doesn't prove that
steroids are useless in improving athletic performances.
(b) is irrelevant. not only does it have nothing to do with the ways in which water is absorbed, but the "increase" is not
quantified at all.
in fact, ironically, the (very indirect) effect of (b) is to contribute to the argument. if frequent exercise can increase
oxygen intake, then it's possible that frequent exercisers might reach a point at which the extra oxygen in the water
would start helping them.
you shouldn't think that much, though. this is a very, very, very distant relationship, and correct answers are always
directly related to the passage at hand.

the boldface needs to be something that shows that the oxygen dissolved in the drink won't help.

according to choice (c), there's no way that the oxygen in the drink could possibly go into the bloodstream (because
drinks go into the stomach and not into the lungs). therefore, choice (c) shows that the drink can't be effective.

GMATprep | Complete_argument_Q1
Which of the following most logically completes the passage?

A certain tropical island received food donations in the form of powdered milk for distribution to its poorest residents,
who were thought to be malnourished. Subsequently, the rate of liver cancers among those islanders increased sharply.
The donated milk was probably to blame: recent laboratory research on rats has shown that rats briefly exposed to the
substances aflatoxin tend to develop liver cancer when fed casein, a milk protein. This result is relevant because
_______.

A. in the tropics, peanuts, a staple of these island residents, support a mold growth that produces aflatoxin
B. the liver is more sensitive to carcinogens, of which aflatoxin may be one, than most other bodily organs
C. casein is not the only protein contained in milk
D. powdered milk is the most appropriate form in which to send milk to a tropical destination
E. the people who were given the donated milk had been screened for their ability to digest milk

I pick A.

The way I read it, the stem says that the rats developed liver cancer when fed casein, which is a milk protein. However,
the stem argues that this happened because the rats were already exposed to aflatoxin.

Thus the argument is that for rats who are exposed to aflatoxin and then fed casein, liver cancer develops.

Similary, if the humans in the tropics are already exposed to aflatoxin, and then consume the powdered milk which
contains casein, liver cancer develops as well.

The argument asks how the experiment related to what cause the cancer in the island. In an ordinary CR question, I
would try to find the relationship between the experiment and the milk. However, this isn't the case.
that after the donation of the milk, the islanders sow a sharp increase in liver cancer.
--------------------------
We are asked to logically conclude the passage

The passage's conclusion is that the introduction of milk caused the increase in cancer.

We need to find out the reason why they think this.

The combination of the info in the question and the info given in A does that:

Question: aflatoxin + milk protein causes cancer. Milk (and hence milk protein) was given to the islanders.

All we are missing is the presence of aflatoxin...

Option A: Islanders have aflatoxin through eating peanuts


------------
Premise: Cassein+aflotoxin= Liver cancer.
Conclusion: we need aflotoxin and only option A does that

MCAT4 | Resolve_paradox_Q1

City Controller: 63% of our residents voted to approve the developer's request to build a national chain hotel on the site
of the old consignment store. The hotel will increase our revenue base and, therefore, provide more money for schools
and community services.

Mayor: But our recent survey showed that the most important reason people want to live here is our small-town feel
resulting from the local ownership of the vast majority of businesses.

What is the best explanation for the apparent contradiction in opinions cited by the controller and the mayor?

Most people believe having a small-town feel is more important than having quality schools.
A locally-owned business might be able to generate as much revenue as a well-known hotel chain.
The recent survey did not ask about preferences for a chain hotel versus a locally-owned bed and breakfast.
An increase in the town's revenue base may not result in additional money for the schools.
------
The recent survey cited by the mayor polled people who are considering moving to the town.
------
-----------------------------------------------------

OA. The majority of residents voted to allow a national chain to build a hotel in town. This appears to contradict the
mayor's claim that a recent survey showed most people want to live in the town because of the locally-owned
businesses. In order to answer the question, we need an additional piece of information which explains away,
or resolves, this apparent contradiction.

(A) If people rank a small-town feel higher in importance than quality schools, then this fact would still contradict the
combination of the vote and the survey response.

(B) If a locally-owned business could generate the same amount of revenue as the national hotel, then this fact would
still contradict the combination of the vote and the survey response.

(C) This does not address why the residents voted to approve the hotel while survey respondents said they preferred
locally-owned businesses.

(D) This appears to contradict the reason given for residents voting to approve the hotel but does not do so in a way that
explains the discrepancy with the survey respondents.

(E) CORRECT. If the survey respondents were not actually residents of the town, this explains why the residents of the
town voted in a way that does not reflect the results of the survey.

---------------------------------------------------------------

This question is a resolve the paradox type question.We have to find out the choice that proves that both the
contradictory opinions can be true at the same time.

the MAYOR never said anything about polling residents, the CONTROLLER did. Differentiating the Controller’s ballot
results from the Mayor’s survey is exactly how we resolve this discrepancy..

Why (e)

There is a paradox. 2 pools talk differently! Why? Do the surveyers of the 2 pools have same tastes? Well... NO.

In more detail, if one of the survey is polled to people who think to move to town, then the results reflect this
consideration as it is in this case! No one, normally, expect to get a result that supports construction of the hotel chain!
To sum up, the recent pool's voters are the ones likely to move town!

e)The recent survey cited by the mayor polled people who are considering moving to the town.

Option c: this is out of scope. compares CH vs. locally-owned bed and breakfast.

] (C) INCORRECT--It actually gives the reason for ACTUAL CONTRADICTION.

C actually points out a contradiction an hence does not explain that there is no contradiction.
GMATprep 1 | weaken_Q3

Although fullerenes--spherical molecules made entirely of carbon--were first found in the laboratory, they have since
been found in nature, formed in fissures of the rare mineral shungite. Since laboratory synthesis of fullerenes requires
distinctive conditions of temperature and pressure, this discovery should give geologists a test case for evaluating
hypotheses about the state of the Earth's crust at the time these naturally occurring fullerenes were formed.

Which of the following, if true, most seriously undermines the argument?

(A) Confirming that the shungite genuinely contained fullerenes took careful experimentation.
(B) Some fullerenes have also been found on the remains of a small meteorite that collided with a spacecraft.
(C) The mineral shungite itself contains large amounts of carbon, from which the fullerenes apparently formed.
(D) The naturally occurring fullerenes are arranged in a previously unknown crystalline structure.
(E) Shungite itself is formed only under distinctive conditions.

Answer- D
Ron: Take another look at that answer choice: It's the crystalline structure of the found fullerenes that's 'previously
unknown'. That means that the fullerenes that have been discovered are fundamentally different from ALL other
fullerenes that have ever been described - including all the ones that have ever been made in labs.
* This means that these particular fullerenes - the ones that have been found in the fissures - are chemically different
from all the KNOWN classes of fullerenes.
* [KEY INFERENCE] That means that we don't know the 'distinctive conditions of temp and pressure' required to
form the NEW kinds of fullerenes.
* Therefore, because of this ignorance, they don't constitute a test case for anything geological.

Hope that helps

RON: Option E irrelevant.


“at the very least, you should be able to articulate WHY things are 'irrelevant'.

this could happen in one of two ways:


1/ a choice mentions X thing, or contains Y distinction, which does not affect the topic at hand (and is perhaps
specifically excluded by the language of the prompt);
or
2/ any correct answer must do Z thing, and you're looking at a choice that doesn't.

here:
this problem is full of big words, but, at the end of the day, it boils down to this: "These things were formed in nature.
We have also made them in the lab. Thus the conditions in nature must have been LIKE those in the lab."

if you want to show that this is wrong, then you MUST come up with something that differs between the lab fullerenes
and the natural ones. any choice that does NOT draw such a distinction is 'irrelevant' (#2 above).”

Option (a)
RON: also note that choice A is even worse, in the sense that it essentially tells you nothing.

the passage has already stated that people found xxxxx stuff. this is already a FACT.
choice A tells us that the people who found xxxxx stuff had to be very, very meticulous about their scientific methods.
well, that's great, but we don't have any new information. so A does nothing whatsoever.

i guess you could regard this as the most general possible form of #2 above: if ANY choice is to strengthen or weaken
an argument, it MUST present new information. (if you were on a jury, this would be obvious: if you want to strengthen
or weaken the case for the prosecution or defense, then clearly you need to present some NEW evidence!)

GP1 |CR6 | weaken_Q4


The country of Ertland has never imported apples in any significant quantity because consumers there generally prefer
the unique texture of Ertland-grown apples. Nevertheless, apple growers from Kosolia, a neighboring country, plan to
sell their apples in Ertland by selling Kosolia-grown apples at half the price of local apples and promoting them as a
nourishing, low-cost alternative.

Which of the following, if true, casts most doubt on the viability of the plan by Kosolia's apple growers to sell their
apples in Ertland?

(A) Most of the varieties of apples grown in Ertland were originally derived from common Kosolian varieties.
(B) Consumers in Ertland tend to spend about the same proportion of their income on fresh fruits and vegetables as do
consumers in Kosolia.
(C) At times in the past, Ertland has exported significant quantities of apples to Kosolia.
(D) Some varieties of apples grown in Kosolia can be harvested throughout most of the year, whereas the varieties
grown in Ertland can be harvested only during two months of the year.
(E) Profits of Ertland-grown apples are high enough in Ertland that growers, wholesalers, and retailers there could easily
afford to reduce the price at which these apples are sold.

--------------------------------------------

Argument:
Ertland has never imported apples in any significant quantity because consumers there generally prefer the unique
texture of Ertland-grown apples.
Apple growers from Kosolia, a neighbouring country, plan to sell their apples in Ertland by selling Kosolia-grown
apples at half the price of local apples and promoting them as a nourishing, low-cost alternative.

Pre thinking:
Kasolia Apple growers plan depends on promoting their apples as low cost alterntive.Anything which counter this will
weaken their plan.

Analysis of answer choices:


A Most of the varieties of apples grown in Ertland were originally derived from common Kosolian varieties.
INCORRECT: This will actually help the plan as the texture can be the same.

B Consumers in Ertland tend to spend about the same proportion of their income on fresh fruits and vegetables as do
consumers in Kosolia.
INCORRECT: irrelevant

C At times in the past, Ertland has exported significant quantities of apples to Kosolia.
INCORRECT: irrelevant

D Some varieties of apples grown in Kosolia can be harvested throughout most of the year, whereas the varieties grown
in Ertland can be harvested only during two months of the year.
INCORRECT: It can help the plan as shortage of apples in Ertland can be taken care.

E Profit of Ertland-grown apples are high enough in Ertland that growers, wholesalers, and retailers there could easily
afford to reduce the price at which these apples are sold.
CORRECT: it will weaken the plan as kasolias plan is based on providing a LOW COST ALTERNATIVES.

----------------------------

Best

The key points of the given passage are:


1) Consumers of Ertland are fond of their home grown apples due to the unique texture of the apples.
2) Apple growers from Kosolia plan to sell their apples in Ertland at half the price of local apples and promoting
them as a nourishing, low-cost alternative.

We need to select the option which casts the most doubt on the viability of Kosolian apple growers' plans.

Let us examine the options.

Option A: Most of the varieties of apples grown in Ertland were originally derived from common Kosolian varieties.
This option in fact reinforces their plan because if the Ertland apples were originally derived from Kosolian
varieties then the texture of the both apples is most probably the same. This adds to the advantage of the
Kosolian apple growers' plan to sell their apples in Ertland because Ertland consumers like their own apples
because of the texture of the apple.
Hence Option A is wrong.

Option B: Consumers in Ertland tend to spend about the same proportion of their income on fresh fruits and vegetables
as do consumers in Kosolia.
This option too strengthens the Kosolian apple growers' plan because they are planning to sell their apples in
Ertland at half the price of local apples and promoting them as a nourishing, low-cost alternative.
Hence Option B is incorrect.

Option C: At times in the past, Ertland has exported significant quantities of apples to Kosolia.
This is irrelevant to the plan.
Hence Option C is incorrect.

Option D: Some varieties of apples grown in Kosolia can be harvested throughout most of the year, whereas the
varieties grown in Ertland can be harvested only during two months of the year.
This again strengthens the Kosolian apple growers' plan. Because of the abundant supply of their apples, they
can always sell their apples to the Ertland people who need apples.
Hence Option D is incorrect.
Option E: Profit of Ertland-grown apples are high enough in Ertland that growers, wholesalers, and retailers there could
easily afford to reduce the price at which these apples are sold.
This will be a problem for the Kosolian apple growers because this will prove detrimental to their plan of selling
apples as a low-cost alternative.
Hence Option E is correct.

So the correct answer is Option E.

-----------------------------------------------------------

There are two new words in conclusion -> Low cost , Same nourishment => we need to attack

The country of Ertland has never imported apples in any significant quantity because consumers there generally prefer
the unique texture of Ertland-grown apples. Nevertheless, apple growers from Kosolia, a neighboring country, plan to
sell their apples in Ertland by selling Kosolia-grown apples at half the price of local apples and promoting them as a
nourishing, low-cost alternative.

Which of the following, if true, casts most doubt on the viability of the plan by Kosolia's apple growers to sell their
apples in Ertland?

A Most of the varieties of apples grown in Ertland were originally derived from common Kosolian varieties.
wrong- it actually strengthens...the varieties are same

B Consumers in Ertland tend to spend about the same proportion of their income on fresh fruits and vegetables as do
consumers in Kosolia.
wrong- Does not attack the conclusion
C At times in the past, Ertland has exported significant quantities of apples to Kosolia.
wrong- not related to conclusion
D Some varieties of apples grown in Kosolia can be harvested throughout most of the year, whereas the varieties grown
in Ertland can be harvested only during two months of the year.
wrong- it does not attack the low cost
E Profit of Ertland-grown apples are high enough in Ertland that growers, wholesalers, and retailers there could
easily afford to reduce the price at which these apples are sold.
Correct - perfectly attacks the low cost word in conclusion

IMO E

--------------------------------------------
GP1 Q. Bank | Verbal Review (2nd Ed.) weaken_Q5

Automobile Dealer's Advertisement:

The Highway Traffic Safety Institute reports that the PZ 1000 has the fewest injuries per accident of any car in its class.
This shows that the PZ 1000 is one of the safest cars available today.

Which of the following, if true, most seriously weakens the argument in the advertisement?

(A) The Highway Traffic Safety Institute report listed many cars in other classes that had more injuries per accident than
did the PZ 1000.
(B) In recent years many more PZ 1000s have been sold than have any other kind of car in its class.
(C) Cars in the class to which the PZ 1000 belongs are more likely to be involved in accidents than are other types of
cars.
(D) The difference between the number of injuries per accident for the PZ 1000 and that for other cars in its class is
quite pronounced.
(E) The Highway Traffic Safety Institute issues reports only once a year.

Official Guide for GMAT Verbal Review, 2nd Edition

Practice Question
Question No.: 24
Page: 124

--------------

Automobile Dealer's Advertisement:


Prem:
The Highway Traffic Safety Institute reports that the PZ 1000 has the fewest injuries per accident of any car in its
class.
Conc:
This shows that the PZ 1000 is one of the safest cars available today.

IMO C:
Reason, if the mentioned class itself is unsafe then PZ might be unsafe compared to the car in other classes.

By JarvisR
Verbal Forum Moderator
-----------------------------------
Automobile Dealer's Advertisement:
Fact : The Highway Traffic Safety Institute reports that the PZ 1000 has the fewest injuries per accident of any car in its
class.
Conclusion: This shows that the PZ 1000 is one of the safest cars available today.

The key here is in its class. It can be clearly inferred from fact 1 that it is the safest car in its class.But if it has to be the
safest car of all the available cars then the class also has to be safest.
If we can find that the class is not the safest then it will weaken the argument.
VarunBhardwaj
Manager
------------------------------------------
Option c is the Best choice-This weakens the conclusion because if this class of cars is more prone to accidents than the
PZ 1000 can still have a lot of accidents and have the fewest injuries per accident but it will not necessarily be the safest
car on the road because this class of cars is the most accident prone...Example. You would not consider the best
"amateur" tennis player in the world to be the best tennis player in the world just because he is the best in his class.

geno5
Manager
---------------------------------
Let's get this in this way.

Ask yourself that under what conditions, the claim of the argument is no longer valid. The claim of the argument is PZ
1000 is the safest car today.
The choice C tells that The class to which PZ 1000 belongs to the group are more likely to be involved in accident.

Add this option to argument and then see what happens. The conclusion will be dubious that PZ 1000 is the safest car.

GP1 Q. Bank | weaken_Q6

Tiger sharks are common in the waters surrounding Tenare Island. Usually tiger sharks feed on smaller sharks, but
sometimes they have attacked tourists swimming and surfing at Tenare's beaches. This has hurt Tenare's tourism
industry, which is second only to its fishing industry in annual revenues. In order to help the economy, therefore, the
mayor of the island has proposed an ongoing program to kill any tiger sharks within a mile of the beaches.

Which of the following, if true, most strongly calls into question the likelihood that implementation of the mayor's
proposal will have the desired consequence?

(A) Even if not all the tiger sharks that come close to the beaches are killed, the existence of the program would reassure
tourists.
(B) Business owners who depend on tourism are willing to pay most of the cost of implementing the program.
(C) Tourists come to Tenare Island for its beaches, even though the island features a number of other tourist attractions.
(D) The small sharks on which tiger sharks prey feed on fish that are commercially important to the island's fisheries.
(E) Not all tourists who come to Tenare Island enjoy swimming or surfing.
RON: here's the basic deal:
* economy has 2 main pillars: TOURISM and FISHING
* tiger sharks are hurting TOURISM
* mayor wants to help TOURISM by killing tiger sharks

if you want to weaken this argument, you want to show that killing the tiger sharks will have a deleterious effect on the
other pillar of the economy - namely, FISHING. (note how much more obvious this is if you diagram the passage, as
i've roughly done above).

choice d is correct, because it shows that, while killing tiger sharks may be good for TOURISM, it will be bad for
FISHING (which, the passage asserts, is ultimately a more important industry).

wrong answers:
choice a has the opposite effect: it actually strengthens the mayor's policy, by assuring that there will be a positive effect
even if the program is not carried out to the fullest.

choice b is neutral (transferring $ from businesses neither infuses $ into the economy nor removes $ from the economy).

choice c strengthens the policy, because it underscores the idea that the beaches are critical to the economy (and
therefore that removal of the tiger sharks is of paramount importance)

choice e is irrelevant, because the non-beach tourists have no bearing on the issue either way.

--------------------------------------------

BTG

D is correct. The highest revenue generating industry in the island is Fisheries and the fishes on which the small sharks
prey on are commercially important to the fisheries. So, if the mayor's plan is implemented, the number of small sharks
will rise because the tiger sharks which prey on small sharks will be killed. This will create an imbalance in the island's
ecology and the commercially important fish will be impacted. Hence the mayor's plan will not improve the economy.

----------------------------

Karishma

Answer is (D) only.

This is a weaken question. There is a plan - to help the economy, kill tiger sharks within a mile of the beaches.

We have to weaken it i.e. we have to prove that killing the tiger sharks will not help the economy.

The argument tells us the following:


Tiger sharks feed on smaller sharks but sometimes attack tourists.
Tenare's fishing industry provides most revenues and tourism industry is second.

A. Even if not all the tiger sharks that come close to the beaches are killed, the existence of the program would reassure
tourists.
This doesn't weaken our plan, it actually strengthens it. Tourists will get reassured and will come back. Economy will be
helped.

B. Business owners who depend on tourism are willing to pay most of the cost of implementing the program.
Again, if anything, it strengthens our plan (very slightly though!). The cost of the program will not be paid by the govt
and hence the revenues generated will not be lost on implementing the program. So economy will be helped.

C. Tourists come to Tenare Island for its beaches, even though the island features a number of other tourist attractions.
Irrelevant

D.The small sharks on which tiger sharks prey feed on fish that are commercially important to the island\'s fisheries.
If tiger sharks are killed, small sharks will prosper which means the number of fish they feed on will decrease. Since this
fish is commercially important, if their availability reduces, the revenues from fishing may reduce. Hence, actually, the
economy may not be helped. This weakens our plan.

E. Not all tourists who come to Tenare Island enjoy swimming or surfing.
Irrelevant.

Answer (D)
_________________

Karishma
Veritas Prep | GMAT Instructor
My Blog

GP1 |CR 11|


Mourdet Winery: Danville Winery's new wine was introduced to compete with our most popular wine, which is sold in
a distinctive tall, black bottle. Danville uses a similar bottle. Thus, it is likely that many customers intending to buy our
wine will mistakenly buy theirs instead.

Danville Winery: Not so. The two bottles can be readily distinguished: the label on ours, but not on theirs, is gold
colored.

Which of the following, if true, most undermines Danville Winery's response?

(A) Gold is the background color on the label of many of the wines produced by Danville Winery.
(B) When the bottles are viewed side by side, Danville Winery's bottle is perceptibly taller than Mourdet Winery's.
(C) Danville Winery, unlike Mourdet Winery, displays its wine's label prominently in advertisements.
(D) It is common for occasional purchasers to buy a bottle of wine on the basis of a general impression of the most
obvious feature of the bottle.
(E) Many popular wines are sold in bottles of a standard design.

Tim, Mgmat staff: D is telling us that consumers may look only at the general shape and design of the bottle rather than
paying careful attention to the label. This clearly undermines Danville's argument. On the other hand, E does nothing to
undermine the argument. Thus it is wrong.
-----------------------

Yes, there is a difference between "many customers" and "occasional purchasers", this means that (D) is not
undermining Danville's response greatly; however it does undermine the Winery's response. Other answer choices do
not undermine the response.

If it is common for occasional purchasers to buy a wine base on bottles most prominent characteristic, then some people
would ignore the gold label on the bottle, thus, undermining Danville's response.

Hope this helps,

Vercules, GC

GP1| CR 12|

Finding of a survey of Systems magazine subscribers: Thirty percent of all merchandise orders placed by subscribers in
response to advertisements in the magazine last year were placed by subscribers under age thirty-five.

Finding of a survey of advertisers in Systems magazine: Most of the merchandise orders placed in response to
advertisements in Systems last year were placed by people under age thirty-five.

For both of the findings to be accurate, which of the following must be true?

(A) More subscribers to Systems who have never ordered merchandise in response to advertisements in the magazine
are age thirty-five or over than are under age thirty-five.
(B) Among subscribers to Systems, the proportion who are under age thirty-five was considerably lower last year than it
is now.
(C) Most merchandise orders placed in response to advertisements in Systems last year were placed by Systems
subscribers over age thirty-five.
(D) Last year, the average dollar amount of merchandise orders placed was less for subscribers under age thirty-five than
for those age thirty-five or over.
(E) Last year many people who placed orders for merchandise in response to advertisements in Systems were not
subscribers to the magazine.

Q1 MOST / MANY / MORE imply more than 50 % ???


Q 2 What about usage of SOME ?? Will "some" mean less than 50 % ??

"most" is more than 50%.

"more" means ... more. it means more than whatever statistic is cited.
e.g. "more than 25%"; "more than 50%". you can't just say "more" by itself.

"many" doesn't indicate any particular percentage. USUALLY it's used to refer to amounts that are "sizable" (i.e., not a
tiny tiny fraction), but short of a majority.
e.g., if my team has 50 athletes, most native speakers would interpret "many of my athletes qualified for the finals" as
meaning somewhere around 15-20 out of 50.

"some" also doesn't indicate any particular percentage, but, qualitatively, it's between "a few" and "many".
e.g., if my team has 50 athletes, most native speakers would interpret "some of my athletes qualified for the finals" as
meaning somewhere around 5-15 out of 50.

------------------------

RON:

this problem features QUANTITATIVE DATA. that may make you grimace, depending on your mindset about
quantitative data, but it shouldn't: all you need to do is look for the discrepancy in the numbers, and then find an answer
choice that accounts for the discrepancy. unlike problems that are all words, these problems feature discrepancies that
will essentially jump out at you if you examine the numbers.

when you examine the numbers, you must also pay close attention to DIFFERENCES, especially SLIGHT categorical
differences, between categories that are being compared. in this problem, the most important difference is
between subscribers who order and people who order. (there's also the distinction between random orders and orders in
response to ads, but that distinction isn't tested, because both surveys are on the same side of it.)

in this problem:
of the orders placed by subscribers in response to ads, only 30% were from the under-35 crowd.
of the orders placed by anyone in response to ads, over 50% were from the under-35 crowd.
there's the discrepancy.

therefore, you need an answer choice that explains where all the extra orders in the 'anyone' category came from. this
means in the 'anyone' category and not in the 'subscribers' category; no more, no less. that's exactly what is described in
choice e.

GP1 | CR 23|

From 1978 to 1988, beverage containers accounted for a steadily decreasing percentage of the total weight of household
garbage in the United States. The increasingly widespread practice of recycling aluminum and glass was responsible for
most of this decline. However, although aluminum recycling was more widely practiced in this period than glass
recycling, it was found that the weight of glass bottles in household garbage declined by a greater percentage than the
weight of aluminum cans.

Which of the following, if true of the United States in the period 1978 to 1988, most helps to account for the finding?

(A) Glass bottles are significantly heavier than aluminum cans of comparable size.
(B) Recycled aluminum cans were almost all beverage containers, but a significant fraction of the recycled glass bottles
had contained products other than beverages.
(C) Manufacturers replaced many glass bottles, but few aluminum cans, with plastic containers.
(D) The total weight of glass bottles purchased by households increased at a slightly faster rate than the total weight of
aluminum cans.
(E) In many areas, glass bottles had to be sorted by color of the glass before being recycled, whereas aluminum cans
required no sorting.

Option A

A. The actual weight of glass is irrelevant as it has remained the same through time.

GMATser1, GC : This shouldn't matter. We are talking about the % decline over a period of time, not the overall
weight.

B. Who cares what was in the glass, as the weight is not effective.

Option C

C. Bingo! There is less glass being circulated, therefore the aggregate weight of glass will be less.

GMATser1, GC :This is our answer. If manufacturers replaced the glass bottles with plastic, as a %, the weight
of the glass bottles compared to everything else would go down. Since this says aluminum cans were replaced
fewer than glass bottles, the conclusion makes logical sense.

Q. I think C is not the right answer, because in C it says that glass bottles are replaced by Plastic containers, it no where
mentions the weight of the plastic containers, it might be possible that plastic containers are heavier than small size
bottles.

Meja401, GC : Paradox: glass decreased (%) more than aluminum did (%) even when recycling aluminum was more
common.
You say the plastic containers can be heavier than glass bottles. That's fine. Plastic can be 1000 tons greater than glass.
Still, the glass was replaced by plastic, and the paradox only requires explanation how glass decreased (%) more than
aluminum (%)

-------------------------------------

GMATIFY: I guess only contention is between Option A) and Option C) - OA is Option C)

Let me explain why Option A) is wrong (BTW : I marked option A in the Mock )

There is decline in the weight of the garbage and majority of the decline is weight is contributed by the recycling
process.

Aluminium recycling > Glass recycling.

But decline in weight of aluminium bottles is less than decline in the weight of glass bottles. The question doesn't say
that therelative decline of aluminium bottles is less than relative decline of glass bottles
Lets say 100Kgs of garbage

Now earlier out of 100 kgs, 30 kgs was contributed by AL bottles and 10 kgs (lets say) is contributed by GL bottles.
Lets say AL is heavier and is 1 Kg and GL bottles is 0.5 kg
So contribution for AL was 30% and for GL bottles it was 10%

Now lets say 10 AL bottles are recycled and lets say 8 bottles
So the weight of AL bottles decreases by 10 Kgs
and for GL it is 4 kgs.

So decline by weight of AL = 10/30 = 33.33%


And decline by weight of GL = 4/10 = 40%

Even though the recycle of AL bottles is more than recycle of GL bottles, the decline in the weight of GL as a % >
decline in the weight of AL as a %.

C) is correct as denominator is changed by Manufacturers and hence it explains the paradox.

-----------------

Optio C & D by RON

this problem turns on the differences between PERCENTAGES and ABSOLUTE NUMBERS.

facts:
* the PERCENTAGE of recycled aluminum was higher than the PERCENTAGE of recycled glass. this is what it means
when we say that aluminum recycling was more widely practiced.
BUT
* the NUMERICAL CHANGE in aluminum in the trash was lower than the NUMERICAL CHANGE in glass in the
trash.

if all the stuff missing from the trash had been recycled, this would be impossible. therefore, we need another
explanation, besides recycling, for WHY THE TOTAL USE OF GLASS HAS GONE DOWN. that's the only way
that these findings are mathematically possible.

(c) is such a statement.

(d) is the EXACT OPPOSITE of this sort of statement. if (d) were true, we would expect to see aluminum, not glass,
declining at a faster rate in the trash.

-------------------

the point is that...


• ... more aluminum than glass (percentage-wise) is being removed from the trash -- for recycling purposes,
BUT
• ... nevertheless, the weight of glass went down by a greater percentage.

these are clearly in conflict with each other, so, we need some other reason why the weight of glass went down --
above and beyond recycling.

Q. P 1 | CR | weakening

Perkins: According to an article I read, the woolly mammoth‘s extinction in North America coincided with a migration
of humans onto the continent 12,000 years ago, and stone spearheads from this period indicate that these people were
hunters. But the author's contention that being hunted by humans contributed to the woolly mammoth‘s extinction is
surely wrong since, as paleontologists know, no spearheads have ever been found among the many mammoth bones that
have been unearthed.

Which of the following, if true, provides the strongest reason for discounting the evidence Perkins cites in arguing
against the contention that being hunted by humans contributed to the North American extinction of woolly mammoths?

A. At sites where mammoth bones dating from 12,000 years ago have been unearthed, bones of other mammals have
rarely been found.

B. The stone from which stone spearheads were made is unlikely to have disintegrated over the course of 12,000 years.

C. Conditions in North America 12,000 years ago were such that humans could not have survived there on a diet that did
not include substantial amounts of meat.

D. Cave paintings in North America that date from 12,000 years ago depict woolly mammoths as well as a variety of
other animals, including deer and buffalo.

E. Because of the great effort that would have been required to produce each stone spearhead, hunters would have been
unlikely to leave them behind.

Possibly the correct wording in E is : “hunters would have been unlikely to leave them behind.”

JarvisR

Verbal Forum Moderato

Perkins: According to an article I read, the woolly mammoth‘s extinction in North America coincided with a migration
of humans onto the continent 12,000 years ago, and stone spearheads from this period indicate that these people were
hunters. But the author's contention that being hunted by humans contributed to the woolly mammoth‘s extinction is
surely wrong since, as paleontologists know, no spearheads have ever been found among the many mammoth bones that
have been unearthed.
Conclusion:
being hunted by humans contributed to the woolly mammoth‘s extinction is surely wrong.

Perkin's conclusion relies upon the following fact:


no spearheads have ever been found among the many mammoth bones that have been unearthed.

If we can somehow show/explain ,why the spearheads were not found among mamoth bones then the conclusion will be
weaken.

Option B: The stone from which stone spearheads were made is unlikely to have disintegrated over the course of 12,000
years.
>>It supports Perkins argument instead of weakening it.If spearheads cant disintegrated then they shld have been present
among the bones, but this is not the case.Hence its possible that human didn't hunt woolly mammoth.

option C:
>>introduces a trap.just mentions about the meat.This meat could be of any animal.
Also it still doesn't answer why the spearheads were not found near the site.We need an answer that links the bone found
and missing spearheads.

Correct answer option E:


Because of the great effort that would have been required to produce each stone spearhead, hunters would have been
unlikely to leave them behind.
>>This could be a reason why the spearheads were not found near the mamoth bones.

------------------------------------------------------

Nevernevergiveup

VP

the woolly mammoth‘s extinction in North America coincided with a migration of humans onto the continent 12,000
years ago, and stone spearheads from this period indicate that these people were hunters.

author's contention: hunting by humans contributed to the woolly mammoth‘s extinction

Perkins conclusion: the author's contention is surely wrong i.e., humans are not responsible for mammoth's extinction.

Perkins reason or evidence: No spearheads have ever been found among the many mammoth bones that have been
unearthed.

Which of the following, if true, provides the strongest reason for discounting(Weaken) the evidence Perkins cites in
arguing against the contention that being hunted by humans contributed to the North American extinction of woolly
mammoths?

We need a choice which weakens the perkins or strenghtens the authors assertion. I interpreted it exactly opposite due to
unknown usage of word discount.

A. At sites where mammoth bones dating from 12,000 years ago have been unearthed, bones of other mammals have
rarely been found...............we are not bothered with other mammals.
B. The stone from which stone spearheads were made is unlikely to have disintegrated over the course of 12,000
years..............then they should have bee found. This strengthens perkins statement. Opposite.

C. Conditions in North America 12,000 years ago were such that humans could not have survived there on a diet that did
not include substantial amounts of meat.............this indicates that they hunted but there is no info to indicate that
humans hunted mammals and caused their extinction.

D. Cave paintings in North America that date from 12,000 years ago depict woolly mammoths as well as a variety of
other animals, including deer and buffalo................we have info that paintings depict animals but it is not known
whether they show any info regarding mammal population or human hunting mannerisms wrt mammals.

E. Because of the great effort that would have been required to produce each stone spearhead, hunters would have been
unlikely to leave them behind............this attacks the Perkins evidence why we could not find spearheads amidst the
bones of mammals. thereby weakening Perkins statement.

-----------------------------------------------------------------

DavidG@VeritasPrep GMAT Titan


Conclusion: humans not responsible for the extinction of the mammoth.
Premise: no spearheads have ever been found among mammoth bones

We're trying to discount this evidence (and show how humans may have been responsible for the death of the mammoth
despite the absence of spear heads in the mammoth bones.) Put another way, imagine the humans did, in fact, hunt the
mammoth to extinction. Why might there be no evidence of spear heads with the bones? E gives us exactly this. If the
humans retrieved their spears after killing the mammoths, the spear heads wouldn't have been found with the bones.

_________________
Veritas Prep | GMAT Instructor

Q.P1 CR| negation

A proposed change to federal income tax laws would eliminate deductions from taxable income for donations a taxpayer
has made to charitable and educational institutions. If this change were adopted, wealthy individuals would no longer be
permitted such deductions. Therefore, many charitable and institutions would have to reduce services, and some would
have to close their doors.
The argument above assumes which of the following?
(A) Without incentives offered by federal income tax laws, at least some wealthy individuals would not donate as much
money to charitable and educational institutions as they otherwise would have.
(B) Money contributed by individuals who make their donations because of provisions in the federal tax laws provides
the only source of funding for many charitable and educational institutions.
(C) The primary reason for not adopting the proposed change in the federal income tax laws cited above is to protect
wealthy individuals from having to pay higher taxes.
(D) Wealthy individuals who donate money to charitable and educational institutions are the only individuals who
donate money to such institutions.
(E) Income tax laws should be changed to make donations to charitable and educational institutions the only permissible
deductions from taxable income.
Keep in mind that assumptions are things that MUST be true, or else the argument falls apart. The (overly) simplified
form of this argument is this:

If we eliminate charitable tax deductions, many institutions will have to reduce their services.

The large gap b/w the premise and conclusion here is that eliminating deductions will somehow affect these services. It's
not going to happen b/c the institutions will look at these eliminations and think they should just reduce services. It's
because of the unstated effects of this elimination: wealthy people will no longer donate as much money. A states
exactly this. And the opposite of A would cause the conclusion to no longer hold true. If everyone donated the same
amount of money w/o the tax exemptions, then non-profits wouldn't have to reduce their services.

Your reasoning for D is correct. Even if non-wealthy people donate to these charities, the conclusion could still hold
true.

The same holds for B. What if only half of the money a non-profit receives is from people who are only doing it b/c of
the exemptions. If the non-profits lose half of their money, they still would have to reduce their services.
Joe Lucero
Manhattan GMAT Instructor

-------------------------------------------------------

Q. How to negate the choice A and B?

RON: when you negate these statements, you should always think about the negations in real-world terms. in other
words, you should basically use common sense to think about what it means if the statement is not true.
(you don't want to try to memorize "rules" for how to negate statements; that's at best way more complicated than it
needs to be, and at worst nearly impossible.)

(a) says, basically, "some rich people would give less money".
if that statement is false, then this means that no rich people would reduce their donations. (again, don't try to make a
"rule" for this; just ask yourself what it would mean for the statement to be false.)
if that's the case, then clearly the argument makes no sense anymore, because removing the deductions would have no
effect at all in that case.

(b) says that rich individuals provide all the funding.


if this is false, then you know that there are other sources of funding, too.
that doesn't destroy the argument, though: even if rich individuals are only one source of donations, a decrement in the $
amount that they donate could still lead to a reduction in services.

--------------------------------------------------

ceilidh.erickson GMAT Instructor

An assumption is something that MUST BE TRUE in order for the conclusion to be logically valid. If you take a
statement that must be true and flip it around, that opposite statement would by necessity DESTROY the entire
argument.

When you're applying the NEGATION test, do not ask "does this weaken the argument?" Weakening simply means
making it less likely to be true, but not necessarily invalidating. Instead, ask yourself "if I take the opposite of this
statement, does it COMPLETELY INVALIDATE the argument?" If so, that's your answer.

A. If we negate: "Without incentives, all wealthy individuals would still donate the same amount." This undermines the
entire argument --> Correct!

D. "Wealthy individuals... are not the only people who donate." Ok, if this is true, it hurts the argument, since the
argument focuses on wealthy people. But it doesn't invalidate the argument - the conclusion could still be true, even with
this negated statement. Thus, the answer choice was not something that HAD to be true.

Does that help?

Also, please always post your sources whenever you post a question!

_________________
Manhattan Prep

Distinguishing between "this helps/hurts the argument" and "this is NECESSARY to the argument" is a crucial
distinction.

Here's more to practice:


http://www.beatthegmat.com/outlaw-fishing-assumption-t168260.html#566447
http://www.beatthegmat.com/the-use-of-negate-in-assumption-problem-t164802.html#560777
http://www.beatthegmat.com/a-newly-discovered-painting-seems-to-be-the-work-of-one-of-t291321.html#776775

--------------------------------

GP1 | CR 70
According to a review of 61 studies of patients suffering from severely debilitating depression, a large majority of the
patients reported that missing a night's sleep immediately lifted their depression. Yet sleep-deprivation is not used to
treat depression even though the conventional treatments, which use drugs and electric shocks, often have serious side
effects.

Which of the following, if true, best explains the fact that sleep-deprivation is not used as a treatment for depression?

(A) For a small percentage of depressed patients, missing a night's sleep induces a temporary sense of euphoria.
(B) Keeping depressed patients awake is more difficult than keeping awake people who are not depressed.
(C) Prolonged loss of sleep can lead to temporary impairment of judgment comparable to that induced by consuming
several ounces of alcohol.
(D) The dramatic shifts in mood connected with sleep and wakefulness have not been traced to particular changes in
brain chemistry.
(E) Depression returns in full force as soon as the patient sleeps for even a few minutes.
Tommy Wallach | Manhattan GMAT Instructor | San Francisco

lifted does not mean raised in this context. It means more like "lifted away". If I say that my depression has lifted, it
means that it has gone away. To be more specific, here are two definitions taken from my trusty dictionary:

[ intrans. ] move upward; be raised : Thomas's eyelids drowsily lifted | their voices lifted in wails and cries.
• [ intrans. ] (of a cloud, fog, etc.) move upward or away : the factory smoke hung low, never lifted | the gray weather
lifted on the following Wednesday.

The question here refers to that second definition.

Ron: first, make sure that you note the purpose of the question. the purpose of the question is to provide a good reason
why we should NOT use sleep deprivation as a treatment for depression.

therefore, we must find a specific reason why sleep deprivation is ineffective against depression.

Option A

Ron: actually, this choice is worse than "irrelevant" -- this actually introduces another positive effect of sleep
deprivation. remember, we're looking for negativeeffects.

Option B

Ron: the comparison between depressed and non-dep people is irrelevant at best
correct.
also, two more things:
1) we are not sure whether it's actually difficult to do this. we are told it's more difficult than it is with non-depressed
patients ... but we don't know about the absolute difficulty. (for instance, if this is incredibly easy to do with non-
depressed patients, then "more difficult" may still refer to a very low level of difficulty.)
2) even if it is difficult to enforce the sleep deprivation, that's not necessarily a reason why we shouldn't use it as a
treatment modality. after all, if it is effective, then it should be used even if it is "difficult"• to use it.

Option C

Ron: choice (c) may argue against the practice of sleep deprivation in general, but it has little, if any, direct relevance to
the issue of depression. not only that, but it requires the assumption that this impairment of judgment is
somehow worse than the depression thereby alleviated (because otherwise it's still a beneficial tradeoff, even
considering the impaired judgment).

Option D

Ron: this choice just discusses the state of theoretical knowledge; there is no real relationship between treatment
modalities and what is written here.
i.e., this argument is solely concerned with, basically, what works and what doesn't. for the sake of this particular
argument, it doesn't really matter why anything works, or how.

Option E
Ron: choice (e) is CLEARLY relevant to the issue of depression, because it says, in so many words, that our alleged
depression treatment will not get rid of depression.

there's no question which of these answer choices is more relevant to the question at hand. don't think too much about
'the big picture' when the question is actually focused on 'the small picture'.

Discussion on Bold face

Bold face Term by Ron

1.CLAIM : a claim is basically any statement that's not factual. opinions, predictions, hypotheses, judgments, etc.
note that the other two categories you've mentioned-- position and judgment-- are subsumed under this category.

2. POSITION : position = the side someone takes on an issue.


normally, if someone takes a position in an argument, that position will be the conclusion of the argument.

3. judgment = an evaluation of something. (e.g., "this is a good/bad decision"; "this plan is/isn't likely to succeed"; etc.)

as you can tell, these two are closely related to each other.
the main difference is that a "position" is normally a side taken by someone who is personally involved in a situation (or
who cares deeply about it), while a "judgment" is usually emotionally detached.
you will not have to make such distinctions on this exam, of course.
"A generalization" and "an explanation" are not going to represent conclusions. A "judgment" could be a conclusion.
"Hypothesis" and "judgment" are both very "conclusion-like," they are not as explicit as conclusions.

"Evidence"• has to be factual. The second boldface is not a fact -- it's a claim -- so it can't be "evidence".

"Evaluate" and "justify", though, are HUGELY different. They're not alike at all.
To justify something is to say why you think its right.
I.e., you've already made up your mind that it's good. You're just saying why.
To evaluate something is to judge it (without necessarily having a pre-existing opinion on it). You might conclude that
it's good, that it's bad, or anything in between.

---------------------

Some terms defined by drimitry @manhattanprep

Premise
Fact
Evidence
Context
Consideration
Position
Principle
Judgment

Sure. Let's start with "premise," "fact," and "evidence." These are all different words for a statement that we accept as
true. For the most part, these terms are interchangeable, but "premise" and "evidence" imply that the information is
being used to support an argument, while "fact" might in some cases refer to information that is not used to support one
side or the other.
"Context" is going to be more neutral. If something is context, it is providing the background for the argument, but
doesn't itself constitute an argument. Typically, context will be factual, but some kind of initial position or disagreement
could also serve as context (this is less likely).

A "consideration" is something we need to consider in making a decision. This will generally also be a premise, but it is
likely to be something new that is introduced after an initial argument. For instance, I might say that because John was
late to his interview, we shouldn’t hire him. If you point out that John was hit by a car on his way to the interview, that
would be a consideration introduced to counter my conclusion.

A “principle” is a different sort of premise—it’s not a fact, but rather an initial idea upon which we build. For instance, I
might say “No one who has committed a felony should be allowed to run for governor. Candidate X robbed a bank, so
she should not be allowed to run for governor.” In this case, “Candidate X robbed a bank” is a factual premise, while
“No one who has committed a felony should be allowed to run for governor” is a principle. For the purposes of the
argument, we would grant both of them as true. Therefore, the only assumption in this argument is that robbing the bank
was a felony.

“Judgment” is tricky. It sounds like a conclusion, but it may refer to a sub-conclusion or principle. For instance, in #123
in OG 13, it is used to refer to a premise. We are basically told that “If people are right to believe X, then Y is also true.”

“Position” means conclusion. It may refer either to the author’s conclusion or an opposing position that the author is
refuting.

------------------------

Daagh

A small note on bold face questions-- The cue words that can expose the relationship

The correct interpretation of the action words largely brings out the relation between the two bold face statements. It is
important to understand the basic tenets of CR passages such as premises, assumption, the conclusion etc; we must also
be thorough with what the argument seeks to establish, or seeks to oppose or in effect what direction is the argument is
heading on. The primary purpose of the argument or the focus of the passage should be clear in the mind before we get
down considering the choices.

The broad classification of the descriptive words is

1) those that are premises or evidence that lead to a conclusion; those that support or oppose a conclusion, unstated
premises such as assumptions. These are facts and mostly derive from the present or past. The cue words for this
category are since, because, although etc.

those that are conclusions, interim conclusions or final or main conclusions; those that are speculations, judgments,
opinions, stance, stands, feelings; These may not be facts and are more in the nature futuristic events and liable to
become wrong.

3) words like claim, consideration, position, strategy, policy ,objection undermining, drawback, define, describe which
are subjective expressions.

1. Premises words
a finding – a discovery , a revelation
an assumption: -- a non-existing factor, to be drawn to lead to a conclusion, invisible factor; take for granted
Consideration - A factor to be considered in forming a judgment or decision
Evidence - grounds in support of that consideration
Phenomenon – an occurrence, a thing of past or present
the circumstance- context, setting, the atmosphere, a situation
Observation—seeing. the past or the present
Platform – a ground from which something is derived

2. Conclusion words :

a hunch – a premonition that something is likely to occur


a judgment – personal assessment
a prediction, guess, premonition, intuition, instinct
a proposal – a futuristic offer
a recommendation – a futuristic advice
conclusion – What the argument seeks to establish
contrary conclusion – opposite decision
Generalization- a general idea developed from smaller premises, a principle, statement, or idea having general
application. Can be both part of a conclusion or an observation
interim conclusion, a minor conclusion leading to the main conclusion
Judgment opinion - thinking , view, estimation
main conclusion - the final conclusion
Opinion, - personal view
Proposal – Proposition, Suggestion
Speculation, -- uncertain, like to happen not a fact

3) descriptive words

a claim – an assertion – averment - insistence


a drawback – a reverse
a policy - guideline, rule
a strategy - modus operandi , a method to reach an aim
an objection – opposition, criticism , undermining argument :

The argument The main point of the topic the text the passage seeks to establish. - the point of the passage supported by
the argument
calls that conclusion into question.- doubts, undermines , weakens , disagrees , disputes
challenges (verb) – questions, negates,
to claim - to demand, to maintain
Contends: to controvert or debate; dispute.
Context- platform, opportunity, situation. Scenario
Contrary --Opposed; contradictory; repugnant; inconsistent criticism
Define: give the meaning of the word, like dictionaries do.
Demonstrate – prove
Describe: give some characteristics of the item defined.
Explain: give reasons why the item is what it is or does what it does; provide more complete information
Evaluate - To ascertain or fix the value or worth of; to examine and judge; weigh carefully; appraise
explanation – giving reasons
identify - recognize, locate,
Illustrate -, explain with examples, to buttress the author’s point
Imply – mean without mentioning
Implications – after effects; side effects
Infer- deduce something from stated facts
In part --- based on one of the premises
in support - to endorse;
in support of the overall position – to agree with the argument
Policy – principle, rule, guideline
potential objection – possible, or implied objection
Presents - to offer for observation, To introduce
presents a situation – provides a platform
provides further evidence – fortifies, reinforces with additional facts
provides support– goes in tandem with the reasoning
questioning – doubting, criticizing, undermining,
supplies a context - provides a ground
support an opposing position – to oppose
that the argument disputes - the argument disapproves, or disputes
the content – the actual wordings or expression
the course - the path
the position A stand taken, one’s thinking ,
to challenge – to question, to criticize .
to support the position – to support some stand
to undermine- weaken, call in to question, raise doubt
an appeal to authority – an automatic acceptance of an expert’s or celebrity’s opinion

While the above is a general idea of the usage of these words in boldface questions, they have to be understood in the
special context in which they are presented. Some of them may also be on borderline; So you must use your contextual
judgment in such cases.

---------------------

ketanth

Manager, GC

Bold Face Questions:

The last hurdle between you and the 700+ score you so richly deserve. But these questions seem to cause a lot of anxiety
among test-takers and test-takers-to-be. Why? I think that it is because these questions are strange and uncharted.
Approaching the BF question is a bit like trying to read Dostoevsky, in Russian, while stumbling around in the dark, in a
room full of holes. It’s disorienting and confusing and generally unpleasant. There are no clear references and no decent
guides. We fear the unknown. We try to avoid what we do not understand. But given our common goal, let’s get to know
the BF question. Let’s come to understand it. If we know it and understand it, then we can kill it.

When you see one of these questions in your actual GMAT, the first thing you should do is congratulate yourself on
having done well enough to have brought the BF challenge upon yourself in the first place. In the final analysis NOT
getting a BF question or two during the exam is definitely WORSE than getting them. So, you’re already doing
something right. The key is to use what you know to split the answer choices. Here’s my approach:

(1) Read the argument. Read it quickly, as you ask yourself, “What’s the point here?”

(2) Identify the Main Conclusion. You’ve got to identify the main conclusion to proceed—the main conclusion is your
“port of entry” into the BF question. So, find main conclusion as quickly as possible and note whether it is one of the
bolded phrases.

(3) Go directly to the answer choices. Do not, I repeat DO NOT, spend any time trying to figure out what roles the
bolded phrases play within the argument without some idea of the terms that are being offered in the answer choices. It’s
a waste of precious time.

IF the main conclusion IS one of the bolded phrases, then find the answer choices that offer that option for the respective
bolded phrase (first or second). A significant number of BF questions can be answered correctly with this information
ALONE. If there is only one choice that matches up with the bolded main conclusion then you’re done. Mark it and
move on.

Otherwise…

-Have a quick look through the choices to discover what the terms in play (see below).

(4) Return to the argument and determine the relationship between each bolded phrase and the argument’s main
conclusion. Do they basically agree with the conclusion of the argument? Does one but not the other? Neither? What
other relationships occur to you?

(5) Return to the answer choices and use these relationships to discard at least two and probably three choices. Let’s
have a review of key terms:

Main Conclusion—a summary of the argument’s primary position;


Intermediate Conclusion—a position utilized by the argument as a stepping stone in order to advance toward the main
conclusion;
Premise—a theory or proposition upon which an argument is based or from which a conclusion is drawn;
Fact—information generally believed to be true OR known to be true—usually advanced as evidence to support a
premise;
Evidence—specific type of fact offered in support of a theory or premise;
Context—a frame of reference of value in the interpretation of aspects of an argument or the argument’s components;
Consideration—a factor (fact) to be taken into account in forming a judgment or decision;
Position—a point of view or attitude about an issue or question;
Assumption—a position or belief that is taken to be true, without proof;
Principle—a basic or essential truth (stronger and broader than a fact).
Judgment—an opinion formed from a consideration of the facts.

(6) Now, take each of the remaining choices one by one, matching similar parts of each answer choice to their respective
BF phrase, then discriminate between the dissimilar parts of each answer choice and their respective BF phrase. That
should take you the rest of the way home.

Lets’s face a typical BF CR question:


“Environmental organizations want to preserve the land surrounding the Wilgrinn Wilderness Area from
residential development. They plan to do this by purchasing that land from the farmers who own it. That plan is ill-
conceived: if the farmers did sell their land, they would sell it to the highest bidder, and developers would outbid any
other bidders. On the other hand, these farmers will never actually sell any of the land, provided that farming it
remains viable. But farming will not remain viable if the farms are left unmodernized, and most of the farmers lack the
financial resources modernization requires. And that is exactly why a more sensible preservation strategy would be to
assist the farmers to modernize their farms to the extent needed to maintain viability.”

In the argument as a whole, the two boldface proportions play which of the following roles?

A. The first presents a goal that the argument rejects as ill-conceived; the second is evidence that is presented as grounds
for that rejection.
B. The first presents a goal that the argument concludes cannot be attained; the second is a reason offered in support of
that conclusion.
C. The first presents a goal that the argument concludes can be attained; the second is a judgment disputing that
conclusion.
D. The first presents a goal, strategies for achieving which are being evaluated in the argument; the second is a judgment
providing a basis for the argument’s advocacy of a particular strategy.
E. The first presents a goal that the argument endorses; the second presents a situation that the argument contends must
be changed if that goal is to be met in the foreseeable future.

Main Conclusion: “A more sensible preservation strategy would be to assist the farmers to modernize their farms to the
extent needed to maintain viability.” So, we didn’t luck out and get the answer just by having identified the main
conclusion. No problem.

Terms in play:

A. (1) A goal (that the argument rejects)


(2) Evidence (as grounds for the rejection)
B. (1) A goal (that the argument judges as unattainable)
(2) Grounds (support for that judgment)
C. (1) A goal (that the argument judges as attainable)
(2) A refutation (of that judgment)
D. (1) A goal (with the strategies for attainment in question in the argument)
(2) Reasoning (for supporting ONE of the noted strategies)
E. (1) A goal (endorsed by the argument)
(2) A factor (effecting the timely attainment of that goal)

Back to the Bold-Faced phrases to determine their relationship to the main conclusion:

BF 1: Preservation of the Wilgrinn land (that’s a goal) => The plan is ill-conceived (not the goal, but the plan) => So,
the goal is preservation of the land (First BF) and that’s also part of the main conclusion => First BF is a goal with
which the argument basically agrees => A and B are gone.

BF 2: Doesn’t really relate to the main conclusion, so

Return to the answer choices and consider second BF description in remaining choices—C, D, and E => E implies that
the argument suggests changing the approach to a goal; it says nothing about changing a situation of any sort => E is
gone. That leaves us with C and D.
Read Choice C in its entirety:

(1) Says the first BF presents a goal that the argument concludes can be attanined. Not exactly—the argument actually
concludes that a different strategy is needed to attain the goal, not simply that the goal is attainable (so this part of this
choice doesn’t match the argument).

(2) Says the second BF in the argument is a judgment disputing whether the goal can be attained. NO, definitely not—
the second BF in the argument suggests a reason why one strategy won’t succeed, but has nothing to do with whether the
goal can be attained (this part of this choice doesn’t match the argument, at all) => C is gone.

Now look at the remaining choice, to make sure it fits:

(1) Goal, strategies for achieving under consideration (YES).

(2) basis (judgement) for supporting an alternative to the earlier plan aimed at achieving the same goal—preservation of
Wilgrinn land (YES).

Done—mark it and move on. That’s it.

Let’ know some basic terms to ace the BF CRs of GMAT:

Principle: something fundamental that we do not question. This would be somewhat stronger than a fact because it is not
specific to a limited number of cases but instead, apply to a broader range of scenarios (and often deeper in meaning).
For instance, you will not talk about the principle that crime is increasing in large cities. Instead, it is a fact which
applies to large cities. However, you will talk about the principles of Physics or the fundamental principles of Human
Rights. Principles convey a stronger connotation than mere facts.

Fact: something taken as true at face value (stats, historical events)

Evidence: what is used to support a conclusion (examples, stats, historical events). Although these may include facts, it
is usually stronger than facts because they are direct elements needed for the conclusion to stand whereas facts are not
necessary for the latter to stand

Pre-evidence: This is a bit of a stretch. It will not often be on the test but it seems very similar to "background"
information as described below.

Background: Elements needed to put the evidence into context but which, as stand alone pieces of information, might
not constitute what is called an evidence necessary to arrive at a conclusion. For instance, blood tests performed on one
thousand persons may reveal that 35% of those persons were HIV infected. However, the background information could
be that the test was performed in more under-informed regions of the world where AIDS knowledge is at a minimum.
As you can see, the fact that the test was performed in more under-informed regions is not in and of itself an evidence
because it does not allow us to come to a conclusion. Instead, the 35% stats, as a stand-alone piece of info, is what will
lead us to the conclusion we want. However, the background info is also crucial and cannot be omitted; it is required
background info.

Consideration: Something which was taken into account or given some thought before arriving at the conclusion.

Premise: For GMAT purpose, Premise and Evidence are the same.
Assumption: Unstated information which will link the argument to a logical conclusion. Without this, the argument falls
apart.

Inference: Something that might not be explicitly stated or proved. For instance, you may say that 95% of GMAT test-
takers have over 340. We can reasonably infer that Anthony will get more than 340 on his GMAT based on the fact
given.

Argument: Central to every CR question is the argument. An argument is an ordered line of reasoning composed of
premises, assumptions, and a conclusion. Understanding the elements of an argument is essential to performing well in
this section.

Conclusion:
The conclusion is the endpoint of the line of reasoning of an argument. Think of it as the result of the argument. The line
of reasoning leading to a conclusion is often where errors in logic are made. Conclusion can be defined as the last
deduction or claim

BF CRs Wrap-Up:
1. Identify the conclusion. Ask yourself what the conclusion is, what the author trying to prove, or what the author’s
main point is.
2. Look for the evidence that the author uses to support or argue against a position.
3. Search for argument indicator to determine the relationship between evidence and conclusion.
Conclusion Indicators
So thus therefore as a result
consequently accordingly hence imply
conclude that follows that means that infer that

Premise Indicators
because since for as
If assume suppose evidence
on the basis of the reason is that may be derived from in that

Counter-evidence Indicators
actually despite admittedly except
even though nonetheless nevertheless although
however In spite of do may

--------------------

A video lesson (7 mins) on bold face from optimus prep

https://www.youtube.com/watch?v=JRNFK2VkhyM

Thursdays with Ron- July 25, 2013


https://vimeo.com/73383001

an article from economist gmat

http://www.beatthegmat.com/mba/2012/07/11/boldface-type-questions-when-simply-breaking-down-the-argument-is-
not-enough

example and explanation from karishma

https://www.veritasprep.com/blog/2017/04/tackling-gmat-critical-reasoning-boldface-questions/

-----------------------------

How to approach Bold face by Matt@veritasprep

Here's what I'd recommend for roles questions:

(1) Make sure you identify the conclusion of the argument, if there is one. Most roles questions will try to tempt you into
saying that one of the two boldface portions is the conclusion, so make sure you know where that conclusion is.

(2) Read EVERYTHING. It's not safe to only read the boldface portions: you need to know their place in the argument
as a whole.

(3) Try putting the argument in your own words first. Don't be intimidated by the density of the argument; most roles
questions feature quite complex arguments -- more complex than other CR arguments -- but this is because your
understanding of the argument doesn't have to be particularly profound. (You don't need to strengthen it, weaken it, or
draw any conclusions from it, only describe it in broad terms.) So put in your own words first, then try to find an answer
that matches your description.

(4) If you're still stuck, look for excuses to eliminate answers. Is the first bold statement really a "contention", or is it just
a fact related by the author? Is the second statement really a "consequence" of the first one, or is merely the opposing
view? etc.

------------------------

Advice by Brent @gmatprepnow

As you are trying to identify the roles played by the boldfaced part(s), keep in mind some of the common roles that
statements can play in arguments. They INCLUDE:
- Concluding
- Summarizing
- Contradicting
- Providing supporting evidence
- Providing an example
- Providing a counterexample
- Generalizing

Cheers,
Brent

----------------------------

e-gmat blog

https://e-gmat.com/blogs/bold-face/

BOLD FACE

WHAT DO BOLD FACE QUESTIONS TEST?

Bold Face questions test your ability to analyse complex arguments or reasoning.
A Bold Face (BF) question contains an argument with one or more highlighted (bolded) statements. The question stem
asks you to select the answer choice that describes the role played by these highlighted statements.

UNDERSTANDING LOGIC VS. APPLICATION


This is different from other Critical Reasoning (CR) questions, such as assumption, weaken/strengthen, evaluate, or
inference questions that test your ability to understand the logic of the arguments and to select a fact or evidence that
would provide additional information to strengthen or weaken the argument. Thus, in these questions you not only need
to understand the logic of the argument and but also apply it in some way.
Bold Face questions, on the other hand, are just about understanding the logic, not its application. While one part is
eliminated, the other part i.e. understanding of the logic is made more difficult. So, one conclusion and two or three
statements (evidences or facts or opinions); a passage in a BF question would generally be longer to make the logic more
complex.

SO WHAT ARE BOLDED STATEMENTS?


These statements may play different roles – premises, intermediate conclusions, facts, opinions, judgements,
interpretations, claims, justifications, etc. In addition, these statements may come in any order. For example: an opinion
may be supported by a fact, then countered by some other evidence and then again favoured for some other reason. Your
job, then, is to figure out what is happening, without getting bogged down by the complexity.

WHAT CORE SKILLS SHOULD YOU MASTER TO ANSWER BOLD FACE QUESTIONS?
1. THE ABILITY TO CLASSIFY BOLD FACE STATEMENTS CORRECTLY
The first foundational skill you need to tackle Bold Face questions is your ability to classify (bolded) statements
correctly. This means an ability to think clearly in complex scenarios in the face of facts, evidences, counter-evidences,
opinions, judgements all thrown together. This requires an ability to remove the disguise of appearances to figure out the
real purpose of that statement in the argument. Read the example below:
 Arg1: Since the average GMAT Club member scores 50 points higher than the average aspirant does, it is clear
that GMAT Club helps students perform better on the GMAT.
 Arg2: Since GMAT Club helps students perform better on the GMAT, Ron should join GMAT Club.
Note, the highlighted (bolded) statement is the same in both the statements. However, in the first argument, the bolded
argument is a claim made by the author. It is also the author’s conclusion, while in the second argument, the author uses
the highlighted statement as a fact (used as a premise) to substantiate his recommendation (conclusion).
In BF questions as well as in real life, you will encounter opinions presented as facts and facts presented as mere
coincidences – all it boils down to is your ability to understand how an argument is framed and correspondingly classify
a statement appropriately.
While some people are naturally good at handling complex reasoning, others can develop this skill by reading diverse
materials, gradually increasing the complexity level and by applying their logical skills to evaluate the reasoning in these
reading materials. The key is to proactively develop this skill.

2. CLEAR UNDERSTANDING OF COMMONLY USED TERMS


Terms such as claim, intermediate conclusion, fact, evidence, principle, belief, etc. should be part of your vocabulary.
This implies that you should understand clearly what a fact is and how it is different from a claim. This is because the
answer choices in a Bold Face question will use these terms.
Another aspect of this key skill is to understand which of the terms above can be used interchangeably. For example, in
most instances, evidence and fact can be used interchangeably. Similarly, claim and opinion can be interchanged in most
instances as well.

3. THE ABILITY TO LINK THE HIGHLIGHTED STATEMENT TO THE CONCLUSION OF THE

ARGUMENT.
Another crucial skill, this question assesses is your ability to ascertain whether a highlighted (bolded) statement
supports/refutes the conclusion. There are many ways to determine this. Some people use the negation test – perfectly
legitimate way to assess the linkage to the conclusion. You negate the highlighted statement. If the conclusion becomes
less believable in the light of the negated statement, then the highlighted statement supports the conclusion, or vice
versa. Let’s apply this to the argument above:
 Original Argument: Since GMAT Club helps students perform better on the GMAT, Ron should join GMAT Club
 Conclusion: Ron should join GMAT Club
 Negated bolded statement: GMAT Club does not help students perform better on GMAT.
 Impact: As you negate the bolded statement, the conclusion becomes less believable. Hence, the original statement
supports the conclusion.

QUOTE:
However, since negation requires time and since most people are not as familiar with the negation test, we have devised
a simpler test – the disappearance test. To apply this test, just remove the bolded statement from the argument. If the
conclusion becomes stronger in the absence of highlighted statement, then the highlighted statement weakens the
conclusion and vice versa.

4. UNDERSTANDING THE PURPOSE OF THE ARGUMENT

This core skill is tested in a few Bold Face questions (about 25%). This skill tests your ability to understand why an
argument is written. Let take an example below to highlight this point:
Most people have opposed the recent tax increase by the president because they believe that the tax increases will
reduce their disposable income and hence have negative impact on the economy. However, the proposed tax increase
only impacts 5% of the population who have amassed huge amounts of wealth, and thus this tax increase is not likely to
impact the economy

What is the purpose of the bolded statement?


A. It describes a circumstance that the argument seeks to explain.
B. It describes a fact that has been taken as opposing the conclusion.
Even though the bolded portion is a circumstance, Choice A is incorrect because the purpose of the argument is not to
explain the circumstance. The purpose of the argument is to debunk the belief that the proposed tax increase will have a
negative impact on the economy.

HOW SHOULD YOU APPROACH BOLD FACE QUESTIONS?

Possessing core skills is like having an ability to walk; however, to reach your destination, you also need a road to walk
on. It would be even better if the road is robust ( fewer pitfalls) and provides an efficient way to your destination.
Similarly, while you work on your core skills, you also require an effective and efficient approach, which lets you
handle the BF questions within the time constraints of the test. Here is the approach that we recommend:
1. Read the passage actively. Make sure that you understand the logic of entire argument. The best way to do this is by
classifying each statement and linking it to the previous statement. Pay special emphasis on the highlighted (bolded)
statements.
2. Classify the bold face statements to prethink the answer: Classify each bold face statement. Write the following about
each statement;
 what is the statement
 how does it relate to the conclusion (Strengthen, Weaken, no relation).
3. Evaluate the answer choices and select the correct answer.

SOLVED OFFICIAL QUESTION

In countries where automobile insurance includes compensation for whiplash injuries sustained in automobile
accidents, reports of having suffered such injuries are twice as frequent as they are in countries where whiplash is
not covered. Presently, no objective test for whiplash exists, so it is true that spurious reports of whiplash injuries
cannot be readily identified. Nevertheless, these facts do not warrant the conclusion drawn by some commentators that
in the countries with the higher rates of reported whiplash injuries, half of the reported cases are spurious. Clearly, in
countries where automobile insurance does not include compensation for whiplash, people often have little
incentive to report whiplash injuries that they actually have suffered.
In the argument given, the two boldfaced portions play which of the following roles?
A. The first is a claim that the argument disputes; the second is a conclusion that has been based on that claim.
B. The first is a claim that has been used to support a conclusion that the argument accepts; the second is that conclusion.
C. The first is evidence that has been used to support a conclusion for which the argument provides further evidence; the
second is the main conclusion of the argument.
D. The first is a finding whose implications are at issue in the argument; the second is a claim presented in order to argue
against deriving certain implications from that finding.
E. The first is a finding whose accuracy is evaluated in the argument; the second is evidence presented to establish that the
finding is accurate.

ARGUMENT ANALYSIS AND PRETHINKING


Start by actively read the passage once and then, look at the options.
1. “In countries where automobile insurance includes compensation for whiplash injuries sustained in automobile
accidents, reports of having suffered such injuries are twice as frequent as they are in countries where whiplash is
not covered” – The statement is written in a factual form, like reporting of a finding. The author simply states a fact that
whiplash injuries are more commonly reported in countries where the automobile insurance includes compensation for
such injuries.
2. “Presently, no objective test for whiplash exists, so it is true that spurious reports of whiplash injuries cannot be readily
identified” – The author first presents a fact – no objective test for whiplash exists – and then presents his opinion or an
intermediate conclusion – so it is true that spurious reports of whiplash injuries cannot be readily identified.
3. “Nevertheless, these facts do not warrant the conclusion drawn by some commentators that in the countries with the
higher rates of reported whiplash injuries, half of the reported cases are spurious” – This statement is quite complex and
may induce errors in understanding in a lot of people. Let’s understand. Firstly, it begins with ‘Nevertheless’, which
means that something counter to the previous statement is going to come here. “these facts…” here refer to previous
conclusion (that spurious reports of whiplash injuries cannot be readily identified) and the first statement. You can see
how an intermediate conclusion is referred to as a fact in the next statement. So what is this statement – this statement is
author’s opinion and may be the overall conclusion of the argument.
4. “Clearly, in countries where automobile insurance does not include compensation for whiplash, people often have
little incentive to report whiplash injuries that they actually have suffered.” – Read it along with the previous
statement. In this statement, the author provides justification for his opinion that the commentators are wrong in drawing
their conclusion. The author says that in countries which don’t include compensation for whiplash injuries, people don’t
have the incentive to report such injuries. This statement is author’s justification to support his conclusion.
BF1: Fact from which a conclusion has been drawn in the argument.
BF2: Author’s justification to support the conclusion of the argument

ANSWER CHOICE ANALYSIS


Now, let’s look at options C, D & E
 Option C – BF1: we see that BF1 is indeed an evidence of the conclusion of commentators. Besides, there is further
evidence for this conclusion in statement 2, which is also used as a premise for the conclusion.
BF2: As we have understood above, BF2 is not the main conclusion. It is author’s justification to support the conclusion
of the argument.
So, option C eliminated.
 Option D – BF1: We see that BF1 is a finding whose implication could refer to the conclusion drawn by the
commentators. This conclusion is at issue. First part is correct.
BF2: is it a claim? Yes, it is presented like a view of the author. Is it presented against some implication of the BF1?
Yes, it is presented as a view to counter the conclusion of the commentators.
So, option D could be correct.
 Option E - We have seen that BF1 is treated like a fact in the argument and its accuracy is nowhere evaluated. Thus, this
is incorrect.
So, we find that option D is the correct answer.

KEY TAKEAWAYS:

Bold Face questions test your ability of complex reasoning, which essentially, is a test of your skill to think clearly and
logically in complex scenarios.
While development of these skills is essential, an efficient and effective approach is also required to achieve the desired
results.
Presence of skills and a good approach will take you far; however, to achieve mastery, you will also need practice.
Practice of BF questions will allow you to figure out your common pitfalls so that you can avoid those during the actual
test.
GP Exam Pack1

People with a certain eye disorder are virtually unable to see in moderately bright light, which seems to them unbearably
intense, since the cells of their retinas are overwhelmed by moderately bright light. These people do, however, show
normal sensitivity to most components of dim light. Their retinal cells are also not excessively sensitive to red
components of moderately bright light.

The information above best supports which of the following hypotheses about people with the disorder described, if they
have no other serious visual problems?

A. In all moderately dim light in which people without the disorder can read large print, people with the disorder cannot
read such print.
B. In an otherwise darkened concert hall, these people will see a dimly illuminated red exit sign more clearly than small
dim white lights that mark the aisles.
C. These people typically see more acutely at night and in dim light than do most people who do not have the disorder.
D. Eyeglasses that are transparent to red components of light but filter out other components of light help these people
see in moderately bright light.
E. These people perceive colors other than red in the same way as do most people who do not have the disorder.

Dear MrSobe17,

Here goes analysis of this question:

FACT 01: Certain people unable to see in moderately bright light because::

Cells of their retinas are overwhelmed by moderately bright light

FACT 02: These people do, however::

Show normal sensitivity to most components dim light + their retinal cells are also not excessively sensitive to red
components of moderately bright light.

Argument Analysis (One line summary):


Certain people cant see in moderately bright light but can see in dim light and show normal sensitivity to red
components of moderately bright light.

Analysis of answer choice E

E. Eyeglasses that are transparent to red components of light but filter out other components of light help these people
see in moderately bright light

As per this answer choice if people will wear eyeglass that is transparent to red component of light (will provide no issue
to category of people we are discussing as they have normal sensitivity of red component) and that will filter out other
components (there would not be stimulation of retinal cells), then the people would PROBABLY able to see in
moderately bright light

Hope it helps!!!!

LakerFan24, Manager

main idea here (that i will repeat in my answer explanations) is: people w/ a certain eye disorder cannot see in
moderately bright light. BUT, they can see RED components of moderately bright light. What does this mean?
These people w/ the disorder must not be able to see other color components in moderately bright light (i.e. blue, green,
etc).

Option B

mvictor

This cannot be inferred from the argument. The argument does not state that people with disorder see better in dim light.

abhishekdadarwal2009

The ratina of people with this discussed disorder is insensitive to the red light or red frequency of the visible light,so
even if there is no light but red they will be able to see less.

warriorguy, Chat Moderator

in dim light they show normal sensitivity.

LakerFan24, Manager

can't make this comparison b/c no info is given

Option D

abhishekdadarwal2009

Correct answer since the people with the disorder cannot bear the intensity of bright light and they are less sensitve to
red light they can see better with glaases that filter all but red color .Correct answer.

mvictor
Aha! The argument tells that in red components of moderately bright light do not make the retina cells more sensitive.
Looks good.

LakerFan24, Manager

MUST be true. idea here is that people w/ this disorder can see red components of moderately bright light...the "issue" is
that these same people cannot see other components of moderately bright light well (i.e blue, green, etc.) by filtering out
these other colors (that people w/ the disorder cannot see well), people with disorder will be able to see in moderately
bright light

GP Exam Pack 1

Whales originated in the freshwater lakes and rivers of ancient Asia about sixty million years ago. Not until about ten
million years later did species of whales develop specialized kidneys enabling them to drink salt water. Although fossil
evidence shows that some early whale species that lacked such kidneys sometimes swam in the Earth’s saltwater oceans,
these species must have had to return frequently to freshwater rivers to drink.

Which of the following is most strongly supported by the information given?

(A) Fossils of whale species dating from between sixty million and fifty million years ago will not be found on
continents that were at the time separated from ancient Asia by wide expanses of ocean.

(B) Among whale fossils that date from later than about fifty million years ago, none are fossils of whale species that
drank only fresh water.

(C) Fossils of whale species that drank fresh water will not be found in close proximity to fossils of whale species that
drank salt water.

(D) The earliest whales that drank salt water differed from fresh-water-drinking whales only in their possession of
specialized kidneys.

(E) Between sixty million and fifty million years ago, the freshwater lakes and rivers in which whales originated were
gradually invaded by salt water.

Word meaning:

Expanse (n.) - an area of something, typically land or sea, presenting a wide continuous surface.
karishma

This is an inference question - Which of the following is supported by the information given?
So the argument supports one of the options. So the correct option provides the conclusion.
Option (A) provides the conclusion.

A strengthen question would be framed something like this: Which of the following best supports the argument/author's
position etc?
So here you are looking for a strengthener.

-------

The question stem is:


"Which of the following is most strongly supported by the information given?"

So you need to find the option that is supported BY the argument. The argument supports one option. Hence, that option
would be the inference.

The word "support" could indicate either strengthen or inference question type. If the argument supports the option, you
are looking for an inference. If you need the option to support the argument, you are looking for a strengthener.

----------------

Karishma: Here is the explanation:


- 60 million yrs ago, whales originated in freshwater (which means lakes and rivers) of Asia
- Special kidneys that allowed whales to drink salt water evolved 50 million yrs ago.
- Sometimes whales without special kidneys swam into saltwater but they had to return frequently to freshwater.

Think about your geography - Continents are separated by huge saltwater oceans. A continent has lakes and rivers
(freshwater) within it. The rivers connect to the ocean. If you want to travel from Asia to North America by water, you
will need to go through the huge ocean between them.
60 million years ago, whales originated in Asia's freshwater so they were in the continent of Asia. They did not have
special kidneys so they could not travel across the ocean. Even if they did enter the ocean, they needed to return back to
the freshwater of Asia. Point is, they couldn't swim far from Asia. Till 50 million yrs ago, special kidneys did not
develop. So for the 10 million yrs from 60 million to 50 million yrs ago, whales couldn't leave the coast of Asia and
swim far. So they couldn't reach the continents far from Asia separated by saltwater oceans.

Answer (A)

Option A

A. Fossils of whale species dating from between sixty million and fifty million years ago will not be found on continents
that were at the time separated from ancient Asia by wide expanses of ocean.
Clearly supports the statement from the prompt that although some whales did swim in salt water oceans, they had to
return to freshwater rivers to drink. Hence it is implied that whales could not have travelled vast expanses of oceans.
Consequently fossils of whales cannot be found in such continents that were very far from Asia

mejia401, Senior Manager, GC


OK - The conclusion must tie together two disparate claims: whales originated in Asia + species must have returned
after swimming into salt water. This is the only option that supports the authority of the claims. Take note of the "must"

DanaJSite Admin ,btg

Think about it this way: if they need to drink only fresh water (don't have the appropriate kidneys yet), they can't travel
for long without dying of thirst.

Option B

B. Among whale fossils that date from later than about fifty million years ago, none are fossils of whale species that
drank only fresh water.
scorpionz, Senior Manager, GC

Contradicts the information in the prompt that only around 10 million years ago saltwater whales started emerging. Also,
emergence of saltwater whales does not mean that freshwater whales ceased to exist

mejia401, Senior Manager, GC

Reverse answer. This conclusion does not make sense after the premises given.

mbaapp1234, Manager

B) "None" is a strong word. Perhaps one such fossil was found.

DanaJSite Admin ,btg

B is not necessarily true. While we are told that 'species of whales' developed the ability to drink salt water around 50
mil years ago, there is no 'all' in that phrase. Some of the species might not have developed those kidneys even after 50
mil.

Option C

C. Fossils of whale species that drank fresh water will not be found in close proximity to fossils of whale species that
drank salt water.
scorpionz, Senior Manager, GC Irrelevant. Nothing in the prompt supports or contradicts this.

mejia401, Senior Manager, GC

Reverse answer. The fossils must be found in close proximity because "these species" must have had to return.
KyleWiddison, Manhattan GMAT Instructor

This question asks us to determine which choices is supported by the argument, or in other words we need to draw a
conclusion. The correct answer, or conclusion, will be supported by the argument, which states: 1 - Whales started in
fresh water (60 million yrs ago) 2 - they took 10 million years to get kidneys good for salt water 3 - some whales w/out
these kidneys did go in salt water but they must have had to return to fresh water.

Choice C says there were no mammalian species able to drink salt water before 50 million years ago. The only
information we have from the argument relates to whales. We are given no information about other mammals so this
choice is not supported by the argument.

Patronus, Current Student

It is possible that area where salt water and fresh water died is the same, like some end of river/ river basin.

DanaJSite Admin ,btg

C is again not necessarily true. You might think that each species spent time in its specific habitat (i.e. salt water vs.
fresh water), but we are told in the text that some early whale species that drank fresh water sometimes swam in salt
water. This is already a proof that the development stage of the kidney (and therefore the ability to drink salt water or
fresh water) does not imply that whales spent all their time in this or that habitat. They might be moving around a bit.

Option D

D. The earliest whales that drank salt water differed from fresh-water-drinking whales only in their possession of
specialized kidneys.
scorpionz, Senior Manager, GC Nothing in the prompt supports this. Specialized kidneys has been mentioned as an
attribute of saltwater whales, but not the ONLY attribute.

mejia401, Senior Manager, GC

Could be true; shell game. It's tempting because indeed the kidneys must have been different. Even so, it need not be
true. If it were true, it would need different support that what is provided.

DanaJSite Admin ,btg

D - there's actually no info to support this. What if they were of a different color? There's nothing in the text to point to
the fact that they were exactly identical.

Option E

E. Between sixty million and fifty million years ago, the freshwater lakes and rivers in which whales originated were
gradually invaded by salt water.
scorpionz, Senior Manager, GC Irrelevant. Nothing in the prompt supports or contradicts this.
mejia401, Senior Manager, GC

Could be true. If there freshwater lakes were not invaded by salt water, it is still possible that whale's kidney evolved
because of a different cause.

DanaJSite Admin ,btg

E could be a possible inference to explain why whales developed the special kidneys: they had to adapt to the changing
environment. But this is not the unique explanation. There are others, such as the fact that salt waters contained more
food than fresh waters. Some whales might have developed the special kidneys because of feeding needs and not
necessarily because of a gradual change in their habitat.

GP Exam Pack 1

Fact: Asthma, a bronchial condition, is much less common ailment than hay fever, an allergic inflammation of the nasal
passages.

Fact: Over 95 percent of people who have asthma also suffer from hay fever.

If the information given as facts above is true, which of the following must also be true?

A. Hay fever is a prerequisite for the development of asthma


B. Asthma is a prerequisite for the development of hay fever
C. Those who have neither hay fever nor asthma comprise less than 5 percent of the total population
D. The number of people who have both of these ailments is greater than the number of people who have only one of
them
E. The percentage of people suffering from hay fever who also have asthma is lower than 95 percent.

Brent

This is a pretty "mathy" CR question. These are often pretty tricky.

PREMISES:
- 95% of asthma sufferers have hay fever.
- Hay fever is MORE COMMON than asthma

Let's say that we have a population of 1000 people and 100 people have asthma. The first premise tells us that 95 people
suffer from BOTH asthma AND hay fever.
Since hay fever is more common, the number of hay fever sufferers must be GREATER than 100.
So, there must be 6 or more people who have hay fever but not asthma.

Option A

Brent:
A

B.Asthma is a prerequisite for the development of hay fever.


Since there are people who have hay fever but not asthma, this statement is not supported. ELIMINATE

Option E

Brent

E.The percentage of people suffering from hay fever who also have asthma is lower than 95 percent
In other words, (# of people with BOTH hay fever AND asthma)/(# of people with people with hay fever) < 0.95
This is true.
Check our example, we have (# of people with BOTH hay fever AND asthma)/(# of people with people with hay fever)
= 95/(some number GREATER than 100)
= less than 0.95

----------------

Let's say 100 people suffer from Asthma. 95 of them also have hay fever. Since hay fever is more common, there must
be more than 5% of people who suffer from hay fever only.

For E, if this was true out of 100 people who have hay fever, 95% of those have asthma as well; 95 people have both,
and only 5% would only have asthma. This contradicts our interpretation of the facts. We need MORE than 5% to suffer
from hay fever only.

_________________
Vivian Kerr
GMAT Rockstar, Tutor

OG 2018 | CR 661

Which of the following most logically completes the argument given?

Asthma, a chronic breathing disorder, is significantly more common today among adult competitive swimmers than it is
among competitive athletes who specialize in other sports. Although chlorine is now known to be a lung irritant and
swimming pool water is generally chlorinated, it would be rash to assume that frequent exposure to chlorine is the
explanation of the high incidence of asthma among these swimmers, since __________.

A. young people who have asthma are no more likely to become competitive athletes than are young people who do not
have asthma
B. competitive athletes who specialize in sports other than swimming are rarely exposed to chlorine
C. competitive athletes as a group have a significantly lower incidence of asthma than do people who do not participate
in competitive athletics
D. until a few years ago, physicians routinely recommended competitive swimming to children with asthma, in the belief
that this form of exercise could alleviate asthma symptoms
E. many people have asthma without knowing they have it and thus are not diagnosed with the condition until they begin
engaging in very strenuous activities, such as competitive athletics

Mike McGarry

From the prompt, we get


(a) competitive swimmers have asthma more frequently than do other competitive athletes
(b) chlorine is a lung irritant
The naive conclusion is that all the time in the chlorinated pools is what causes the asthma. The author warns us that this
naive conclusion is rash, and the blank should provide some kind of alternative explanation of why competitive
swimmers have asthma more frequently than do other competitive athletes.

A. young people who have asthma are no more likely to become competitive athletes than are young people who do not
have asthma
Hmm. This is about those with and without asthma going into sports, but it tells us nothing about who goes in
competitive swimming rather than other sports. This is irrelevant.

B. competitive athletes who specialize in sports other than swimming are rarely exposed to chlorine
We are looking for a new explanation, and this does the exact opposite: it strengthens the explanation that the author
called "rash." This is incorrect.

C. competitive athletes as a group have a significantly lower incidence of asthma than do people who do not participate
in competitive athletics
While this may be true, the argument is about comparing competitive swimmers vs. those who do other competitive
athletes. Thus, this statement is irrelevant.

D. until a few years ago, physicians routinely recommended competitive swimming to children with asthma, in the belief
that this form of exercise could alleviate asthma symptoms
Interesting. This would provide a completely different explanation of why so many people with asthma wound up in
competitive swimming. It's not that being in the pool causes the asthma (the naive, rash conclusion); instead, the people
who already had asthma were sent to the pool. This is promising.

E. many people have asthma without knowing they have it and thus are not diagnosed with the condition until they begin
engaging in very strenuous activities, such as competitive
Again, the comparison in the prompt is between competitive swimmers vs. those who do other competitive athletes. If
all the people with "latent asthma" develop full-blown asthma when they start doing strenuous athletic activity, why
would it show up only for swimmers and not for all other kinds of athletes. It's not as if swimming is the only kind of
strenuous activity out there. This is irrelevant.

The only possible answer is (D), the OA.

Does this make sense?


Mike
_________________

Mike McGarry
------------------

HanoiGMATtutor, Manager
We are looking for a statement that if true would negate the argument that exposure to swimming pool water is the
reason swimmers have higher incidence of asthma than other athletes. Well, this argument is a classic correlation-
causation error. If we can point out that the correlation does not imply causation in this case, e.g., by showing that
something else causes the correlation, we will weaken that argument. Answer choice D explains why swimmers have
relatively high incidence of asthma - asthmatic children tended to train to become swimmers because their doctors
advised them to.

GP Exam Pack 1|

A survey of entrepreneurs who started companies last year shows that while virtually all did substantial preparatory
research and planning, only half used that work to produce a formal business plan. Since, on average, the entrepreneurs
without formal plans secured the capital they needed in half the time of those with plans, these survey results indicate
that, in general, formal plans did not help the entrepreneurs who produced them to secure the capital they needed.

Which of the following, if true, most seriously weakens the argument?


A - Companies started by entrepreneurs who had used formal business plans to attract investment were on the whole as
profitable in their first year as were companies started by entrepreneurs who had not produced such plans.

B - In surveys of entrepreneurs who have attempted without success to raise sufficient capital, more than half of the
respondents indicate that they have produced a formal business plan.

C - Among the entrepreneurs surveyed, those who did not produce formal business plans sought and received a much
larger proportion of their capital from investors with whom they had a long-standing business relationship.

D - The entrepreneurs surveyed who did not produce a formal business plan spent nearly as much time doing
preparatory research and planning as the entrepreneurs who produced plans.

E - The entrepreneurs who produced business plans generally reported later that the process of writing the plan had
increased their confidence that their company would succeed. ---

Rich @ empower gmat

The thing to know about Strengthen and Weaken questions is that there are usually several ways to either Strengthen or
Weaken the prompt, depending on the information you've been given and the logic that the prompt is based on.

Here, we're told about two groups who both did research:

Those WITH a formal Business plan


Those WITHOUT a formal Business plan

We're then told that those WITHOUT a plan received capital in half the time of those WITH a plan. The author then
makes a HUGE leap in logic to tell us that those WITH plans did not receive the capital that they needed.

Since the author pointed out JUST ONE difference between the two groups (plan vs no plan), the author assumes that
EVERYTHING ELSE IS EQUAL. One of the ways to weaken this prompt is to find an answer that states that
everything else was NOT equal.
Answer C describes a major difference between the two groups that affected how quickly those WITHOUT plans were
able to get their money. The other 4 answers bring up describe things that have little-to-no bearing on the prompt.

GMAT assassins aren't born, they're made,


Rich

-------------------------

faraz_jeddahMaster | Next Rank: 500

The argument states that there are two types of researchers


1 - Do the research and do not make a plan
2 - Do the research and do make a plan

Group 1 secures the funds in half the time. Then the Conclusion states - Plans did not help GROUP 2.
To weaken the conclusion we should find a premise that shows the plans did help group 2.

Answer choice C talks about Group 1.

-------------------------

Ceilidh Erickson
Manhattan Prep GMAT & GRE instructor

The right answer here hinges very much on the specific language used in the conclusion: "formal plans did not
HELP the entrepreneurs who produced them to secure the capital they needed."

The evidence that we're given to support this conclusion is that entrepreneurs without plans secured the capital in half
the time. Just because those with formal plans took longer to secure capital, does that mean that the plans didn't help?

Often, CR arguments contain logical flaws by conflating two different metrics: time = help. So we need to ask ourselves
- is there a reason that the plans might have helped to secure capital, even though they took longer? As others have
pointed out, the author is making the erroneous assumption that these two groups of entrepreneurs are otherwise equal.
We want an answer choice that shows a difference between these two groups, and indicates that a formal plan might be
helpful for those who used it, but not necessary for those who didn't.

Quote:
To weaken the conclusion we should find a premise that shows the plans did help group 2.

You're right - that's one thing that would have weakened the conclusion. But it might also be enough to indicate why
Group 1 didn't need formal plans: they already had personal connections. We can infer from this that Group 2 likely
didn't have these connections, and thus might need the formal plans that Group 1 didn't need.

---------------------

A - Companies started by entrepreneurs who had used formal business plans to attract investment were on the whole as
profitable in their first year as were companies started by entrepreneurs who had not produced such plans.
Carcass: Out of context as we are talking of securing the capital and not profitability. Even if it were within the context,
it may just be strengthening rather than weakening the arguement as the profitability in both cases is comparable
B - In surveys of entrepreneurs who have attempted without success to raise sufficient capital, more than half of the
respondents indicate that they have produced a formal business plan.
Carcass: It rather strengthens the argument , in which it says that there is an equal chance of failing even after making
formal plan.

C - Among the entrepreneurs surveyed, those who did not produce formal business plans sought and received a much
larger proportion of their capital from investors with whom they had a long-standing business relationship.
Carcass: CORRECT... this proves that entrepreneurs who have been able to raise funds without plan were able to make
up the capital due to their relationship with investors..

D - The entrepreneurs surveyed who did not produce a formal business plan spent nearly as much time doing
preparatory research and planning as the entrepreneurs who produced plans.
Carcass: Out of context .. we are not talking of time

E - The entrepreneurs who produced business plans generally reported later that the process of writing the plan had
increased their confidence that their company would succeed.
Carcass: Again out of context

Veritas prep:

Environmentalist: It takes less energy to make molten glass from recycled glass than from raw materials. Once the
recycled glass or raw materials have been turned into molten glass, making bottles from recycled glass follows the same
process as making bottles from raw materials. Obviously, soft drink bottlers who make a large percentage of their bottles
from recycled glass have significant
energy savings. Therefore, by using recycled glass instead of glass made from raw materials, bottlers can lower their
costs and benefit the environment at the same time.

Which one of the following is an assumption on which the argument relies?


(A) The process of making bottles from plastic that has been recycled is not significantly more energy efficient than is
the process of making bottles from glass that has been recycled.
(B) The amount of glass that is currently recycled each year is enough to supply the major soft drink bottlers with
materials for a large
percentage of the glass bottles they make that year.
(C) Most consumers are not able to distinguish bottles made from recycled glass from glass bottles made from raw
materials.
(D) Purchasing and transport costs are not so much greater for recycled glass than for raw materials that they outweigh
the savings in energy costs resulting from the use of recycled glass.
(E) The process of making molten glass from recycled glass requires fewer steps than does the process of making molten
glass from raw materials.

Excellent solution: Brain’s video in youtube.

Veritas Prep

Criminals released from prison on parole have generally been put under routine supervision. A recent program has
allowed criminals to leave prison early under intensive supervision; they must obey curfews and in some cases they must
be electronically monitored. The percentage of released criminals arrested while under supervision is the same for
intensive supervision as for routine supervision, so intensive supervision is no more effective than routine supervision in
preventing criminals from committing additional crimes.

Which one of the following is an assumption on which the argument relies?

(A) The criminals under intensive supervision, but not those under routine supervision, were required to work or attend
school during their supervision period.

(B) All of the criminals who were arrested while under routine supervision had been in prison more than once before
being paroled and put under supervision.

(C) The proportion of arrests to crimes committed was not significantly higher for criminals under intensive supervision
than those under routine supervision.

(D) Of the criminals arrested while under intensive supervision, some would not have committed crimes if they had been
under routine supervision.

(E) The number of criminals put under routine supervision was not significantly greater than the number of criminals put
under intensive supervision.

Nice explanation by Brain

GMAT Critical Reasoning Tip: Identifying Subtle Gaps In Logic

https://www.youtube.com/watch?v=p0w9ulQx7bs

-----------------
Karishma:

Premises:
Criminals released on parole are put under routine supervision.
A recent program allows criminals to leave prison early under intensive supervision;
The percentage of re-arrests is same for intensive supervision as well as for routine supervision.

Conclusion:
intensive supervision is no more effective than routine supervision in preventing criminals from committing additional
crimes

The key word is highlighted. We are saying that since both types of supervision result in the same percentage of re-
arrests, it means both have the same success rate in preventing crimes. There is a gap in this logic. Based on the number
of re-arrests (which is same for both), we are assuming the number of crimes committed are the same under both types
of supervision. What if, number of crimes committed is much higher under routine supervision but re-arrests are rarer?
Then can we say that both methods are equally effective in preventing crimes? No. So option (C) is an assumption. It
says that the proportion of arrests to crimes committed was similar in both.

On the other hand, (E) doesn't affect our argument. The number put under each is of no consequence to us. We are
comparing percentages. Since their percentages are similar, we can certainly compare their effectiveness even if the
number of criminals is different.

Answer (C)
_________________

Karishma

---------------

We are given that the percentage of re-arrests is the same. This means that if 100 people each were released under the
two systems, in both cases there were say, 10 re-arrests. (So 10% re-arrests in both cases) - This is given.
But to arrive at the conclusion, we are assuming that the percentage of crimes committed was the same in both cases. So
if 100 people were released under each program, we need to assume that the same number committed crimes in both
cases to be able to say that "intensive supervision is no more effective than routine supervision in preventing criminals
from committing additional crimes".

Point is that the argument gives the link between re-arrests and criminals released. But it draws conclusion of the link
between re-arrests and the actual number of crimes. The argument tells us nothing about the actual number of crimes.
The actual number of crimes could have been very different in the two cases. Perhaps 30 of the people released under
routine supervision committed crimes again but only 10 got re-arrested... On the other hand, only 15 of the people
released under intense supervision committed crimes again and 10 got re-arrested. We are assuming that this is not the
case to conclude that "intensive supervision is not more effective".
_________________

Karishma
Veritas Pre
Option D

Sayaltanc2k

The negative statement of option D implies that some people would have committed crime irrespective of whether they
were under routine supervision or intensive supervision. This implies that the intensive supervision was NOT more
effective for "some " people - this statement is the same as the conclusion, whereas negating D should have broken
down the argument, if D were an assumption. (If D were an assumption, negation of D would prove that Intensive
supervision IS more effective)

------------------------

Mitch Hunt:

This CR exhibits a LANGUAGE SHIFT:


Whereas the premise is about X, the conclusion is about Y.

Premise: The percentage of released criminals ARRESTED while under supervision is the same for intensive supervision
as for routine supervision.
Conclusion: Intensive supervision is no more effective than routine supervision in preventing criminals from committing
additional CRIMES.
Whereas the premise is about BEING ARRESTED, the conclusion is about COMMITTING CRIMES.

The assumption that there is a link between the number of arrests and the number of crimes.
Since the assumption is WHAT MUST BE TRUE, apply the NEGATION TEST.
When the correct answer choice is negated, the conclusion will be invalidated.
C, negated:
The proportion of arrests to crimes committed was significantly higher for criminals under intensive supervision than
for those under routine supervision.
If criminals under intensive supervision were arrested 1 time for every 2 crimes committed (a ratio of 1/2), while
criminals under routine supervision were arrested 1 time for every 10 crimes committed (a rate of 1/10), the criminals
under intensive supervision committed FEWER CRIMES for each arrest, invalidating the conclusion that intensive
supervision is no more effective than routine supervision in preventing criminals from committing additional CRIMES.
Since the negation of C invalidates the conclusion, C is the assumption: WHAT MUST BE TRUE for the conclusion to
be valid.

The correct answer is C.

----------------------

David@VeritasPrep

You have found a great CR question from the Veritas Prep Advanced Verbal Lesson.

I understand why you may have some difficulty with this one as it is a difficult question.

However, it is also a question that follows a certain form that is quite common to the GMAT. That form is the "switch in
terms."
You can notice the switch in terms if you focus closely on the evidence and on the conclusion. The evidence is about the
percentage of criminals on parole who are "arrested" - that percentage is about the same for intensive and for routine
supervision. Yet the conclusion switches terms to talk about "crimes committed" and claims that both types are equally
good for preventing crimes from being committed.

When you spot this switch you most likely have either an assumption question or a weaken question. If it is an
assumption question like this one you want to say that the two things are equally, thereby saving the argument. So you
say that evidence about arrests is good for proving a conclusion about crimes committed. In other words you want
arrests to translate to crimes committed. Choice C guarantees that translation.

If this is a weaken question then you will simply want to state that arrests are different from crimes committed. That is
enough to weaken this argument.

As a bonus I want to tell you about the statistics on this question. From what I have observed from hundreds of students
choice E is just about as popular as the correct answer C. Of course there is one word that makes E not the correct
answer for this question. That word is "percentage" as in "The percentage of released criminals arrested while under
supervision is the same..." percentages are already comparable and do not need to be adjusted or weighted. If the
evidence had said "the NUMBER of criminals arrested" then you would want to go for choice E as we would a statistics
problem.

_________________
Veritas Prep | GMAT Instructor

Veritas Prep

Citizen: Each year since 1970, a new record has been set for the number of murders committed in this city. This fact
points to the decreasing ability of our law enforcement system to prevent violent crime.

City Official: You overlook the fact that the city's population has risen steadily since 1970. In fact, the number of
murder victims per 100 people has actually fallen slightly in the city since 1970.

Which one of the following, if true, would most strongly counter the city official's response?

A. The incidence of fraud has greatly increased int he city since 1970.
B. The rate of murders in the city since 1970 decreased according to the age group of the victim, decreasing more for
younger victims.
C. Murders and other violent crimes are more likely to be reported now than they were in 1970.
D. The number of law enforcement officials in the city has increased at a rate judged by city law enforcement experts to
be sufficient to serve the city's increased population.
E. If the health care received by assault victims last year had been of the same quality as it was in 1970, the murder rate
in the city last year would have turned out to be several times what it actually was.
Excellent solution by Brian

GMAT Prep Tip - Critical Reasoning

https://www.youtube.com/watch?v=o67_UX4-TRw

Veritasprep / LSAT

About two million years ago, lava dammed up a river in western Asia and caused a small lake to form. The lake existed
for about half a million years. Bones of an early human ancestor were recently found in the ancient lake-bottom
sediments on top of the layer of lava. Therefore, ancestors of modern humans lived in Western Asia between 2 million
and 1.5 million years ago.

Which one of the following is an assumption required by the argument?

(A) There were not other lakes in the immediate area before the lava dammed up the river.

(B) The lake contained fish that the human ancestors could have used for food.

(C) The lava under the lake-bottom sediments did not contain any human fossil remains.

(D) The lake was deep enough that a person could drown in it.

(E) The bones were already in the sediments by the time the lake disappeared.

Veritas Blueprint for Critical Reasoning

https://www.youtube.com/watch?v=wsMFmnV0NBY

Video Time for this questions: 14 to 17 mins

Manhattan Lsat forum (Official LSAT PrepTest - February 1997 Answers & Explanations - LR)

ohthatpatrick

Essentially, 2 million years ago a lake formed. The lake existed for about half a million years. So the lake stopped
existing 1.5 million years ago. We found some human bones at the bottom of the dried up lake.

Thus, the author concludes, humans were living near that lake during the timeframe the lake existed, between 1.5 - 2
million years ago.

Do you see any gaps there?

When arguments deal with fossil evidence / archeological findings / old specimens, etc. there's almost always some
assumption of "where we found it = where it came from".

Here the author is assuming that since we found these human bones at the bottom of a lake that existed 1.5 - 2 mya, then
humans existed near this lake 1.5 - 2 mya.

Let's look at answer choices and just keep judging their relevance to the premise/conclusion.

(A) This is extreme, which is dangerous for Necessary Assumption. Does the author need to assume there were NO
other lakes in the same area? If there WERE other lakes in that area would it hurt his conclusion? Nope.

(B) This would somewhat strengthen the author's hypothesis that humans lived near this lake when it existed. But does
the author HAVE to assume the lake contained fish? If the lake did NOT contain fish, would that hurt the author's
argument? No, he was never saying that the lake provided sustenance. He's only saying that since we found human
bones in the lake, we know humans lived in the area when the lake existed.

(C) This deals with whether there are any human bones in the lava. Does the author need to assume anything about what
was or wasn't in the lava? Not really. If we negate this answer, it's saying that there WERE human bones in the lava.
That would just suggest that humans were living in this area 2 million years ago when the lava came. That doesn't hurt
the conclusion. If anything, it may help it.

(D) Do the author have to assume that the human bones found in the lake were a result of drowning? No. Cause of death
is out of scope. If the lake were too shallow to drown in, that wouldn't kill the author's conclusion.

(E) Does the author have to assume that the bones were in the lake prior to the lake drying up? When did the lake dry
up? 1.5 million years ago. The author thinks that finding human bones in the lake is evidence that humans lived around
there between 1.5-2 mya. So, yes, the author is assuming that these human bones were in the lake during that period.

If we negate (E), it's saying that the bones weren't at the bottom of the lake until AFTER the lake dried up. That means
that the bones weren't in the lake during the 1.5 - 2 mya time period of the conclusion. So negating (E) would severely
hurt the argument, telling us that this is the correct answer.

To simplify this argument in retrospect, we're getting this:


Prem: human bones found at the bottom of a lake that existed between 1.5 - 2 mya
Conc: humans lived around the lake during that time period.

Assumption: the bones existed during that time period.

-----------

hazelnut, VP GC

General Description: This question asks you to find the assumption required by the argument. In other words, find the
statement whose truth is required if the argument is to succeed in demonstrating its conclusion.

A. Incorrect. The existence of other lakes in the area is irrelevant to the argument.

B. Incorrect. If response (B) turned out to be true, that might provide a reason why humans were in the area of the lake,
but this particular explanation need not be assumed in order for the argument to succeed in demonstrating its conclusion.
C. Incorrect. It does not matter for the argument whether or not there were such remains in the lava, and the argument
does not state or imply that there were no humans in the region prior to two million years ago. This was by far the most
popular incorrect response.

D. Incorrect. The remains could have gotten into the lake in any number of other ways; to give just one, perhaps the
people in the area put their dead into the lake.

E. Correct. If the bones were not already in the sediments when the lake dried up, that means that they got into the
sediments later; that is, less than one-and-a-half million years ago. But then their existence would not provide evidence
that there were human ancestors in western Asia between two million and one-and-a-half million years ago; that is, the
conclusion of the argument would not follow if (E) is false.

Difficulty Level: Medium difficulty

Tips and Pitfalls: Another way to think about the question of whether an assumption is required by an argument is to
think about what happens to the argument if the assumption turns out to be false. If the argument cannot possibly
succeed when the assumption is false, then the assumption is required by the argument. Also, when asked for a
necessary assumption, be careful not to pick an assumption that is stronger or broader than what is strictly necessary for
the argument to succeed, even if making that assumption might strengthen the argument.

--------------------

Brian Veritas prep

Now, as you think back to the stimulus, ask yourself to paraphrase the conclusion. Do you come up with
“the earliest modern humans lived in Western Asia between two million and 1.5 million years ago?” Many will, but in
doing so you’re inferring the superlative term “earliest” or “first” — the actual conclusion only states that human
ancestors lived in that period at that time; we don’t need them to be the first, only to have existed.

If you inferred that “first” superlative in there, you’re apt to select answer choice C — far and away the most popular
incorrect choice. After all, if there were human fossils beneath the lake-bottom sediments, that would mean that the
fossils in those sediments are not evidence of the earliest humans. But, because we’re not tasked with proving that those
humans were the first — only that they existed — it doesn’t matter whether there were people there before them, and C
is therefore incorrect.

Choice E is correct — if the bones were not in the lake sediments when the lake disappeared (1.5 million years ago, the
end of our time period in which we need to prove that humans lived), our argument doesn’t hold. However, people often
prefer choice C because: 1) it comes before E, and has already captured their fancy; and 2) it speaks to our innate
predisposition toward “-st” words.
Know this about the GMAT — it will take advantage of your “delusions of grandeur” as you attempt to go “from better
to best.” Make sure that you make decisions based explicitly on what the test says, and be aware of your tendencies to
want to go farther.

Veritasprep |

Opponents of peat harvesting in this country argue that it would alter the ecological balance of our peat-rich wetlands
and that, as a direct consequence of this, much of the country’s water supply would be threatened with contamination.
But this cannot be true, for in Ireland, where peat has been harvested for centuries, the water supply is not contaminated.
We can safely proceed with the harvesting of peat.

Which one of the following, if true, most strengthens the argument?

(A) Over hundreds of years, the ecological balance of all areas changes slowly but significantly, sometimes to the
advantage of certain flora and fauna.
(B) The original ecology of the peat-harvesting areas of Ireland was virtually identical to that of the undisturbed
wetlands of this country.
(C) The activities of the other industries in coming years are likely to have adverse effects on the water supply of this
country.
(D) The peat resources of this country are far larger than those of some countries that successfully harvest peat.
(E) The peat-harvesting industry of Ireland has been able to supply most of that country’s fuel for generations.

Source: LSAT PrepTest 30 Q#3

hazelnut

VP

The author’s conclusion is that we can safely proceed with the harvesting of peat. How do we know that, contrary to
opponents’ claims, such harvesting will not threaten our water supply? Easy, according to the author: because in Ireland
they have been harvesting peat for centuries and the water supply is not contaminated. Do you see what’s going on here?
It’s simple: The author is reasoning by analogy—harvesting peat in this country will be safe because harvesting peat in
Ireland is safe. Once you identified that the author argues by analogy, you should have been able to predict that the
correct answer would tell you something about how Ireland and this country are similar. And that’s exactly what (B)
does, telling you that the ecology of peat harvesting areas in Ireland is identical to the ecology of wetlands in this
country.

(A) tells us that environments change, but that tells us nothing about the similarity between Ireland and this country
that’s necessary for this argument by analogy to work.

(C) is outside the scope because we aren’t concerned with the effect of other industries on the water supply. We only
want to know what effect peat harvesting will have.

(D) is irrelevant, since it doesn’t compare this country to Ireland, which is after all the relevant “other country.” If any-
thing, (D) might be a weakener, since it points out a potentially relevant difference between this country and other
countries. In any case, (D) is no strengthener.

(E) is irrelevant. We’re not concerned with peat harvesting as a source of fuel; we want to know about its effects on the
water supply.

• In arguments by analogy, the classic way to strengthen the argument is by showing how the two things that are
being compared are similar to each other; the classic way to weaken these arguments is by showing how the two
entities are different from each other.
_________________

Manhattan forum lsat:


The topic is peat-harvesting, and the author disagrees with opponent and further concludes that "We can safely proceed
with the harvesting of peat". Here is how I processed the argument:
The opponent presents a disadvantage to peat harvesting; "alter the ecological balance of our peat-rich wetlands and that,
as a direct consequence of this, much of the country's water supply would be threatened by contamination". The way the
author responds is not by putting forward an advantage that may out weigh a disadvantage, but rather uses Ireland as an
example of a place "where peat has been harvested for centuries, the water supply is not contaminated". By using Ireland
as a example, the author assumes that the features or characteristics of Ireland must have something in common that is at
least relevant to the country talked about in the stimulus. Having said that, the correct answer choice brings forward a
relevant feature of both country x, and Ireland. In which case, it strengthens the argument by pointing out that in Ireland,
the original ecology of the peat-harvesting areas are virtually identical to that of the up disturbed wetlands of this
country (feature or characteristic) And so now knowing this piece of information, having been told in the stimulus that in
Ireland, where peat is harvested, the water supply is not contaminated we have strengthened the argument by bringing
forward an assumption the author makes in using Ireland as an example, and a reason why the opponent's claimed
consequence will not occur based on the idea that, since the ecology of the original peat-harvesting areas in Ireland are
the same as the current ecology in Country x, and since no water contamination occurred in Ireland after peat-harvesting,
the same condition should occur in Country x (that being, no water contamination after peat-harvesting).

Veritas prep

Acid rain, caused by high levels of sulfur dioxide and mercury trapped in global air currents, affects one third of China’s
territory.Coal-fired power plants are notorious for emitting large quantities of these pollutants.To combat the problem,
the Chinese national government has set aggressive environmental goals for the next five years.These goals include a
20-percent improvement in energy efficiency and a pledge that, by the end of the five years, 10 percent of the nation’s
energy will come from renewable resources such as hydroelectric or wind power.

Which of the following, if true, is the best criticism of the Chinese government’s strategy as a method for achieving a
reduction in acid rain?
(A) Some forms of air pollution, such as heavy particulate fumes, would not be affected by the suggested energy
improvements.
(B) Once the changes have been implemented, the actual reduction in acid rain would vary from region to region.
(C) The goals would be forced on every region in China, including those that have no problems with acid rain.
(D) Acid rain is also caused by other factors, such as volcanic eruptions or pollution from neighboring countries, over
which China has no control.
(E) Regional Chinese officials tend to ignore environmental regulations in order to meet aggressive economic
requirements imposed on their regions’ industries.

ab2014

Intern

Here the argument is:

Problem: Coal-fired power plants emit large quantities of sulfur dioxide and mercury -> leads to acid rain -> affect
1/3rd of China's territory
Government's strategy to reduce acid rain: aggressive goals set for next 5 years that focus on improving energy
efficiency and increase in usage of renewable resources -> reduce emission of Sulphur dioxide & mercury pollutants

Question stem wants us to identify something that can weaken our belief in the success of Chinese government’s
strategy

Let us try to pre-think the potential reasons that can lead to a failure:
1. The goal to improve energy efficiency leads to adoption of technologies/anything else which increase Sulphur dioxide
and mercury pollutants.
We know that there are 2 goals that are laid out- (i) improve energy efficiency, (ii) increase usage of renewable
resources. What if the first goal leads to adoption of some technology that increases the emission of Sulphur dioxide and
mercury pollutants?

2. The strategy execution fails due to certain adverse factors:


The government has set targets. Anything that shows that these targets might not be achieved shall reduce our belief in
the success of the strategy.

Any option choice that shows that one of the above can happen shall be a potential weakener.

Let us evaluate the option choices:


A. Some forms of air pollution, such as heavy particulate fumes, would not be affected by the suggested energy
improvements.
We are only concerned about those pollutants that lead to acid rain, not other air pollutants. The government is currently
focusing on reduction of Sulphur dioxide and mercury pollutants to combat acid rain. Therefore, "some pollutant" are
irrelevant here.

B. Once the changes have been implemented, the actual reduction in acid rain would vary from region to region.
The goal is to combat acid rain that affects 1/3rd of China's territory. So as long as there is a noticeable reduction, it does
not matter if there is a variation in this reduction across regions. Therefore, the option does not suggest that the goal will
not be achieved.

C. The goals would be forced on every region in China, including those that have no problems with acid rain.
The option does not tell anything that undermines the success in achieving the goals by those regions that are
affected by acid rain. It does not really matter if the goals are forced on other regions.

D. Acid rain is also caused by other factors, such as volcanic eruptions or pollution from neighbouring countries, over
which China has no control.
We already know that acid rains that are caused by sulphur dioxide & mercury emissions affect China's 1/3rd
territory. The government is trying to reduce acid rain by reducing the emission of these particular pollutants. Therefore,
other factors that also cause acid rain hardly matter. Therefore, the option does not suggest that the goal will not be
achieved.

]E. Regional Chinese officials tend to ignore environmental regulations in order to meet aggressive economic
requirements imposed on their regions’ industries.
This is in-line with the 2nd statement in pre-thinking. It says that the Chinese officials tend to ignore the environment
regulations since they are more concerned with economic requirements in their territory. This creates a doubt that the
officials might rather ignore the government's plan as well in order to ensure that they meet their economic
requirements.
----------------

adi_800Legendary Member, BTG

(A) Some forms of air pollution, such as heavy particulate fumes, would not be
affected by the suggested energy improvements. - We do not have any idea about those air pollutants and we don't know
whether they are causing any pollution. There is no information in the passage.
(B) Once the changes have been implemented, the actual reduction in acid rain
would vary from region to region. - We don't need anything after implementation
(C) The goals would be forced on every region in China, including those that have
no problems with acid rain. - This is just a statement and we know that this is going to happen
(D) Acid rain is also caused by other factors, such as volcanic eruptions or pollution
from neighboring countries, over which China has no control. - This is good contender. However, if this is true then
China whould have known this and it will not implement any regulations for the thing, which is not in their hand. I think
this is an additional information which we don't need
(E) Regional Chinese officials tend to ignore environmental regulations in order to
meet aggressive economic requirements imposed on their regions’ industries. - IMO This is correct. Regional Chinese
officials are ignoring the regulations on the regions’ industries. So these regions’ industries will keep on producing
pollutants and it will harm China's plan

GP Exam Pack 1

Charcoal from a hearth site in Colorado, 2,000 miles south of Alaska, is known to be 11,200 years old. Researchers
reasoned that, since glaciers prevented human migration south from the Alaska-Siberia land bridge between 18,000 and
11,000 years ago, humans must have come to the Americas more than 18,000 years ago.

Which of the following pieces of new evidence would cast doubt on the conclusion drawn above?

A.Using new radiocarbon dating techniques, it was determined that the charcoal from the Colorado site was at least
11,400 years old.

B.Another campsite was found in New Mexico with remains dated at 16,000 years old.

C.A computer simulation of glacial activity showed that it would already have been impossible for humans to travel
south overland from Alaska 18,500 years ago.

D.Using new radiocarbon dating techniques, it was proved that an ice-free corridor allowed passage south from the
Alaska-Siberia land bridge at least 11,400 years ago.

E.Studies of various other hunting-gathering populations showed convincingly that, once the glaciers allowed passage,
humans could have migrated from Alaska to Colorado in about 20 years.

Posted on gmatclub 09 Nov 2004

Hearth : The noun hearth refers to a fireplace and the area around it.
Charcoal: a porous black solid, consisting of an amorphous form of carbon, obtained as a residue when wood, bone, or
other organic matter is heated in the absence of air.

Conclusion: humans must have come to the Americas more than 18,000 years ago.

Reasoning: A blockade prevented humans traveling south between 18,000 to 11,000 years ago. If some information
showed that the blockade was passable, then the new evidence would cast doubt on the conclusion. Correct answer
weakens the conclusion; incorrect answers are neutral to the conclusion or strengthen the conclusion.

------------------

semwal, Senior Manager

PREMISE 1. Charcoal from a hearth site in Colorado, 2,000 miles south of Alaska, is known to be 11,200 years old.
PREMISE 2. Researchers reason that, glaciers prevented human migration south from the Alaska-Siberia land bridge
between 18,000 and 11,000 years ago.
CONCLUSION- Humans must have come to the Americas more than 18,000 years ago.

ASSUMPTION- CHARCOAL FOUND AT COLORADO COULD NOT HAVE BEEN THAT OF HUMAN WHO
MIGRATED BETWEEN 18-11000 YEARS AGO, FROM ALASKA....BUT OF HUMANS THAT WOULD HAVE
MIGRATED BEFORE THIS BLOCK PERIOD...

WEAKENER---- COULD BE.....1. SOME EVIDENCE THAT HUMANS DID COME SOUTH DURING THIS
BLOCK PERIOD ALSO....
2. NATIVE HUMANS( NOT THE MIGRATED ONES) KNOWN TO HAVE RESIDED ALL THROUGH THIS
PERIOD IN COLORADO..
3. NOT POSSIBLE FOR HUMANS TI COME SOUTH MORE THAN 18000 YEARS BACK...

--------------------

A.Using new radiocarbon dating techniques, it was determined that the charcoal from the Colorado site was at least
11,400 years old.

- Neutral answer. If the charcoal was exactly 11,400 years old, then the conclusion is weakened. Whereas, if the charcoal
was greater than 18,000 years old, then this would strengthen the conclusion. It can go both ways.

semwal STILL FALLS UNDER THE BLOCK PERIOD.....DOES NOT WEAKEN THE ASSUMPTION, ABOVE, IN
ANY CASE

B.Another campsite was found in New Mexico with remains dated at 16,000 years old.

- Neutral. The correct answer must prove that people crossed in between 18,000 and 11,000 years ago. This proves that
someone was there 16,000 years ago; the humans could have crossed 2,000 years prior, around 18,000, or sometime
shortly after 18,000 years ago.

semwal STILL FALLS UNDER THE BLOCK PERIOD.....DOES NOT WEAKEN THE ASSUMPTION, ABOVE, IN
ANY CASE
C.A computer simulation of glacial activity showed that it would already have been impossible for humans to travel
south overland from Alaska 18,500 years ago. Strengthens. If the it were impossible to travel south overland from
18,500 years ago, then the conclusion would be strengthened.

semwal THATS OK BUT THEY COULD HAVE MIGRATED IN THE PERIOD-- 18,500 - 18,000

D.Using new radiocarbon dating techniques, it was proved that an ice-free corridor allowed passage south from the
Alaska-Siberia land bridge at least 11,400 years ago.

Correct. If it were possible to cross at least 11,400 years ago, it provides doubt that the glacier prevented all human
migration between 18,000 and 11,000 years ago.

cansLegendary Member : As, even 11,400 years ago, there was a passage, we can't say humans came 18,000 years
ago. Correct

E.Studies of various other hunting-gathering populations showed convincingly that, once the glaciers allowed passage,
humans could have migrated from Alaska to Colorado in about 20 years.

Out of scope/Neutral. This does nothing the conclusion.

semwal TIME TAKEN TO MIGRATE IS IRRELEVANT

GP QP 1 |

Electric utilities pay less for low-quality coal per ton delivered than for high-quality coal. Yet more low-quality coal
than high-quality coal must be burned to generate the same amount of electricity. Moreover, per ton of coal burned, low-
quality coal generates more ash than does high-quality coal, and the disposal of ash is becoming more and more
expensive.
The considerations above, if true, most strongly support which of the following claims?

A. A coal-burning utility might not be assured of benefiting economically by always adhering to the policy of keeping
its overall coal purchasing costs as low as possible.
B. In those regions where the cost of disposing of coal ash is negligible, it is more expensive for coal-burning utilities to
use high-quality coal than low-quality coal.
C. Transportation costs represent a smaller proportion of the cost per delivered ton for low-quality coal than for high-
quality coal.
D. It is no less expensive to dispose of a ton of coal ash that results from the burning of high-quality coal than it is to
dispose of a ton of coal ash that results from the burning of low-quality coal.
E. In regions where coal-ash disposal is the least expensive, reserves of low-quality coal are likely to decline at a faster
rate than are reserves of high-quality coal.

Break down of argument

Ceilidh Erickson :

When you're answering a DRAW CONCLUSION / INFERENCE question, the most important thing to ask yourself is
WHAT MUST BE TRUE?
In this argument, we're given:
LQC costs less per ton than HQC... but we don't know how much less
more LQC must be burned than HQC... but we don't know how much more
LQC generates more ash (which costs $) than HQC... but you guessed it, we don't know how much more

Can we say for certain that HQC is a better investment? Or that LQC is? No, because we don't have any information
about RELATIVE difference in costs or amounts.

When you're going through the answer choices, eliminate anything that MIGHT NOT be true!

--------

Rich.C@EMPOWERgmat.comThis is an example of an Inference question; these questions can sometimes be


difficult, but this one is fairly straight-forward if you take good notes.

The facts:
-Low quality coal costs less per ton than high quality coal
-More low quality coal is needed (than high quality coal) to generate the same amount of electricity
-Low quality coal generates more ash than high quality coal
-Disposal of ash is becoming more and more expensive

We need an answer that logically follows from the above list (the correct answer might be based on any combination of
the above facts). Before looking at the answers, the facts that stand out to me are the last two (low quality coal creates
more ash AND disposal of ash is BECOMING MORE AND MORE EXPENSIVE).

-----------

A. A coal-burning utility might not be assured of benefiting economically by always adhering to the policy of keeping its
overall coal purchasing costs as low as possible.

Ceilidh Erickson :

We LOVE the phrase "might not" on draw conclusion questions! It's really hard to prove that wrong. This says exactly
what we deduced - we don't really know which one is more cost-effective.

Rich.C@EMPOWERgmat Answer A addresses how keeping coal purchasing costs low (re: buying low quality coal)
might not be a benefit economically (because of the coal ash expense and how it's increasing). This is a GREAT
MATCH.

B. In those regions where the cost of disposing of coal ash is negligible, it is more expensive for coal-burning utilities to
use high-quality coal than low-quality coal.

Ceilidh Erickson :

We don't know that it's definitely more expensive, because we don't know about relative amounts of coal needed.

Rich.C@EMPOWERgmat
Answer B addresses specific regions, which the prompt doesn't address.

C. Transportation costs represent a smaller proportion of the cost per delivered ton for low-quality coal than for high-
quality coal.

Ceilidh Erickson :

Does this have to be true? Not necessarily.

Rich.C@EMPOWERgmat

Answer C addresses transportation costs, which the prompt doesn't address.

D. It is no less expensive to dispose of a ton of coal ash that results from the burning of high-quality coal than it is to
dispose of a ton of coal ash that results from the burning of low-quality coal.

Ceilidh Erickson :

We aren't given any information about disposal costs PER TON. All we're told is that LQC produces a greater
AMOUNT of ash. We can't know if this is true.

Rich.C@EMPOWERgmat

Answer D doesn't mesh with the facts. We're NOT told if any particular type of coal ash is more/less/equal expensive to
dispose of; we're only told that low quality coal generates MORE ash.

Galib: As per argument, ‘the disposal of ash is becoming more and more expensive.’ No indication is given regarding
the cost of disposing a ton ash from high quality coal or low quality coal.

E. In regions where coal-ash disposal is the least expensive, reserves of low-quality coal are likely to decline at a faster
rate than are reserves of high-quality coal.

Ceilidh Erickson :

We're not given any information about different behaviors in different regions. This isn't necessarily true.

When you're going through the answer choices, focus on DEALBREAKER LANGUAGE!
Wrong answers will often have definitive/extreme language: it must be, only, cannot, etc
Right answers will usually have softening, qualifying, hedging-our-bets language: might be, some, likely, probably,
perhaps

Rich.C@EMPOWERgmat

Answer E addresses coal reserves, which the prompt doesn't address.

GyanOne: The rate of decline of coal reserves are not based on the costs involved in disposing off ash generated through
burning the coal. We cannot reasonably draw this conclusion.
Galib: There is no strong indication of preference on Low quality coal. We can’t measure the relative ‘least expensive
ash disposal’ and ‘lower cost of low quality coal per ton’

OG16 | CR 100 | new question

A certain cultivated herb is one of a group of closely related plants that thrive in soil with high concentrations of metals
that are toxic to most other plants. Agronomists studying the growth of this herb have discovered that it produces large
amounts of histidine, an amino acid that, in test - tube solutions, renders these metals chemically inert. Hence, the herb's
high histidine production must be the key feature that allows it to grow in metal-rich soils.

In evaluating the argument, it would be most important to determine which of the following?

(A) Whether the herb can thrive in soil that does not have high concentrations of the toxic metals.
(B) Whether others of the closely related group of plants also produce histidine in large quantities.
(C) Whether the herb's high level of histidine production is associated with an unusually low level of production of some
other amino acid
(D) Whether growing the herb in soil with high concentrations of the metals will, over time, reduce their concentrations
in the soil.
(E) whether the concentration of histidine in the growing herb declines as the plant approaches maturity

Mitch hunt:

This is a CAUSAL argument.


In a causal argument, A and B are observed together, and the CR concludes that A CAUSES B.

Premise = A and B are observed together:


A cultivated herb produces histidine and is able to grow in metal-rich soils.
Conclusion = A causes B:
Histidine CAUSES the herb to be able to grow in metal-rich soils.

The correct answer choice must help us determine whether this conclusion is valid.
Rephrase the answer choices as STATEMENTS.
Rephrased as a statement, the correct answer choice will either STRENGTHEN or WEAKEN the link between
HISTIDINE and THE ABILITY TO GROW I METAL-RICH SOILS.
B, rephrased as a statement:
Others of the closely related group of plants also produce histidine in large quantities.
Here, other plants in the closely related group THAT THRIVE IN SOIL WITH HIGH CONCENTRATIONS OF
METALS also produce HISTIDINE, STRENGTHENING the conclusion that histidine CAUSES a plant to be able to
grow in metal-rich soils.

The correct answer is B.

Option A

A: The herb can thrive in soil that does not have high concentrations of the toxic metals.

mitch hunt :
The ability to thrive in NON-METAL soils does not strengthen or weaken the conclusion that histidine enables the herb
to thrive in METAL-RICH soils.
Eliminate A.

Option C
C: The herb’s high level of histidine production is associated with an unusually low level of production of some other
amino acid.

mitch hunt :
The association discussed here does not strengthen or weaken the conclusion that histidine enables the herb to thrive in
metal-rich soils.
Eliminate C.

Karishma : Here, we are trying to connect histidine to "some other amino acid". We don't know if "some other amino
acid" will have any connection with metals. Shouldn't we instead try to connect Histidine to metals directly?

Option D

D: Growing the herb in soil with high concentrations of the metals will, over time, reduce their concentrations in the
soil.

mitch hunt : A subsequent decrease in metal-concentration does does not strengthen or weaken the conclusion that
histidine enables the herb to thrive in METAL-RICH soils -- when the concentration of metal is still HIGH.
Eliminate D.

Option E
E: The concentration of histidine in the growing herb declines as the plant approaches maturity.

mitch hunt :
This option seems to contradict the premise that growth of the herb produces LARGE AMOUNTS OF HISTIDINE.
A premise is a FACT: it cannot be contradicted.
Eliminate E.

Karishma : (E) doesn't help us evaluate the argument. Even if the concentration of histidine decreases after the plants
mature, histidine could certainly be that substance which helps them grow from saplings into mature plants on metal rich
soil. Also, perhaps the plants need less histidine to handle the metals after they mature - we do not know. Knowing
whether the histidine levels decrease as the plants mature cannot help us judge whether histidine helps them survive in
metal rich soils.

David , VP : The gist of the discussion was whether it was acceptable to dismiss E on the grounds that an affirmative
answer would contradict the premise that the herb produces large amounts of histidine. Because the premise never
mentions how much histidine the herb produces/absorbs at various stages of its life cycle, that isn't necessarily the case.
But the broader point is that how the histidine level changes over time isn't relevant. We know the herb produces a large
amount of it. And we want to know if histidine is responsible for the herb's ability to grow in metal-rich soil. E wouldn't
tell us this.

_________________

-----------------------

Argument:

- A certain cultivated herb is one of a group of closely related plants that thrive in soil with high concentrations of metals
that are toxic to most other plants.

So this herb is one of a group of plants. All plants in this group of plants thrive in soil with high concentration of metals.
These metals are toxic to most other plants.

- Agronomists studying the growth of this herb have discovered that it produces large amounts of histidine, an amino
acid that, in test - tube solutions, renders these metals chemically inert.

We are given that this herb produces histidine which makes these metals inactive.

Conclusion: The herb's high histidine production must be the key feature that allows it to grow in metal-rich soils.

We don't really know what makes the herb flourish in high metal soil. It could be histidine or it could be that it produces
10 other chemicals which allow it to flourish in metal rich soil. Since all these plants are closely related and only they
are the ones who thrive in metal rich soils, it is likely that they have some common feature that helps them thrive. We
are guessing its histidine. How do we find out whether our claim holds value?
If we evaluate whether the other plants in the group also produce histidine, it helps us. How? If other plants also produce
lots of histidine, it becomes more likely that histidine is the one which helps these plants thrive. If other plants do not
produce histidine, it becomes likely that they all produce some other chemical which helps them thrive.
Hence (B) helps us evaluate "what is it that allows this herb to grow in metal-rich soils?"

Answer (B)
_________________

Karishma

OG16 | CR 97 | new question

Most of the year, the hermit thrush, a North American songbird, eats a diet consisting mainly of insects, but in autumn,
as the thrushes migrate to their Central and South American wintering grounds, they feed almost exclusively on wild
berries. Wild berries, however, are not as rich in calories as insects, yet thrushes need to consume plenty of calories in
order to complete their migration. One possible explanation is that berries contain other nutrients that thrushes need for
migration and that insects lack.

Which of the following, if true, most seriously calls into question the explanation given for the thrush’s diet during
migration‘?
A. Hermit thrushes, if undernourished, are unable to complete their autumn migration before the onset of winter.

B. Insect species contain certain nutrients that are not found in Wild berries.

C. For songbirds, catching insects requires the expenditure of significantly more calories than eating wild berries does.

D. Along the hermit thrushes’ migration routes, insects are abundant throughout the migration season.

E. There are some species of wild berries that hermit thrushes generally do not eat, even though these berry species are
exceptionally rich in calories.

OG16 | CR 121| new questions

In Colorado subalpine meadows, nonnative dandelions co-occur with a native flower, the larkspur. Bumblebees visit
both species, creating the potential for interactions between the two species with respect to pollination. In a recent study,
researchers selected 16 plots containing both species; all dandelions were removed rom eight plots; the remaining eight
control plots were left undisturbed. The control plots yielded significantly more larkspur seeds than the dandelion-free
plots, leading the researchers to conclude that the presence of dandelions facilitates pollination (and hence seed
production) in the native species by attracting more pollinators to the mixed plots.

Which of the following, if true, most seriously undermines the researchers’ reasoning?

(A) Bumblebees preferentially visit dandelions over larkspurs in mixed plots.

(B) In mixed plots, pollinators can transfer pollen from one species to another to augment seed production.

(C) If left unchecked, nonnative species like dandelions quickly crowd out native species.
(D) Seed germination is a more reliable measure of a species’ fitness than seed production.

(E) Soil disturbances can result in fewer blooms, and hence lower seed production.

JarvisR

Verbal Forum Moderator

Assumption: dandelions absence is the only reason for cause.


Any other factor that can lead to same behavior can be used as a weakener.

------------------------

EMPOWERgmatMax

Type: Weaken
Boil It Down: Dandelions removed, larkspur down -> Dandelions boost pollination of native
Missing Information: Were there other factors?
Goal: Find the option that shows that the difference in pollination between the two groups in the study is not about the
presence of dandelions.

-----------------

Mitch hunt : Premise:


After dandelions were removed from 8 plots, these 8 plots yielded fewer larkspur seeds than did the control plots.
Conclusion:
The presence of dandelions facilitates pollination.

The assumption is that there is NO OTHER EXPLANATION for the lower seed yield in the 8 plots from which
dandelions were removed.
One way to weaken the conclusion is to suggest an ALTERNATE EXPLANATION for the lower seed yield in these 8
plots.

Answer choice E: Soil disturbances can result in fewer blooms, and hence lower seed production.
Implication:
The lower seed yield in the 8 plots was due NOT to the absence of dandelions but to the SOIL DISTURBANCE that
occurred when the dandelions were removed, WEAKENING the conclusion that the presence of dandelions facilitates
pollination.

------------------------------------

(A) Bumblebees preferentially visit dandelions over larkspurs in mixed plots.


mike mcgarry : If all the bees were visiting dandelions, then this would not explain why the larkspurs got so pollinated,
producing a great quantity of seeds. This is simply inconsistent with the evidence. This is not correct.

Empower Max: This option doesn't weaken for two reasons: 1) A preference doesn't necessarily mean that bumblebees
still wouldn't visit both plants 2) The option flat out doesn't weaken the argument. In fact, if anything it appears to have
the flavor of a strengthener since a preference toward dandelions would seem to support the findings of the study that
dandelions are luring bumblebees to native plants.

(B) In mixed plots, pollinators can transfer pollen from one species to another to augment seed production.
mike mcgarry : This is a strengthener. If this is true, then it would explain why having dandelions in the plot would
result in more pollination for the larkspurs. This is a typical GMAT CR trap, having a strengthener for a weakener, or
vice versa. This is not correct.

Empower Max: This option would also appear to strengthen the argument that the presence of dandelions boosts
pollination of native plants since this option would suggest that pollination is enhanced in mixed plots (those that
include dandelions).

Marty Murray : This could possibly support a different conclusion. It may be that the dandelions do not attract additional
pollinators but rather that they enable the transfer of pollen and thus the augmentation of seed production. So this may
be the answer we are looking for.

(C) If left unchecked, nonnative species like dandelions quickly crowd out native species.
mike mcgarry : This is a problems down the road, a reason why in the big picture dandelions might be a problem for
larkspars, but it doesn't do anything to address the issue of which plants get pollinated and how many seeds are
produced. This is not correct.

Empower Max: Overcrowding could very well occur, but that wouldn't weaken the claim that the presence of dandelions
still boosts native plant production. Gone.

Mitch Hunt : To weaken the researchers' conclusion, the correct answer choice must offer an alternate explanation for
the results of the researchers' study.
In the researcher's study, there was no overcrowding.
It is stated as a PREMISE -- as a FACT not in dispute -- that the plot with both dandelions and larkspurs yielded more
larkspur seeds than did the dandelion-free plots.
Since the information in C has no bearing on the results of the researcher's study, eliminate C.

(D) Seed germination is a more reliable measure of a species’ fitness than seed production.
mike mcgarry : Even if this is true, it is not relevant, because in terms of the experiment, only seed production was
measured. There was no measurement of seed germination: at most, it was merely inferred from seed production. The
experiment left no means to distinguish between these two, so the distinction in this context is experimentally
meaningless. This is incorrect.

Empower Max: This option centers on the measurement of species fitness, but species fitness is a completely different
topic. This option is completely Out of Focus.

(E) Soil disturbances can result in fewer blooms, and hence lower seed production.
mike mcgarry : This may appear irrelevant at first glance, but think about it. The control plot (with dandelions) produced
more seeds than the experimental, dandelion-free plot. How was that latter plot prepared? The prompt says: "all
dandelions were removed from eight plots." In other words, all the dandelions were ripped out, disturbing the soil.
According to this answer choice, this soil disturbance would have inhibited seed production in the control group. This
provides an alternative explanation to the experimental results: according to this view, the fact that the control group had
more seeds than the experimental group has nothing to do with the presence or absence of dandelions, but with the
presence or absence of soil disturbance. Providing a cogent alternative explanation shatters the reliability of the
argument. This is a weakener.
Empower Max: Yes! Here we have an option that directly offers an alternative explanation. The difference in pollination
in the study wasn't the presence of dandelions (or the lack of them) but rather the fact that the removal of the dandelions
in the study disrupted blooms, and thus pollination. This option exposes the dark truth that this study is miserably
flawed.

Marty murry : This information supports an alternative conclusion. Given what is said in this answer choice it seems
likely that the reason seed production by native species is higher in the mixed plots is simply that the process of
removing the dandelions from the other plots disturbed the soil, causing a reduction in blooms and in seed production.
Since this information supports a conclusion different from that arrived at by researchers, it undermines their
conclusion.

So B or E could work, and really in a way I am not sure that one is better than the other. The game here is to somehow
get the right answer though, and so we need a way to find it.

In this case, in the prompt is discussed not simply plots with fewer dandelions but plots from which the dandelions were
removed. So I am going to use that as a clue for determining which choice the writer believes to be the best. Removal of
dandelions would disturb the soil. So I am going to go with the answer choice that includes discussion of soil
disturbances.

The use of a hack like this particular one is not generally necessary for correctly answering official questions, but
hacking in general is useful, and so this hack is still illustrative of the types of things one can do to arrive at OA's.

GP2 | 108

From 1980 to 1989, total consumption of fish in the country of Jurania increased by 4.5 percent, and total consumption
of poultry products there increased by 9.0 percent. During the same period, the population of Jurania increased by 6
percent, in part due to immigration to Jurania from other countries in the region.

If the statements above are true, which of the following must also be true on the basis of them?

(A) During the 1980s in Jurania, profits of wholesale distributors of poultry products increased at a greater rate than did
profits of wholesale distributors of fish.

(B) For people who immigrated to Jurania during the 1980s, fish was less likely to be a major part of their diet than was
poultry.

(C) In 1989 Juranians consumed twice as much poultry as fish.

(D) For a significant proportion of Jurania’s population, both fish and poultry products were a regular part of their diet
during the 1980s.

(E) Per capita consumption of fish in Jurania was lower in 1989 than in 1980.
A.During the 1980's in Jurania, profits of wholesale distributors of poultry products increased at a greater rate than did
profits of wholesale distributors of fish.

nishatfarhat87 - The passage talks about percentages and this talks about absolute numbers. Hence, a disconnect. We
only know that the their is more increase in demand from the numbers previously sold, this doesn't mean that the profit
margins per product will increase. Maybe poultry has lesser margin.

Rich@empowergmat : A: This mentions profits, but we have no information on that.

B.For people who immigrated to Jurania during the 1980's, fish was less likely to be a major part of their diet than was
poultry.

nishatfarhat87 - We only have percentages. Again this talks about absolute values. We don't know the base values of
fish or poultry. Maybe fish was anyway sold more then their lesser increase in percentage might also mean greater
increase in numbers.

Bill@Veritas Prep: It is a possible explanation, but when we're looking for a conclusion, we're looking for the one
answer that is guaranteed to be true.

Rich@empowergmat : B: The likely diets of the people who immigrated to the country is also something we have no
information on. Maybe the people who immigrated ate more poultry, but maybe they actually ate more fish and the
locals ate more poultry.

C.In 1989 Juranians consumed twice as much poultry as fish.

nishatfarhat87 - Same as B.

Rich@empowergmat : C: This is a distortion - we don't know how much fish or poultry was consumed, so we can't
say that twice as much poultry was consumed as fish.

D.For a significant proportion of Jurania's population, both fish and poultry products were a regular part of their diet
during the 1980's.

nishatfarhat87 - We can't infer that. Maybe, the previous populations were not eating it regularly and there has been a
sudden increase due to immigrants.

Rich@empowergmat : D: We don't know what percent of the population ate fish and poultry regularly, so we can't say
that it was necessarily a significant portion of the population.

E.Per capita consumption of fish in Jurania was lower in 1989 than in 1980.

nishatfarhat87 - Correct. Can be proven with numbers.

Mitch hunt: The correct answer choice is WHAT MUST BE TRUE.


Answer choice E: Per capita consumption of fish in Jurania was lower in 1989 than in 1980.
Let f = total fish consumption in 1980 and t = total population in 1980.
Per capita consumption of fish means the average amount of fish consumed by each person.
In 1980, per capita consumption of fish = f/t.
In 1989, the total fish consumption increases by 4.5% and the total population increases by 6%.
Per capita consumption of fish in 1989 = (1.045f)/(1.06t).
(1.045/1.06)(f/t) < f/t.
1989 < 1980.
Thus, answer choice E is WHAT MUST BE TRUE.

The correct answer is E.

Rich@empowergmat : E: The per capita consumption of fish was lower in 1989 than in 1980. This we CAN infer
because the population grew at a faster rate than the consumption of fish. It doesn't matter what the starting and ending
numbers are, as long as those percentage increases take place, the RELATIONSHIP between the numbers in 1980 and in
1989 will show a decrease in per capita consumption of fish.

gmatmachoman: per captia fish consumed = Total Fish consumed/ Total population

We shall take few cases to prove E is correct.

In 1980 : Total Fish consumed : 5000 Kgs


Total population : 100 people

per captia fish consumed = 50

In1989 : Total Fish consumed ; 5000 + 4.5 % ( 5000)

= 5022.5
Total Population : 100 + 6% ( 100)
: 106

per captia fish consumed : 5022.5/106


: 47.38

50 > 47.38 : Per capita consumption of fish in Jurania was lower in 1989 than in 1980

SO E does that!!

GP

Which of the following most logically completes the argument ?

The growing popularity of computer-based activities was widely predicted to result in a corresonding decline in
television viewing. Recent studies have found that, in the United States, people who own computers watch, on average,
significantly less television than people who do not own computers. In itself, however, this finding does very little to
show that computer use tends to reduce television viewing time, since_______.

(A) many people who watch little or no television do not own a computer.
(B) even though most computer owners in the United States watch significantly less television than the national average,
some computer owners watch far more television than the national average.
(C) computer owners in the United States predominately belong to a demographic group that have long been known to
spend less time watching television than the population as a whole does.
(D) many computer owners in the United States have enough leisure time that spending significant amounts of time on
the computer still leaves ample time for watching television.
(E) many people use their computers primarily for tasks such as correspondence that can be done more rapidly on the
computer, and doing so leaves more leisure time for watching television.

GP

Many people suffer an allergic reaction to sulfites, including those that are commonly added to wine as preservatives.
However, since there are several winemakers producing wine to which no sulfites are added, those who would like to
drink wine but are allergic to sulfites can drink these wines without risking an allergic reaction to sulfites.

Which of the following, if true, most seriously weakens the argument?

a) Sulfites occur naturally in most wine.


b) The sulfites that can produce an allergic reaction are also commonly found in beverages other than wine.
c) Wines without added sulfites tend to be at least moderately expensive.
d) Apart from sulfites, there are other substances commonly present in wine that can trigger allergic reactions.
e) Wine without added sulfites sometimes becomes undrinkable even before the wine is sold to consumers

Many people suffer an allergic reaction to certain sulfites, including those that are commonly added to wine as
preservatives. However, since there are several wine makers who add sulfites to none of the wines they produce, people
who would like to drink wine but are allergic to sulfites can drink wines produced by these wine makers without risking
an allergic reaction to sulfites.
Which of the following is an assumption on which the argument depends?

A. These wine makers have been able to duplicate the preservative effect produced by adding sulfites by means that do
not involve adding any potentially allergenic substances to their wine.
B. Not all forms of sulfite are equally likely to produce the allergic reactions.
C. Wine is the only beverage to which sulfites are commonly added.
D. Apart from sulfites, there are no substances commonly present in wine that give rise to an allergic reaction.
E. Sulfites are not naturally present in the wines produced by these wine makers in amounts large enough to produce an
allergic reaction in someone who drinks these wines.

Mitch Hunt: It is crucial that we notice CHANGES IN SCOPE.


The conclusion is very limited in its scope: people allergic to sulfites can drink WINES produced by these wine makers
without risking AN ALLERGIC REACTION TO SULFITES.

Any answer choice that broadens the scope to other types of beverages, allergens or allergic reactions can be eliminated.

A broadens the scope to other potential allergens. Eliminate A.


C broadens the scope to other types of beverages. Eliminate C.
D broadens the scope to other types of allergic reactions. Eliminate D.

This CR exhibits a language shift.


The premise is about X: some winemakers do not ADD sulfites.
The conclusion is about Y: people allergic to sulfites can DRINK wines by these winemakers.
The argument assumes that if sulfites are not ADDED, then people allergic to sulfites can DRINK the wines by these
winemakers.

The negation of E trashes this conclusion: Sulfites ARE naturally present in the wines produced by these wine makers in
amounts large enough to produce an allergic reaction in someone
who drinks these wines.
The negation of E trashes the conclusion that people allergic to sulfites will be able to drink the wines. Thus, E is the
necessary assumption: WHAT MUST BE TRUE for the conclusion of the passage to be valid.

The correct answer is E.

----------

David@VeritasPrepThis is an assumption question. So we are looking to protect the evidence that we have, rather than
acquire new evidence. This is an important difference between an assumption and a regular strengthen question.

The evidence that we have is that some wine makers do not add sulfites to their wines. From this evidence the
conclusion is reached that people who are allergic to sulfites can drink these wines without a reaction. So the implication
here is that there are no sulfites in the wine. Now we need to protect the evidence that we have - which is that no sulfites
are added and therefore the wine is sulfite free. What would weaken this? If the wine naturally has sulfites those with
allergies could not drink it at all. So we want to protect against this. Answer choice E does this - it states that sulfites are
not naturally in the wines in large amounts. This preserves the link between our evidence and our conclusion.

None of the other answer choice are absolutely required.

------------
Option A : These wine makers have been able to duplicate the preservative effect produced by adding sulfites by means
that do not involve adding any potentially allergenic substances to their wine.

-------------

Testluv, Kaplan teacher at torrento

We don't care whether or not they're able to duplicate the preservative effect without adding any allergenic substances. I
can understand why this choice is tempting. If they aren't able to duplicate the preservative effect without adding any
allergenic substances, then it suggests that they are adding allergenic substances in which case the argument falls apart.
However, we can't assume they are still interested in duplicating the preservative effect in the first place--that's outside
the scope of the argument. Also, the conclusion is about allergic reactions TO THE SULFITES--thus, allergic reactions
to substances other than sulfites is also outside the scope (that's also why choice D is wrong).

---------------------

The conclusion here is that many people can safely drink wines without the concern of an allergic reaction caused by
sulfites because some wine makers don't put sulfites as preservatives. An assumption would strengthen the conclusion,
hence

A:Possible, but does that stregnthen the conclusion, NO.

Galib: it says ‘ the wine makers successfully abled to duplicate the effect of sulfites.’ But it doesn’t close the gap that
sulfites may naturally present in wine.

Option B: Not all forms of sulfite are equally likely to produce the allergic reactions.

Option C : Wine is the only beverage to which sulfites are commonly added.

Out of scope. we are not concerned about the forms of sulfite that don't cause the reaction, but forms that do.

Option D : Apart from sulfites, there are no substances commonly present in wine that give rise to an allergic reaction.

Mitch hunt: The conclusion here is not about ALL people but only about PEOPLE WHO ARE ALLERGIC TO
SULFITES:
People who would like to drink wine but are allergic to sulfites can drink wines produced by these wine makers without
risking an allergic reaction to sulfites.
Since D discusses allergens APART FROM SULFITES, it has no bearing on this conclusion, which is constrained to
people who are ALLERGIC TO SULFITES.
Eliminate D.

-----------

The problem with D is that, it talks about allergic reactions from substances other than sulfites.

But the conclusion states that "People can drink these wines without Allergic Reaction to sulfites". (The conclusion only
talks about Allergies arising out of sulfites, not from other substances). So even if you negate answer D, Conclusion
holds; so D cannot be the answer.

-------------

Plausible, but here the focus of the conclusion is on safely drinking the wine(which does not contain sulfites) and not
safely drinking the wine(which does not contain other allergy causing substance)

-------------

Out of scope, Argument is concerned with allergy caused by sulfite only.

--------------

Option E : Sulfites are not naturally present in the wines produced by these wine makers in amounts large enough to
produce an allergic reaction in someone who drinks these wines.

This is pertinent. The statement says that good wine makers do not add sulphites. it does not say they remove sulphites.
That would mean that if sulphites are naturally present, they will not be removed and cause allergy.

======================================

Weaken Questions

======================================

Weaken List :

OG 16 : 4, 30, 45, 48, 50, 77, 79, 96, 97, 102, 103, 113, 117, 121, 124

Cause & effect : 4, 30 (reverse cause-effect), 45


OG16 CR 30| new questions

People who do regular volunteer work tend to live longer. on average, than people who do not. It has
been found that ‘doing good.’ a category that certainly includes volunteer work, releases endorphins, the brain's natural
opiates, which induce in people a feeling of well-being. Clearly, there is a connection: Regular releases of endorphins
must in some way help to extend people's lives.
Which of the following, it true most seriously undermine the force of the evidence given as support the hypothesis that
endorphins promote longevity

(A) People who do regular volunteer work are only somewhat more likely than others to characterize the work they do
for as a ‘doing good.’

(B) Although extremely high levels of endorphins could be harmful to health, such levels are never reached as a result of
the natural release of endorphins.

(C) There are many people who have done some volunteer work but who do not do such work regularly.

(D) People tend not to become involved in regular volunteer work unless they are healthy and energetic to begin with.

(E) Releases of endorphins are responsible for the sense of well-being experienced by many long-distance runners while
running.

OE:

Situation People who volunteer regularly live longer on average than people who do not. Doing good work, including
volunteer work, releases endorphins, which induce a feeling of well-being.

Reasoning What additional findings would suggest that the cited evidence does not indicate that endorphins increase
longevity?

The argument implicitly assumes that the reason regular volunteers tend to live longer is that volunteering lengthens
their lives.

It further assumes that no factor that is correlated with volunteering, other than the endorphin release, would plausibly
explain how volunteering could have this effect.

Findings that cast doubt on either of these assumptions would undermine the connection between the cited evidence and
the conclusion that endorphins promote longevity.

DavidG@VeritasPrep

To undermine the force of the evidence is to undermine the logic of how the evidence leads to the conclusion, not to
undermine (or refute) the evidence itself. This is typically what we're doing in weaken questions.

--------------------

EMPOWERgmatVerbal
EMPOWERgmat Instructor

Type: Weaken
Boil It Down: Volunteer -> Endorphins -> Live longer
Missing Information: There are no other factors causing these people to live longer
Goal: Find the option that exposes another reason why these people are living longer other than endorphins

-----------------

OptimusPrepJanielle

People who do volunteering tend to live longer because they feel happy because of the endorphin release.
Conclusion:Volunteering work extend's people's lives.

We need to weaken this conclusion.

1. What if something else increases the lives?


2. What if the persons are already healthy?
In these cases, we cannot conclude that endorphins released as a result of volunteering increase lives

Of the given option, option D resonates with point 2

--------------------

(A) People who do regular volunteer work are only somewhat more likely than others to characterize the work they do
for as a ‘doing good.’

Empowergmat: Ⓐ Whether the people “doing good” are or are not aware that it would be classified as “doing
good” is entirely irrelevant. What is relevant is whether “doing good” is responsible.

OE: Volunteering might greatly boost volunteers’ endorphin levels even if the
work the volunteers do for a living is no different from other people’s work.

(B) Although extremely high levels of endorphins could be harmful to health, such levels are never reached as a result of
the natural release of endorphins.

Empowergmat: Ⓑ Out of Focus. Are EXTREMELY high endorphin levels within the focus of discussion in this
prompt? No. Boot it.

OE : Even if unnaturally high endorphin levels could harm health, the levels
attainable through volunteer work may promote health.

(C) There are many people who have done some volunteer work but who do not do such work regularly.

Empowergmat:Ⓒ The fraction of people who do this kind of work is also outside of the logical focus of this
argument. This argument deals with people who do volunteer, not what fraction of people in general volunteer.

OE : The argument is about an observed correlation in a certain group of


people (those who regularly do volunteer work). How many people are
outside that group (i.e., do not regularly do volunteer work) is independent
of the question of what causes the observed correlation. Even if some people
volunteer only occasionally, volunteering regularly may promote longevity
by causing regular releases of endorphins.

(D) People tend not to become involved in regular volunteer work unless they are healthy and energetic to begin with.

Empowergmat:Ⓓ Yes! This option exposes that the people who volunteer tend to be healthier and energetic to
begin with. Therefore maybe their greater longevity has nothing to do with the volunteerism, but rather they’re
predisposition to longer lives anyhow.

OE : Correct. This suggests that the initially better health of people who
choose to volunteer could fully explain the cited correlation between
volunteering and longevity.

(E) Releases of endorphins are responsible for the sense of well-being experienced by many long-distance runners while
running.

Empowergmat: Ⓔ Are we at all concerned with the sensation that runners feel? Mind-bogglingly out of focus.
This option takes us on a total tangent.

OE : Unless we are also given evidence that long-distance runners tend not to live longer than other people, this does
not undermine the purported evidence in the argument. Endorphins might promote longevity in both regular volunteers
and long-distance runners.

OG16 CR 4

Editorial: The roof of Northtown's municipal equipment-storage building collapsed under the weight of last week's
heavy snowfall. The building was constructed recently and met local building-safety codes in every particular, except
that the nails used for attaching roof supports to the building's columns were of a smaller size than the codes specify for
this purpose. Clearly, this collapse exemplifies how even a single, apparently insignificant departure from safety
standards can have severe consequences.

Which of the following, if true, most seriously weakens the editorial's argument?

(A) The only other buildings to suffer roof collapses from the weight of the snowfall were older buildings constructed
according to less exacting standards than those in the codes.
(B) The amount of snow that accumulated on the roof of the equipment-storage building was greater than the predicted
maximum that was used in drawing up the safety codes.
(C) Because the equipment-storage building was not intended for human occupation, some safety-code provisions that
would have applied to an office building did not apply to it.
(D) The municipality of Northtown itself has the responsibility for ensuring that buildings constructed within its
boundaries meet the provisions of the building-safety codes.
(E) Because the equipment-storage building was used for storing snow-removal equipment, the building was almost
completely empty when the roof collapsed.

EMPOWERgmatVerbal
Type: Weaken
BID: Nail size deviation -> Roof collapse
Missing Information: No other factors
Goal: Find the option that exposes that the collapse was the result of something other than the deviation from code.

----------------------

Mitch Hunt :

Premise: The NAILS were of a SMALLER SIZE than the codes specify.
Conclusion: Even a single, apparently insignificant departure from safety standards can have SEVERE
CONSEQUENCES.

This is a CAUSAL argument.


The passage assumes that the SMALLER SIZE OF THE NAILS had a SEVERE CONSEQUENCE -- namely, the
collapse of the roof.
One way to weaken a causal argument is to suggest an ALTERNATE CAUSE.

-------------------------

OE : Argument Evaluation

Situation The roof of a recently constructed building collapsed under heavy snowfall. The only way the building did not
meet safety standards was that some nails for the roof supports were smaller than prescribed by the building codes.

Reasoning What would make it less likely that the building’s collapse resulted from a single, apparently minor
departure from safety standards? The building met safety standards except for the size of the nails. So if the collapse
exemplifies how a departure from safety standards can have severe consequences, as the conclusion claims, then the size
of the nails had to be responsible for the collapse. Thus, evidence that a factor other than the size of the nails could
fully account for the collapse would weaken the argument.

Empowergmat :

(A) The only other buildings to suffer roof collapses from the weight of the snowfall were older buildings constructed
according to less exacting standards than those in the codes.

Empowergmat : Ⓐ Other, older buildings collapsing is Out of Focus, and if anything seems to reinforce the unusual
nature of this particular collapse in the prompt.

Dmitry : actually strengthens the argument by suggesting that no building that conformed to code collapsed.

OE : This suggests that the snow would not have been heavy enough to collapse the roof if the construction had
completely met the safety standards, so it strengthens, rather than weakens, the argument.

(B) The amount of snow that accumulated on the roof of the equipment-storage building was greater than the predicted
maximum that was used in drawing up the safety codes.
Empowergmat : Ⓑ Yes! Here we go. This option takes the circumstances beyond the code. In other words, with option
B, the collapse may have had nothing to do with the nail size discrepancy

Dmitry : provides this other cause. The snow was too much even for a built-to-code structure to withstand.

OE : This suggests that the snow could have collapsed the roof even if the nails had met the safety standards, thus
casting doubt on the assumption that the nails’ inadequacy was responsible for the collapse.

Mitch Hunt : Answer choice B: The amount of snow that accumulated on the roof of the equipment-storage building was
greater than the predicted maximum that was used in drawing up the safety codes.
Implication:
Even if the nails had been of the appropriate size, the roof still would have collapsed, since THE AMOUNT OF SNOW
WAS GREATER THAN THE PREDICTED MAXIMUM.
By suggesting that an ALTERNATE CAUSE -- the amount of snow greater than the predicted maximum -- caused the
collapse of the roof, B WEAKENS the conclusion that a single departure from safety standards can have severe
consequences.

(C) Because the equipment-storage building was not intended for human occupation, some safety-code provisions that
would have applied to an office building did not apply to it.

Empowergmat : Ⓒ 180 and irrelevant. This option actually expresses that some codes that usually apply to OFFICE
buildings wouldn’t apply to this building.

OE : The claim that the safety requirements for this building were weaker than some others tends slightly to strengthen,
rather than weaken, the hypothesis that the bad consequences resulted partly from a failure to comply. Even if safety-
code provisions for an equipment-storage building differ from those for an office building, they may still be adequate to
ensure the roof’s stability.

Dmitry : hints that perhaps the building wasn't up to the best standard, but this is irrelevant. The conclusion is about the
effect of a departure from the code, and the nails were the only departure. In any case, it seems fair to assume that even
buildings not intended for human occupation are not supposed to collapse!

(D) The municipality of Northtown itself has the responsibility for ensuring that buildings constructed within its
boundaries meet the provisions of the building-safety codes.

Empowergmat : Ⓓ Who’s at fault for the failure of compliance is TOTALLY irrelevant. The argument is claiming that
it was the very failure to comply with the nail size aspect of the code that caused the collapse. Who’s at fault is entirely
Out of Focus from that discussion.

Dmitry : We don't care who is responsible. We are just investigating the physical cause of the collapse.

OE : The question of who was responsible for ensuring compliance with the safety codes is irrelevant to whether a
failure to comply was responsible for the roof’s collapse.
(E) Because the equipment-storage building was used for storing snow-removal equipment, the building was almost
completely empty when the roof collapsed.

Empowergmat : Ⓔ Easy Out #3 This is definitely the worst option here. It says that the building happened to be empty
because it stores snow removal equipment. Neither of those details have any relevance at all to our discussion as to what
caused the collapse. Gone.

Dmitry : I'm glad no one was injured, but what does this have to do with anything?

OE : This suggests that the alleged consequences of failing to meet safety standards were less severe than they could
have been, but it is irrelevant to determining the cause of the collapse.

OG16 CR 45

In the last decade there has been a significant decrease in coffee consumption. During this same time, there has been
increasing publicity about the adverse long-term effects on health of the caffeine in coffee. Therefore, the decrease in
coffee consumption must have been caused by consumers' awareness of the harmful effects of caffeine.

Which of the following, if true, most seriously calls into question the explanation above?

(A) On average, people consume 30 percent less coffee today than they did 10 years ago.

(B) Heavy coffee drinkers may have mild withdrawal symptoms, such as headaches, for a day or so after significantly
decreasing their coffee consumption.

(C) Sales of specialty types of coffee have held steady as sales of regular brands have declined.

(D) The consumption of fruit juices and caffeine-free herbal teas has increased over the past decade.

(E) Coffee prices increased steadily in the past decade because of unusually severe frosts in coffee-growing nations.

David Goldstein, a Veritas Prep GMAT instructor

This one is straightforward enough to diagnose – we actually get the phrase “caused by” in the argument! As an arrow
diagram, it looks like this:

Awareness of harmful effects of caffeine –> decrease in coffee consumption


According to our earlier analysis, this can be weakened in one of two ways. If cause and effect were reversed, the diagram be:

Decrease in coffee consumption –> Awareness of harmful effects of caffeine

Well, that doesn’t make sense. How could a decrease in coffee consumption cause a heightened awareness of the ill effects of
caffeine? So we must be looking for an alternative cause:

Something else –> decrease in coffee consumption.

So that’s what we’re after: that alternative underlying cause.


------------

OE : Situation The decrease in coffee consumption in the last decade can be explained by consumers’ increased
awareness of the detrimental effects of the caffeine in coffee.

Reasoning What point weakens this explanation? A conclusion offering an explanation for some occurrence may be
weakened when another explanation at least as compelling as the original is offered. Coffee consumption may have
decreased over the decade for some reason other than consumers’ awareness of the adverse health effects of
caffeine. If the price of coffee has increased in the same period that consumption has decreased, then the decrease may
well be the result of consumers’ attention to price rather than their attention to health. Higher prices would offer a good
alternative explanation that would weaken the original explanation.

--------------------

(A) On average, people consume 30 percent less coffee today than they did 10 years ago.

David : There’s no different underlying cause here. In fact, this is reiterating the notion that coffee consumption has
decreased. We already knew this. Eliminate A.

OE : This point merely tells us how much coffee consumption has decreased; it does not make the explanation offered in
the conclusion any less likely to be correct.

(B) Heavy coffee drinkers may have mild withdrawal symptoms, such as headaches, for a day or so after significantly
decreasing their coffee consumption.

david : This isn’t an alternative reason for why people are drinking less coffee. In fact, the unpleasant withdrawal symptoms
would be a pretty compelling reason to continue drinking plenty of coffee! Eliminate B.

OE : Withdrawal symptoms would occur only after decreased consumption has occurred and so cannot explain why the
decrease occurred.

(C) Sales of specialty types of coffee have held steady as sales of regular brands have declined.

David : Again, no real alternative cause presented here. And, logically, this doesn’t weaken the argument at all. It’s certainly
possible that while many coffee drinkers have cut back on their coffee consumption, the kind of aficionados who drink
specialty coffee will continue to drink their double latte espressos without reservation. Eliminate C

OE : Suppose that the specialty coffees that had their sales hold steady were all caffeine-free coffees; note that nothing
rules this out. If this were the case, the explanation would remain plausible.

(D) The consumption of fruit juices and caffeine-free herbal teas has increased over the past decade.

David : This one is often tempting. Students sometimes argue that it’s the appeal of fruit juices that is the alternative
underlying cause we’re looking for. The problem is that we’re trying to weaken the argument, and this answer choice really
isn’t incompatible with the conclusion. To see why, imagine that the argument is true: people find out that caffeine is bad for
them, and so drink less coffee. It would be perfectly reasonable for them to then replace that morning coffee with alternatives
like fruit juice and herbal tea. In other words, the increase in the consumption of other beverages wouldn’t be a cause of the
decrease in coffee, but rather, a consequence of that decrease. D is out.
OE : An increase in the consumption of these drinks could plausibly be the result of some coffee drinkers switching to
these drinks to avoid the negative effects of caffeine.

(E) Coffee prices increased steadily in the past decade because of unusually severe frosts in coffee-growing nations.

David : Now we have our alternative cause. Perhaps it’s not the awareness of the ill effects of caffeine that’s caused this drop
in coffee consumption, it’s an increase in price. The new arrow diagram looks like this:

Increase in price –> Decrease in consumption


And this makes perfect sense. E is our answer.

The takeaway: A simple arrow diagram can powerfully simplify the logic of any causality argument.

OE : This statement properly identifies a plausible alternative explanation and therefore undermines the given
explanation.

Question Pack 1 | CR 1

According to the Tristate Transportation Authority, making certain improvements to the main commuter rail line would increase ridership
dramatically. The authority plans to finance these improvements over the course of five years by raising automobile tolls on the two
highway bridges along the route the rail line serves. Although the proposed improvements are indeed needed, the authority's plan for
securing the necessary funds should be rejected because it would unfairly force drivers to absorb the entire cost of something from which
they receive no benefit.

Which of the following, if true, would cast the most doubt on the effectiveness of the authority's plan to finance the proposed improvements
by increasing bridge tolls?

(A) Before the authority increases tolls on any of the area bridges, it is required by law to hold public hearings at which objections to the
proposed increase can be raised.
(B) Whenever bridge tolls are increased, the authority must pay a private contractor to adjust the automated toll-collecting machines.
(C) Between the time a proposed toll increase is announced and the time the increase is actually put into effect, many commuters buy more
tokens than usual to postpone the effects of the increase.
(D) When tolls were last increased on the two bridges in question, almost 20 percent of the regular commuter traffic switched to a slightly
longer alternative route that has since been improved.
(E) The chairman of the authority is a member of the Tristate Automobile Club that has registered strong opposition to the proposed toll
increase.

Karishma : The plan is to finance the improvements by increasing the toll on 2 highway bridges. What casts doubt on the plan?
The plan will be in trouble if the increase does not lead to increased collection.

Option A

Karishma : You need to cast doubt on the effectiveness of the plan i.e. if the plan is implemented, will it be effective? Will it bring in additional
revenue?
You don't have to worry about whether the plan can be implemented or not. The point is - Will it be effective?

Option (A) gives you a possible hurdle in implementing the plan. It doesn't talk about the effectiveness of the plan once it is implemented.

Option C vs D by Ron
in any case, it's pretty clear that (d) has a bigger effect than does (c) in this case, for the following TWO reasons:

(1) even if token hoarding does occur, it is extremely unlikely that the effects of token hoarding will amount to 20% OF ALL TOLLS PAID
FOR THE NEXT 5 YEARS.
that's a lot of tokens.

(2) perhaps more importantly:


IF TOKENS ARE HOARDED, then all that is lost is the DIFFERENCE BETWEEN THE OLD AND NEW FARES. this is just a % of the
fare, perhaps even a small %.
on the other hand,
IF COMMUTERS SWITCH ROUTES, then THE ENTIRE FARE IS LOST.

if you combine these 2 factors, then there's no question that (d) has a bigger effect than does (c).

Q. Because choice D cited a past fact about how people responded to the strategy, it has no relevance to the current situation.

Ron: ... but you're looking at essentially the same situation again, so, yes, it's relevant.

remember, you have to use real-world sense on these problems.


if you saw this situation on the TV news, and your friend said, "Hey, remember what happened last time there was a toll hike?" ... you
would clearly not say "that's irrelevant". because, in this situation, it's not.

there's not going to be a "rule" for whether this sort of thing is relevant; you're going to have to judge each individual situation
independently.
...which is the whole point of these problems in the first place.

Option E

Manhattan forum : This is kind of like answer (A): it might be a reason why the plan will struggle to be implemented (because too many people
object for various reasons), but it doesn’t touch on whether the plan itself is or could be effective. Remember what you were asked to analyze: not
whether the plan will be implemented but whether the plan is effective. (Plus: just because he’s a member doesn’t mean he agrees with the Club’s
opinion.

Karishma : The authority has proposed this plan. We are not concerned with who is supporting it and who is opposing it. What we need to do is
find why the plan may not work once it is brought into action. A reason why it may not be effective. We are not concerned with whether it will
actually be brought into action.

GMATprep & OG10 CR 194

A major impediment to wide acceptance of electric vehicles even on the part of people who use their cars almost exclusively for commuting is the
inability to use their electric vehicles for occasional extended trips. In an attempt to make purchasing electric vehicles more attractive to
commuters, one electric vehicle producer is planning to offer customers three days free rental of a conventional car for every 1,000 miles that they
drive their electric vehicle.

Which of the following, if true, most threatens the plan's prospects for success?

(A) Many electric vehicles that are used for commercial purposes are not needed for extended trips.

(B) Because a majority of commuters drive at least 100 miles per week, the cost of the producer of making good the offer would add considerably
to the already high price of electric vehicles.

(C) The relatively long time it takes to recharge the battery of an electric vehicle can easily be fitted into the regular patterns of car use
characteristic of commuters.

(D) Although electric vehicles are essentially emission-free in actual use, generating the electricity necessary for charging an electric vehicle's
battery can burden the environment.

(E) Some family vehicles are used primarily not for commuting but for making short local trips, such as to do errands.

Mitch hunt :

The premise is about implementing a plan.


The conclusion is about enticing commuters to purchase an electric car.

The correct answer will break the link between these two ideas: it will show that implementing the plan will DISCOURAGE commuters
from purchasing an electric car.

A: Outside the scope. The argument is not about cars used for commercial purposes but about cars used by commuters. Eliminate A.
B: Correct. If implementing the plan makes the cars expensive, commuters will be discouraged from buying them.
C: Outside the scope. The correct answer must connect the PLAN to unhappy commuters.
D: Outside the scope. The argument is not about pollution.
E: Avoid the word SOME. How many is SOME? Some could mean two vehicles out of THOUSANDS.

---------------------
Option A

Q. how this option is irrelevant?

Ron: read the first sentence of the argument again. that sentence states the problem that the manufacturer is trying to mitigate: people are
complaining that they can't take the electric cars on extended trips.

choice (a) just describes a subset of people who don't need to take the extended trips -- i.e., people who don't have the aforementioned
problem at all. since those people don't have the problem in the first place, there is no need to take them into account in considering any
plan to remedy the problem.

analogy:
if i put forward some plan to decrease obesity among americans, "lots of americans are not obese in the first place" does not, and cannot,
weaken my plan -- the non-obese americans are irrelevant to the success or failure of the plan.

Option B:
Ron: if that choice is true:

* it will make the electric vehicles even more expensive than they already are (= worse);
WHILE
* ... doing nothing for the existing problem; i.e., the new plan still doesn't make electric vehicles suitable for extended trips (= no change in
the problem).

so, that plan makes another thing (the price) worse, while not affecting the original issue.

Explanation of above comment by student vivek:

Consider this. If true, option B forces the producer to provide free rentals to customers once almost every 10 weeks! As his car sales
increase, so is the trouble he is going to get into with all the free rentals he is throwing in!
Since this is not a 'draw a conclusion / inference' type of question, we are definitely allowed to evaluate whether it would be a wise business
choice for him, in other words, whether this decision is strengthening his purpose or weakening it. Not only is this solution an extremely
poor business decision, but it doesn't even alleviate the original problem i.e. the inability to use electric vehicles for occasional extended
trips.
Weakens.

Q. are you saying that the producer will pass the cost to consumers by raising price? so that's why this offer makes the electric
vehicles more expensive and less people will be attracted?

Ron: this is exactly what you have to infer here.

the answer choice actually makes this quite explicit, by referring to "add[ing] considerably to the already high price of electric vehicles".
the only people buying electric vehicles in this passage are the consumers, so there's only one thing that could possibly mean (i.e., what you
just wrote).

Q. then B also states that "Because a majority of commuters drive at least 100 miles a week", is this relevant?

Ron : that part is not the point; the point is that the result will endanger the plan.
remember, you have only one focus in these problems: how do these things affect the issue? if something doesn't affect the issue, you can
ignore it.

e.g., here's an argument:


my friend has started eating 200 fewer calories per day. therefore, he should lose weight.

if i tell you Because of a recent shoulder surgery, my friend will have to stop swimming for a few months ... then that weakens the
argument, since reduced exercise works against the idea of weight loss. the notion of shoulder surgery is irrelevant, since that's not the part
that affects the issue.

Q. so what the argument assumed was that "offering consumers three day free rental" can solve the major impediment "occasional
extended trips"?

Ron: that's the idea behind the plan, yes (since that's essentially the only thing that the rental car can do, but that the electric car can't).

Q. 1) what the question talks about is that it is the disadvantage(inability to use electric vehicles for occasional extended trips) that hinders
the sales. It doesn't talke about money at all.
2) My predict of the correct option is one that can prove the plan will still not solve the key problem mentioned. But surprisingly it weakens
the plan from another perspective.
Ron: 1) the plan gives free car rentals. the free car rentals will solve that problem.

thus, that problem becomes a non-issue.

2) !!!

this should absolutely not be any kind of 'surprise'.


in fact, this is how ALL weakening problems work.
all of them.
every single one.

if you need to 'weaken an argument', you will ALWAYS weaken the argument by introducing an issue that has NOT been
considered in the argument.

Option E :

Ron: The focus of the problem is on people who are reluctant to buy EV's because the EV's cannot be used for extended road trips.

That choice is irrelevant to the issue. We don't care whether the primary use is for commuting or for errands; that distinction is irrelevant to
the actual problem that the plan addresses (= longer road trips)

GP2 CR 17

At present the Hollywood Restaurant has only standard-height tables. However, many customers come to watch the
celebrities who frequent the Hollywood, and they would prefer tall tables with stools because such seating would afford
a better view of the celebrities. Moreover, diners seated on stools typically do not stay as long as diners seated at
standard-height tables. Therefore, if the Hollywood replaced some of its seating with high tables and stools, its profits
would increase.

The argument is vulnerable to criticism on the grounds that it gives reason to believe that it is likely that

(A) some celebrities come to the Hollywood to be seen, and so might choose to sit at the tall tables if they were available
(B) the price of meals ordered by celebrities dining at the Hollywood compensates for the longer time, if any, they spend
lingering over their meals
(C) a customer of the Hollywood who would choose to sit at a tall table would be an exception to the generalization
about lingering
(D) a restaurant's customers who spend less time at their meals typically order less expensive meals than those who
remain at their meals longer
(E) with enough tall tables to accommodate all the Hollywood's customers interested in such seating, there would be no
view except of other tall tables

Stuart Kovinsky | Kaplan GMAT Faculty


"vulnerable to criticism" in the stem indicates that we have a logical flaw question. Flaw questions are fairly rare on the
GMAT, but they do show up occasionally. However, since flaw questions are very closely related to weakening
questions, the most common CR question type, there's no need to panic when you see one.

The main difference between flaw and weakening questions is in the types of answers that we see. For flaw, the answers
are general statements about the logic of the argument, one of which accurately characterizes the problem; for
weakening, the answers are true facts about the world, one of which makes us doubt that the conclusion is true.

In both cases, we want to deconstruct the argument.

Conclusion: swapping to some high tables and stools would increase the restaurant's profits.

Evidence 1: celebrity watchers would prefer taller tables and stools.

Evidence 2: typical sitters on stools don't stay as long as other diners.

For an argument to be valid, everything needs to be matched up. For this particular argument to be persuasive, the author
has to be assuming that celebrity watchers are typical stool-sitters.

However, since the whole reason these people are at the restaurant is to look for celebrities, common sense leads us to
believe that celebrity watchers are NOT just going to eat and run and are, therefore, atypical. (not representative of a
type, group, or class.)

(C) is a perfect match for our prediction: choose (C)!

----------------------

the author's main support for increased profits is that people who sit at the stools/high tables typically take less time to
eat.

However, in this specific case, the whole reason that many of the diners attend the restaurant is to look for and observe
celebrities. Do we expect these particular customers to eat and run? No - if their whole purpose is to celebrity watch,
then these customers are more likely to eat slowly, paying attention to the celebrities rather than their food.

So, since the specific customers of this restaurant do not match "typical" stool-sitters, the author can't properly use
evidence about typical tool-sitters to draw a conclusion about this particular restaurant.

In more general terms: when an author uses evidence about a general group to draw a conclusion about a specific group,
the author is assuming that the general group is representative of the specific group.

For a strengthening question, look for an answer that shows that the specific group is just like the general group.

For a weakening question, look for an answer that shows that the specific group is different from the general group.

For a flaw question (much rarer), look for an answer that simply states that the specific group may be atypical.

-------------------

GMATninja : This is a twisted version of a “weaken” question, and that means that we must have a conclusion in here
somewhere. And the conclusion is clearly stated at the end of the passage: "Therefore, if the Hollywood replaced some
of its seating with high tables and stools, its profits would increase."
So how did the author arrive at that conclusion? It’s funny, the passage isn’t really explicit in connecting the evidence to
the conclusion. The supporting evidence is this:

1) "customers... would prefer tall tables with stools because such seating would afford a better view of the
celebrities."
2) "diners seated on stools typically do not stay as long as diners seated at standard-height tables."

Hm, I kind of wanted something better than that, to be honest. I guess we’re left to assume that this evidence would lead
to higher profits because more customers would be attracted to the restaurant to watch celebrities, and because the diners
wouldn’t stay as long, so the restaurant could serve more people. But the passage isn’t explicit about this. And that’s part
of what makes the question so tricky: the connection between the evidence and the conclusion is left partly to the
reader’s imagination.

And the question is funky, too. "The argument is vulnerable to criticism on the grounds that it gives reason to believe
that it is likely that..." So yes, this is a “weaken” question, sort of. But there’s more to it than that. The argument "gives
reason to believe" that something is likely -- and the "something" would weaken the conclusion that the tall tables will
lead to higher profits. Tricky!

Regarding question Stem

Ron : it's more like "the argument is weak because it's likely / quite possible that..."
in other words, this problem is closest in profile to a WEAKEN problem, not a "find the assumption" problem

----------

Ron : this question works like every other 'weaken' question.

the difference—and the reason why the prompt is written differently—is that the 'weakener' is something that is almost
certainly true, given the information provided. (people come to this place to hang out and watch celebrities. thus, people
who sit at tall tables HERE—unlike people who sit at tall tables elsewhere—will probably stay for quite a while.)
BUT
this difference is irrelevant to the functionality of the problem, since ALL 'weaken' problems, including this one, deal
exclusively with the consequences that ensue if each answer choice is true.

IN SIMPLER TERMS:

• most weaken problems are


"these things may or may not be true. but, IF THEY ARE ALL TRUE, which one weakens the argument?"

• this problem is
"which one of these is probably true... and, IF IT IS TRUE, weakens the argument?"

so, functionally, you're still doing the same thing.

most importantly—
• if this problem still confuses you after a while, just walk away from it. then come back to it after a couple of weeks.

• at that point you'll have 'new eyes', and you might find that you suddenly have a better intuition for what's going on
here.

• if the problem still confuses you at that point, then just...


...smile,
and
...ignore it.

this single problem is, quite clearly, of little importance overall. (i think it's the only GMAC problem ever in this
particular format.)
do not let it distract you from more fundamental concerns.

---------------

Option A

Annynomous : A and B are straight out as they discuss about the celebrities coming for dinner.

Daagh : But still this choice is not related to making profits at all.

Kavin, magoosh : Well, this is not a problem. This is just more support for having taller tables. The celebs will come to
sit at the tall tables making it easier for people to see them. This is not a criticism. Eliminate.

Ninja : First, the passage does not "give us reason to believe" that (A) would be true. Plus, I don’t know why it would
undermine profitability. (A) is gone.

Option B

Kavin, magoosh : This too focused on the celebrities. The argument and the restaurant does not base its profits on how
much celebs spend. Profits are based on all the other people coming to the restaurant. This is too narrowly focused so
eliminate.

Ninja :
There are plenty of reasons why (B) is wrong. The passage does not "give us reason to believe" that this would be true,
for starters. Plus, I don’t really see how this would undermine the conclusion. I don’t think that the
spending by celebrities is the main issue here – or the main source of revenue for the restaurant. And if you think it is,
then this would actually strengthen the argument a little bit. (B) is out.

Option C

Ninja : Hm, yeah -- the passage definitely gives us reason to believe that this is likely. After all, the passage indicates
that celebrity-watching is the reason why customers come to the restaurant. And if (C) is true, then the restaurant
wouldn’t "turn tables" quickly, and profits would be hurt. Keep (C).
Ron: the original reasoning behind the tall tables -- i.e., the flawed reasoning described in the passage, which doesn't
account for the "paparazzi factor -- is that the diners won't stay as long.
in other words, the original thinking is that people will order normal food but leave faster if they sit at the tall tables.

on the other hand, if people want the tall tables so that they can gawk at celebrities, they're likely to stay longer ... to
gawk at celebrities.
there's no reason to suppose that they'll order any additional food (certainly, they'll order less additional food than a
completely new set of people would!), so the consideration raised in choice (c) means that the tall tables are less likely to
bring in the additional revenue that they're meant to bring in.

Ron : .. yep, that's the generalization about lingering: "diners seated on stools typically do not stay as long as diners
seated at standard-height tables".
first, it's a generalization (these people typically do this).
second, it's about lingering ("linger" = to stay longer).
... so, it's a generalization about lingering.

Q . I'm confused as to how the owner believes the tall tables will result in higher profits?

Ron: "diners seated on stools typically do not stay as long..."


--> comparable amounts of money, less time. so, more money overall.

Option D

Ninja : This might be a little bit tempting, because (D) makes it sound like it would harm profits. But remember the
exact phrasing of the question! The correct answer "gives reason to believe that it is likely that..." And there’s no reason
why this would be likely based on the passage. Plus, it’s not clear that the effects of ordering cheaper meals would
necessarily offset the effects of shorter dining times. (D) is out.

-----------

Ron : As for choice D, there is no mention of the cost of meals anywhere in the passage, so, nothing in the passage could
possibly support any notion about differential meal prices. (In fact, strictly speaking, the passage doesn't even state that
Hollywood Restaurant serves meals in the first place! For all we know, it's just a bar.)

-----------------

Ron : choice (d) does say that they spend less money on food ... but it also says that they spend less time at the tables.
either one of these factors could outweigh the other.
e.g., if they only spend 60% as much on food, but they only spend 50% as much time at the tables, then the restaurant
will wind up making 20% moremoney as a result.
(you shouldn't have to plug in specific numbers like this while you're solving the problems, but it's just an illustration.)

---------------

Q. The price didn't reflect the profit.


For example:
Beer sells at $5 but with cost $4
Coke sells at $3 but with cost $1

Ron : also remember—you should NEVER have to "plug in" NUMERICAL VALUES for a VERBAL question!
really... don't do this.

there's an entire section of this exam where you may actually want to do things like that. that's the QUANT section!

in CR, if numbers are involved, you should ALWAYS just think in very general terms about the numbers/statistics
involved and how they interact.

for instance, in choice D here, you just need to realize that "less expensive meals" don't mean lesser profits—because the
customers are leaving earlier!
you just need to think about the fact that people spending less individually could still lead to greater revenues overall, IF
they are coming and going faster. "plugging in numbers" is not going to help you understand this. (in fact, you'd have to
understand the concept ALREADY in order to "plug" the relevant numbers!)

Option E

Ninja : A gain, we have no reason to think that this is likely, and the impact on profits is a little bit murky, too. For (E) to
be correct, we’d have to assume that this actually chases customers away somehow, and that isn’t clear.

Ron: option (e) talks about a plan that is much more drastic than the plan that is actually discussed in the passage.
the plan discussed in the passage only mentions "replacing some of the tables"• with tall tables. however, option (e)
goes much further than this: it talks about the consequences of a plan in which enough tall tables are installed to
accommodate everyone who wants a tall table. this is much more extreme than the plan given in the prompt, so it's
irrelevant.

------------

Daagh : No reference to profits.

Kavin, Magoos : This might also be a problem, but the argument doesn't say that they are going to jam tables into the
restaurant. There is no mention of adding more seating. They are merely going to replace tables that they have. So this is
outside the scope of the argument and wrong.

Q. How can I improve in CR?

Ron : My best two answers here are...


... Name the speaker,
... Imagine the argument as an actual conversation with that speaker.

If you can successfully formulate this problem as a conversation, then it will be obvious that "diners on stools typically
do xxxx" is a generalization -- because you'll understand why the conversation is happening. The conversation is
happening because the Hollywood doesn't have stools at all right now, and Joe is arguing that some should be installed.

(Framing arguments as conversations will also do away with the useless (and counterproductive) habit of looking for
"rules" that will solve CR problems. After all, no one looks for rules when they have conversations! You just think about
what the other person is saying.)

======================================

Assumption Questions

======================================

Assumption List : OG16 40, 47, 52, 84

- Comparison of Option A & Option B : OG13 41 (OG16 47), OG13 48 (OG16 52), OG16 90,
- Fee imposed : OG13 46 (OG16 40)
- Cause and effects : OG13 90,
- Assumptions with Fill in the gaps :

OG16 CR 84

Proposed new safety rules for the Beach City airport would lengthen considerably the minimum time between takeoffs
from the airport. In consequence, the airport would be able to accommodate 10 percent fewer flights than currently use
the airport daily. The city’s operating budget depends heavily on taxes generated by tourist spending, and most of the
tourists come by plane. Therefore, the proposed new safety rules, if adopted, will reduce the revenue available for the
operating budget.

The argument depends on assuming which of the following?

(A) There are no periods of the day during which the interval between flights taking off from the airport is significantly
greater than the currently allowed.

(B) Few, if any, of the tourists who use the Beach City airport do so when their main destination is a neighboring
community and not Beach City itself.

(C) If the proposed safety rules are adopted, the reduction in tourist numbers will not result mainly from a reduction in
the number of tourists who spend relatively little in Beach City.

(D) Increasing the minimum time between takeoffs is the only way to achieve necessary safety improvements without a
large expenditure by the city government on airport enhancements.
(E) The response to the adoption of the new safety rules would not include an increase in the number of passengers per
flight.

OE :

Situation Proposed safety rules for a city airport would reduce the number of daily flights the airport can accommodate.
The city’s operating budget depends heavily on taxes generated by tourists, who mostly come by plane. Therefore,
adopting the safety rules will result in lower revenue available for the operating budget.

Reasoning What must be true in order for the cited facts to support the conclusion that the proposed rules would reduce
the revenue for the operating budget?

The implicit reasoning is that since the rules would reduce the number of flights that can be accommodated, they would
thereby reduce the number of tourists arriving by plane, which in turn would reduce the tax revenue that tourist spending
generates for the operating budget. This assumes that the actual number of daily flights would fall along with the number
that the airport can accommodate; that fewer daily flights would mean fewer people flying into the airport; that fewer
people flying into the airport would mean fewer tourists flying into the airport; that fewer tourists flying into the airport
would mean fewer tourists visiting the city; that fewer tourists visiting the city would mean less taxable spending by
tourists; and that less taxable spending by tourists would mean less revenue overall for the operating budget

-------------------------------

First look at the stimulus, and understand the argument

After you decompose the argument into premises and conclusion, detect which are one, and fully understand how the
passage flows, then it is necessary pre-think the answer.

Pre-thinking :

For this, think about how can the conclusion not be true. Our conclusion is that the proposed new safety rules, if
adopted, will reduce the revenue available for the operating budget so now think about possible scenarios where this
might not be true.

1-What if proposed legislation in fact will not necessarily mean lower revenues, is that possible? if it is possible then
how? are the same number of passenger going to fly, is the ticket price going to increase while maintaining the same
number of passengers?

Once you start thinking of possible weakeners, then you predict your assumption that such weakeners are not going to
happen. In doing so, you are "protecting your argument" and that is what an assumption really does. "An assumption
(the hidden idea or premise we take for granted when reading our argument) must be true for the conclusion to hold true.

So next step is to find some prediction similar to your pre-thinking analysis, eliminating wrong answer choices i.e. out of
scope, too extreme, too broad, or simple re-statements (These are the usual wrong answers in such questions)

Answer choice E, IF we negate it, it means that "The response to the adoption of the new safety rules would include
an increase in the number of passengers per flight. If it includes an increase, such increase could either cancel out the
negative effect or bring even more customers and hence more money per flight. Simply our conclusion would fall apart.

--------------

Pre-thinking :

WillGetIt, verbal forum moderator :

Possible assumptions are :


- Tourists will spend in similar capacity as to before adoption of safety laws
- Tourists have not decided to stay back in city more.
- The number of tourists per plane increased as compared to situation before adoption of safety laws.

Karishma :

Premises:
New rules will increase the minimum time between takeoffs (say from 10 mins to 15 mins)
The airport would be able to accommodate 10 percent fewer flights (Airport capacity will decrease by 10%)
City’s operating budget depends on taxes generated by plane-using tourists.

Conclusion: So new rules, if adopted, will reduce the revenue available for the operating budget.

To arrive at the conclusion, you are making a lot of assumptions:


1. Decrease in capacity will actually lead to decrease in number of flights.
2. Decrease in number of flights will actually lead to decrease in number of tourists in the city
3. Decrease in number of tourists will actually lead to decrease in revenue (tourists will not start spending extra)
4. Decrease in tourist revenue will actually decrease revenue available for budget (it will not be compensated in another
way).

Q. I was just wandering whether the assumption number 01 mentioned by you is actually a assumption?

Karishma : Say, the airport can handle 48 flights in a day since it is essential to have 30 mins between takeoffs. So the
capacity of the airport is 48 flights.
But it actually handles only 24 - with one hour between take offs.
Now, say new norms require that there should be at least 45 mins between two flights so the capacity of the airport goes
down. But will the number of flights reduce? No. There is already more than 45 mins between successive takeoffs. So a
decrease in capacity needn't actually cause a decrease in the number of flights.

-------------------

Empowergmat :

Type: Assumption
Boil It Down: Safety rule reduces # of flights -> Tourism revenue/operating budget down
Missing Information: No other factors
Goal: Find the option that exposes some missing information the argument takes for granted.
Analysis: The person making the argument makes the wholesale assumption that just because the number of flights will
be reduced that the number of people on the remaining flights would be reduced. What if they're flying super-jumbo jets
such as the Airbus A380? How do we know that the capacity of the remaining flights couldn't be increased by using
higher capacity aircraft such as the A380?
-------------------------

LogicGuru1

---------A increases-------------------------> B decreases ----------------------------> C decrease


Security check time increase--------->Number of flight decreases-------->Number of Tourist decreases
. .(A). .(B). .(C).

Our goal is to Maintain C (Number of tourists)

A (Security Check time )cannot be tinkered with, It will stay as it is,


B (Number of Flights ) cannot be changed either ; BUT BUT BUT it can be modified in such a way that C (Number of
tourists ) stays constant ---> In summary although the number of flight cannot be increased but size of the planes can be
increased, so that every flight coming to the airport can carry more number of tourist whenever it arrives.

So the argument assumes that it cannot happen--> Number of people in flight cannot be increased and therefore number
of tourist will decrease

What options says so :--->


E) The response to the adoption of the new safety rules would not include an increase in the number of passengers per
flight

Thus E is the assumption that is made by the author.

THEREFORE CORRECT ANSWER IS E

--------------------------------

hazelnut, Senior SC Moderator

Beach City Airport

Step 1: Identify the Question

The phrase argument depends on assuming in the question stem indicates that this is a Find the Assumption question.

Step 2: Deconstruct the Argument

new rules = ↑ time between takeoffs = ↓ flights


↓ flights = ↓ tourists = ↓ operating budget

The argument depends on a series of connections: if one thing decreases, then another will also decrease. Note that if
any one of these connections were invalid—for instance, if the decrease in flights didn’t actually decrease the number of
tourists—the argument would no longer be valid.

Step 3: Pause and State the Goal

On Assumption questions, the goal is to pick a statement on which the argument’s logic depends. The right answer will
be something the author must believe to be true in order for the argument to be reasonable.
Step 4: Work from Wrong to Right

-------------------------

Mitch Hunt :

Premise:
The proposed new safety rules will reduce by 10% the number of flights available to tourists, and the city’s operating
budget depends heavily on taxes generated by tourist spending.
Conclusion:
The proposed new safety rules will reduce the revenue available for the operating budget.

Apply the NEGATION TEST.


When the correct answer choice is negated, the conclusion will be invalidated.
E, negated:
The response to the adoption of the new safety rules would include an increase in the number of passengers per flight.
Here, an increase in the number of passengers per flight invalidates the conclusion that there will be a decrease in tourist
revenue.
Since the negation of E invalidates the conclusion, E is the correct assumption: WHAT MUST BE TRUE for the
conclusion to be valid.

--------------------------

Option A There are no periods of the day during which the interval between flights taking off from the airport is
significantly greater than the currently allowed.

OE : Even if flights depart the airport less frequently during some periods of the day, increasing the minimum time
between flights at busy times of day could reduce the total number of daily flights from the airport.

mmelendez, GC

There are no periods of the day during which the interval between flights taking off from the airport is significantly
greater than the currently allowed. This could be true but it is not necessarily true. If we negate this option, meaning
that There are periods of the day during which the interval between flights taking off from the airport is
significantly greater than the currently allowed and we critically think as how this affect our conclusion, we see that
this option does not make the argument less likely. Even if there are no periods greater than the currently allowed, this
reasoning does not give me the connection that the conclusion states that the proposed legislation will lead to decreased
revenues.

Karishma :

There is a problem with (A). There could be periods of day during which interval between flights is more - say the 12
noon to 4 pm slot. But still, it is possible that the number of flights are reduced, say in the peak hours of 7 pm to 10 pm.
We don't know whether it is feasible to readjust flight timings to occupy free slots. Hence, we cannot assume that there
are no free slots.
Empowergmat : A) This option that there are no periods of time between takeoffs greater than the allowed minimum
(as some sort of possible hidden way to increase the number of flights perhaps) is entirely irrelevant since the argument
establishes as a fact: "the airport would be able to accommodate 10 percent fewer flights than currently use the airport
daily."

hazelnut: This answer choice appears to support the connection between increased time between takeoffs and a
decreased number of flights. If there were currently ‘quiet periods’ at the airport, couldn’t extra flights be squeezed in, to
avoid reducing the total number of flights while still obeying the rules? However, the argument already specifies that the
new rules will result in at least a 10% decrease in the number of flights. This is a statement of fact, so no further
assumptions need to be made in order to support it.

Option B: Few, if any, of the tourists who use the Beach City airport do so when their main destination is a neighboring
community and not Beach City itself.

OE : Even if half the tourists flying into the airport were bound for other nearby towns, the other half could still spend
enough in town to generate lots of revenue for the operating budget.

Empowergmat : B) This option discusses an entirely irrelevant factor. The argument could still hold whether some
people go to neighboring communities. This option is not something that the logic of the argument hinges on.

Karishma : Even if some tourists are hopping over Beach city, there is no reason to believe that only they will be the
ones switching to another route. Fewer flight availability would mean fewer for everybody. The flights are not allotted
according to final destinations. No preference is given to people entering the city as opposed to people leaving from the
airport itself. Hence, (B) is not correct.

hazelnut: Even if Beach City tourists represented a very small fraction of those arriving in the city by airplane, a
decrease in the number of flights would still decrease their numbers proportionally, resulting in a lower operating
budget.

Mitch Hunt : B, negated:


Most of the tourists who use the Beach City airport do so when their main destination is a neighboring community.
Since tourists visiting a neighboring community probably spend most of their vacation money in the neighboring
community, this negation seems to weaken the PREMISE that Beach City's operating budget depends heavily on taxes
generated by tourist spending.
A premise is a FACT.
It cannot be weakened.
Any answer choice or negation that attempts to weaken a premise is WRONG.
Eliminate B.

Option C : If the proposed safety rules are adopted, the reduction in tourist numbers will not result mainly from a
reduction in the number of tourists who spend relatively little in Beach City.

OE : It is possible that most tourists spend relatively little in the city, but a few spend a lot. In that case, even if a
reduction in tourist numbers resulted mainly from a declining number of tourists who spend relatively little, it
could also greatly reduce the already small number of tourists who spend a lot.

Karishma : Negated (C): If the proposed safety rules are adopted, the reduction in tourist numbers will result mainly
from a reduction in the number of tourists who spend relatively little in Beach City.

The reduction will be in the number of tourists who spend relatively little.
Is it still possible that the revenue available for the operating budget will reduce?

Yes, these tourists are spending little, relatively speaking. They could still be spending a substantial amount. Also, the
number of these tourists could be rather high and a big reduction could actually cause a substantial reduction in the
revenue. Anyway, our conclusion talks about a reduction, not a substantial reduction so if there is any kind of reduction,
the conclusion holds.

On negating (C), the conclusion can hold and hence (C) is not our assumption.

Empowergmat: C) This one involves some tricky logic, so let's evaluate with the Opposite Test: Even if the reduction
in travellers were mainly attributable to people who spend relatively little, the argument still holds because this group
would still be spending something. This option is DEFINITELY not something the logic of the argument requires.

hazelnut: If the reduction will not consist mostly of low spenders, then it will consist mostly of high spenders. A
reduction in the number of tourists who spend a lot would have a large effect on the operating budget. Therefore, this
answer choice strengthens the argument. However, although this is a strengthener, it isn’t an assumption, because it
doesn’t have to be true in order for the logic of the argument to hold. Imagine a scenario in which 10% of the tourists
spent $1 in Beach City, while the remaining 90% spent $1000 each. Even if the 10% who spent $1 were those who
stopped visiting due to a lack of flights, that still represents an overall decrease in revenue. Although this answer choice
would strengthen the argument, it isn’t necessary to the argument, since it could be false and the argument could still
hold.

Option D : D Increasing the minimum time between takeoffs is the only way to achieve necessary safety improvements
without a large expenditure by the city government on airport enhancements.

OE :This suggests that the proposed rules might be financially better for the city than any alternative way to improve
safety, whereas the argument’s conclusion is that the proposed rules are financially disadvantageous.

Empowergmat : D) The argument doesn't require the proposal to be the ONLY way to increase the safety standards.
There could be other proposals, and this argument about this proposal still stands. Therefore, this option is not required
either. Gone.

hazelnut: It doesn’t matter whether there are other ways to achieve safety improvements. The conclusion addresses
only the effects of this particular improvement, not why it was selected or whether it was superior to the alternatives.

Option E The response to the adoption of the new safety rules would not include an increase in the number of
passengers per flight.

OE : Correct. If adopting the proposed rules would result in a large in increase in the number of passengers per flight,
fewer daily flights would not necessarily mean fewer passengers or fewer tourists overall.
mmelendez, GC: IF we negate it, it means that "The response to the adoption of the new safety rules would
include an increase in the number of passengers per flight. If it includes an increase, such increase could either
cancel out the negative effect or bring even more customers and hence more money per flight. Simply our conclusion
would fall apart.

Karishma : This is our point 2 given above. We are assuming that decrease in number of flights will lead to decrease in
number of tourists. So we are assuming that the reduced flights will not carry increased number of passengers.
This is correct.

Empowergmat : E) Here we are! The argument ABSOLUTELY assumes that the number of passengers per
flight won't increase. Applying the Opposite Test: if the number of passengers per flight were to increase, then
the whole condition on which argument is based (fewer travellers) is nullified, and therefore the argument would
collapse. This option IS required for the logic of the argumenq to hold---the argument depends on this
assumption.

hazelnut: This must be true in order for the argument to be logically sound. If it weren’t true, then the number of
passengers per flight would increase and it would no longer be possible to conclude that the overall number of tourists
coming to Beach City would decrease. In this case, the operating budget might not decrease after all.

Manhattan GMAT CAT

A newly discovered painting on wooden panel by Michelangelo must have been completed after 1507 but before 1509.
It cannot have been painted earlier than 1507 because one of its central figures carries a coin that was not minted until
that year. It cannot have been painted after 1509 because it contains a pigment that Michelangelo is known to have
abandoned when a cheaper alternative became available in that year.

Which of the following is an assumption on which the argument depends?


- No stocks of the abandoned pigment existed after 1509.
- Michelangelo did not work on the painting over the course of several years.
- The coin depicted in the painting was known to general public in 1507.
- The wooden panel on which the painting was executed cannot be tested accurately for age.
- Michelangelo's painting style did not change between 1507 and 1509.

Geva@EconomistGMAT

The answer is B, but it's difficult to see without trying to predict what the answer should do before going to the answer
choices. For many assumption questions, the key is to think critically - think about what would weaken the conclusion.
Attacking an argument is a good way to expose the underlying assumptions that the author makes, but does not state.

There are actually two sets of premises and conclusions in this argument:
Premise: the painting contains a coin that was not available before 1507.
Conclusion: the painting was not painted before 1507.

AND

Premise: Painting contained a pigment that Michelangelo abandoned in 1509.


Conclusion: The painting was finished by 1509
the second one is easier to attack: the fact that the painting contained a pigment (not painted entirely in the pigment -
just contained the pigment) does not by itself say that Michelangelo could not continue working on painting beyond
1509 using different kinds of pigments.

The first is attacked in the same manner: the fact that part of the painting (in this case, a coin) has a "not before 1507"
date does not meant that the painting as a whole was painted after 1507: It is possible that other parts of the painting
were painted before 1507, with the coin added AFTER 1507 at a later stage.

These are weakening hypotheses, not found in the text, but they expose the assumption - the author is assuming that
"expiry dates" on parts of the painting represent the entire painting as a whole - that if he started on the coin in 1507, he
started the entire painting at 1507, and if he finished with the pigment in 1509, he must have finished the entire painting
before that time.

B works against that assumption: If Michelangelo paused in his work on the painting, that would explain why the
painting could be painted outside of the range, with only the relevant parts painted within the range. Therefore, in
reaching his conclusion that the parts represent the whole, the author must assume that the painting was painted more or
less within a short period of time - a year or two. B is a necessary assumption because it eliminates a possible counter
explanation that would weaken the conclusion.

--------------

Ron: it appears that you misunderstand the basic nature of "find the assumption" questions.

in these questions, your job is to find an assumption that is NECESSARY IN ORDER TO DRAW THE STATED
CONCLUSION.
in other words:
the conclusion is there. YOU ARE NOT CONCERNED WITH WHETHER OTHER CONCLUSIONS MIGHT ALSO
BE DRAWN. you are only trying to pick something that MUST be true IF your conclusion is to be considered valid.

Option A

Stacey : Note: option A says that no stock existed after 1509 period - anywhere, not just in Michelangelo's possession. Is
it possible that the pigment still existed in the world but Michelangelo didn't use it for the stated reason? Sure - that
wouldn't affect this author's claim that the painting was painted between 1507 and 1509.

Option B

Stacey : For option B, however, try this: Michelangelo started painting the canvas in 1505, but put it aside for a while
and then finished it in 1507, at which time he painted in the new coin. Or, Michelangelo started painting with the old
pigment in 1508 and finished that particular color then. He finished the rest in 1510 but didn't need to use the particular
pigment in question again - maybe it was blue and he was done with all of the blues in 1508 but added some yellow in
1510. If either of those scenarios is true, there goes the author's claim.
Q. Why would I assume that "Michelangelo did not work on the painting over the course of several years" if the
argument leaves open the possibility that he could have started the painting in 1508 and finished it in the same year?

Tim : Exactly. If we assume that he completed the work all in a single year, then 1508 is definitely one of the years this
could have happened, which is entirely consistent with the argument. Assuming he did everything in a single year
restricts the dates in such a way that the conclusion is sound. If on the other hand we allowed him to to work for several
years, he could have started before 1507 and/or ended after 1509. This is why we need the assumption in order to be sure
of the conclusion..

OG16 CR 8

A newly discovered painting seems to be the work of one of two seventeenth-century artists, either the northern German Johannes Drechen or the
Frenchman Louis Birelle, who sometimes painted in the same style as Drechen. Analysis of the carved picture frame, which has been identified as
the painting’s original seventeenth-century frame, showed that it is made of wood found widely in northern Germany at the time, but rare in the
part of France where Birelle lived. This shows that the painting is most likely the work of Drechen.

Which of the following is an assumption that the argument requires?

(A) The frame was made from wood local to the region where the picture was painted.
(B) Drechen is unlikely to have ever visited the home region of Birelle in France.
(C) Sometimes a painting so resembles others of its era that no expert is able to confidently decide who painted it.
(D) The painter of the picture chose the frame for the picture.
(E) The carving style of the picture frame is not typical of any specific region of Europe.

sayantanc2k :

Premise: Wood found in Germay is used to frame the painting.


Conclusion: The painting was made in Germany.

What you are arguing in your post is that it is not correct to conclude that the painting was made in Germany, but the question is not about whether
it is right or wrong to conclude so - if someone (the author) has already concluded so (as given in the passage), then what is his / her assumption is
behind that conclusion ? Option A states an assumption behind that author's conclusion, linking the premise and conclusion mentioned above (it
does not matter whether the conclusion is right or wrong).

In summary: It is not required to evaluate whether a premise used to conlude something is valid. The point is if someone uses a premise to arrive at
a conclusion (rightly or wrongly), then what is that person's assumption linking the premise and conclusion.

---------

Mike McGarry : Part of what is odd about your analysis is that you don't ever mention the conclusion of the argument and seem unaware of the
conclusion. The conclusion is:
... the painting is most likely the work of Drechen.
The conclusion is about the issue of who painted the painting. You are focusing too much on the frames.

The fact that the wood of the frame is plentiful in German and rare is France is used as evidence. Does this prove without a shadow of a doubt that
the frame was made in Germany? Of course not. But it does make it more likely that the frame was made in Germany. Notice that the conclusion is
stated in terms of likelihood, not ironclad certainty. Nothing in this argument is about establishing any point beyond all doubt.

---------------------

ceilidh.erickson
With FIND ASSUMPTION questions, I recommend the following approach:

1. Try to find the logical gap in the argument before looking at the answer choices. Do this by asking, "taking the premises as true, is there
a way for the conclusion not to be true?"

2. When looking through the answer choices, use the Negation Test: because the right answer will be an assumption necessary to uphold
the conclusion, the opposite of the right answer should break the conclusion.

Premises:
- the German & French painters had the same style
- the frame is made of wood more often found in Germany than France

Conclusion: the painting is most likely the work of the German.

Logical Gaps:
- what if wood was transported from one region to the other?
- what if the painters themselves traveled from one region to the other?
- what if the painting was shipped elsewhere to be framed?
- just because the wood is rare in France, does that mean we don't have any frames from that region made from that wood?
Assumption: The painting, painter, and frame are all from the same region.

Answer choices: Apply the Negation Test.

ceilidh.erickson :(A) The frame was made from wood local to the region where the picture was painted.
If the frame were NOT made from wood local to the region where the picture was painted, then knowing the provenance of the frame
would give us no information about where it was painted, and thus who the painter was. The conclusion would fall apart --> this must be
the necessary assumption!

Mike McGarry : Choice (A) links the evidence to the conclusion--it provides a bridge. That is one job of an assumption. We could also use the
negation test: if the frame was NOT "made from wood local to the region where the picture was painted," then it could have come from Italy or
Russia or wherever, and the composition of the frame tells us nothing about the likely artist. In other words, if we deny this, the argument
collapses. That is also a telltale sign of an assumption.

hazelnut : (A) CORRECT. This must be true in order for the author’s reasoning to be sound. If the frame had instead been made from wood that
originated elsewhere, the author couldn’t draw any connection between the frame’s origin and the painting’s origin.

(B) Drechen is unlikely to have ever visited the home region of Birelle in France.
ceilidh.erickson : If Drechen DID visit France, this still wouldn't negate the idea that German wood in frame indicates a German painting.
He could still visit that area, but if he mostly painted in and used frames from Germany, the argument would still hold. (If Birelle had
visited Germany and possibly acquired German wood, that would be more relevant). The right answer on an Assumption question must
be necessary for the conclusion to hold. This answer might slightly strengthen, but it's not necessary.

Mike McGarry : Choice (B) is a weak distractor. Let's flamboyantly negate that. Suppose Drechen "visited the home region of Birelle in France"
every six months of his life! Suppose Drechen had family there whom he always visited. On these frequent trips, would Drechen have brought his
painting supplies? Would he have brought a frame? Would he have bought a new frame while he was there? We don't know the answers to any of
these questions. Without further information, we can't tell whether this would strengthen or weaken the argument. If we negate a statement, and the
effect on the argument is ambiguous, then we definitely do not have an assumption.

hazelnut :(B) Since the painting was framed with German wood, not French, whether Drechen visited France is irrelevant. The argument already
makes it clear that the painting was not framed with wood from France, regardless of who visited the area and who created the painting.

(C) Sometimes a painting so resembles others of its era that no expert is able to confidently decide who painted it.
ceilidh.erickson :
Does this HAVE to be true for the argument to hold? No. We're just trying to ascertain the provenance of this particular painting.

sayantanc2k : The artist himself or someone else in the same location (e.g. a trainee or someone who exhibited the painting) might have chosen
the frame. Option D is not is not a mandatory link between the premise and the conclusion - it can at most be a strengthening statement, but it does
not satisfy "must be true" scenario required by an option to be qualified as an assumption.

(D) The painter of the picture chose the frame for the picture.
ceilidh.erickson :If the painter did NOT choose the frame, would the whole argument fall apart? No - someone else could choose the
frame, but as long as the frames tend to come from the same region as the paintings, the chain of logic in the argument holds.

hazelnut :(D) The artist didn’t necessarily have to choose the frame himself. Somebody else in the same geographic area, such as a local buyer,
might have chosen the frame.

(E) The carving style of the picture frame is not typical of any specific region of Europe.
ceilidh.erickson :

If the carving style WAS typical of a specific region, would the argument fall apart? No, that would actually strengthen the idea that we
know where the frame is coming from. It wouldn't do anything to tell us whether the region of the frame indicates the region of the
painting, though.

The correct answer is A.

hazelnut It’s necessary to assume that the carving style wasn’t especially typical of France, or the argument’s reasoning would be damaged,
since it would then be more likely that the frame was created in France. However, the author didn’t assume that the carving style wasn’t typical of
any European region. It could have been typical of Germany, or of some other region in Europe outside of both artists’ areas, and the conclusion
would still have been reasonable.

OG13 CR77

A recent report determined that although only 3 percent of drivers on Maryland highways equipped their vehicles with radar detectors, 33 percent
of all vehicles ticketed for exceeding the speed limit were equipped with them. Clearly, drivers who equip their vehicles with radar detectors are
more likely to exceed the speed limit regularly than are drivers who do not.

The conclusion drawn above depends on which of the following assumptions?

(A) Drivers who equip their vehicles with radar detectors are less likely to be ticketed for exceeding the speed limit than are drivers who do not.

(B) Drivers who are ticketed for exceeding speed limit are more likely to exceed the speed limit regularly than are drivers who are not ticketed.

(C) The number of vehicles that were ticketed for exceeding speed limit was greater than the number of vehicles that were equipped with the radar
detectors.

(D) Many of the vehicles that were ticketed for exceeding the speed limit were ticketed more than once in the time period covered by the report.

(E) Drivers on Maryland highways exceed the speed limit more often than drivers on other state highways not covered in the report.

OE :

Situation : A recent report determined that although only 3 percent of drivers on Maryland’s highways have radar
detectors in their vehicles, 33 percent of vehicles recently ticketed for driving over the speed limit on Maryland
highways have had radar detectors. Drivers who have radar detectors are thus more likely to exceed the speed limit
regularly than drivers who do not.

Reasoning : What assumption must be true for the conclusion to be drawn? The argument moves from a particular
example, that is, the percentage of vehicles ticketed for exceeding the speed limit that were equipped with radar
detectors, to a generalization about the regular driving behaviors of all drivers who have radar detectors in their vehicles.
The reasoning links the example to the generalization with an assumption. What can the assumption be? Only if the
drivers ticketed in this instance are assumed to make a regular habit of exceeding the speed limit can the conclusion be
drawn that drivers with radar detectors are more likely to do so regularly than drivers who are not ticketed.

Karishma : There must be some disconnect between the premises and conclusion since there is an assumption in the
argument. Look carefully. Premises give you the connection between ‘vehicles that have radar detectors’ and ‘vehicles that
get speeding tickets’. The conclusion, on the other hand, concludes a relation , give a connection between ‘vehicles that get
speeding tickets’ and ‘vehicles that exceed speed limit’.

Sarfaraz : the entire argument hinges on the argument that there is a word ‘regularly’. The evidence definitely shows
that , al least when the report was make, these people having radar exceeded the speed limit proportionately more.
However from this one behavior cann’t be generalized this the regular way they behave. That is basically the cracks of
this argument. So the assumption is that what is observed is actually holds true for all the time.

Ninja :

if we're looking for an assumption, then we probably will need something that connects the conclusion (i.e., that drivers with radar
detectors actually exceed the speed limit regularly) with the evidence about speeding tickets. And as always, you can think of an assumption as
something that not only strengthens or reinforces the conclusion, but also is necessary to draw that conclusion.

Stacey : 3% of all equipped with detectors


33% of vehicles caught speeding equipped with detectors
Conclusion: drivers who use detectors are more likely to speed regularly than those who don't

Author has made a leap between speeding only occasionally and getting (unluckily) caught vs. speeding regularly - but the rest of the
argument does not actually mention anything about the frequency of speeding of various groups. It may be obvious that the more often you
speed, the more chances you have to get caught - but the argument does not spell this out.

--------------

(A) Drivers who equip their vehicles with radar detectors are less likely to be ticketed for exceeding the speed limit than are drivers who do not.

Ninja : (A) strengthens the conclusion: if drivers with the radar detectors are LESS likely to be ticketed than drivers without them, then the
drivers with radar detectors must exceed the speed limit even MORE often than it initially seemed based on the argument.

Trouble is, the question isn't asking us to strengthen the argument; it's asking us to identify a necessary assumption. And we don't need to assume
that drivers with radar detectors are less likely to be ticketed than other drivers. After all, 33% of ticketedvehicles have radar detectors, while only
3% of all vehicles have radar detectors, so if vehicles with radar detectors are just equally likely to receive tickets when they exceed the speed limit,
the conclusion could still hold. Heck, drivers with radar detectors could even be somewhat more likely to receive tickets -- and as long as the
disparity isn't huge, the conclusion could still hold.

So we certainly don't need to assume that vehicles with radars are LESS likely to receive tickets. (A) is out.
OE : While this statement about being ticketed may be true, the conclusion pertains to the recurrent exceeding of the
speed limit, so this statement is not relevant.

(B) Drivers who are ticketed for exceeding speed limit are more likely to exceed the speed limit regularly than are drivers who are not ticketed.

Ninja : This answer choice seems so lame and obvious -- of course you're more likely to exceed the speed limit if you receive more speeding
tickets, right? -- that I kind of want to ignore it completely. But that would be a bad thing to do.

Let's come back to the argument: the conclusion says that people with radar detectors ACTUALLY exceed the speed limit more often than drivers
without them, but the evidence in the passage only mentions the fact that vehicles with radar detectors receive a disproportionate percentage of
TICKETS for speeding. So we absolutely do need to assume that drivers who receive tickets are more likely to actually exceed the speed limit.
Otherwise, there's no connection whatsoever between the evidence and the conclusion.

So let's keep (B).

OE : Correct. This statement properly identifies the conclusion’s necessary assumption about ticketed drivers’ being
more likely to drive in excess of the speed limit than nonticketed drivers.

(C) The number of vehicles that were ticketed for exceeding speed limit was greater than the number of vehicles that were equipped
with the radar detectors.

OE : This statement is about the number of vehicles ticketed, not about the regular habits of drivers, so it is not assumed
for the conclusion.

(D) Many of the vehicles that were ticketed for exceeding the speed limit were ticketed more than once in the time period covered by the report.

OE: While this additional information could help support the conclusion, it is not a necessary assumption in the
conclusion because it is about the particular example of the drivers in Maryland, not about drivers’ habits in general.

Ninja : (D) is kind of interesting, but it doesn't actually help us draw the conclusion.

For starters, we don't know which vehicles were ticketed more than once: if the vehicles with radar detectors were more likely to receive multiple
tickets, then it would reinforce the idea that vehicles with radar detectors receive TONS of tickets. But it could also go in the other direction: if
vehicles WITHOUT radar detectors were more likely to receive multiple tickets, then it would weaken the force of the evidence in the passage, and
undermine the conclusion. For that reason alone, we could ditch (D).

And just as importantly: remember that the conclusion discusses the likelihood that drivers actually exceed the speed limit. (D) doesn't help us
reinforce this conclusion at all, because it just tells us something about the number of tickets received for speeding.

So we can eliminate (D), too.


Assumption End

==================================

Inference Questions

==================================

Question Pack 1 | CR 1

Roland: The alarming fact is that 90 percent of the people in this country now report that they know someone who is unemployed.

Sharon: But a normal, moderate level of unemployment is 5 percent, with 1 out of 20 workers unemployed. So at any given time if a person knows
approximately 50 workers, 1 or more will very likely be unemployed.

Sharon's argument is structured to lead to which of the following as a conclusion?

(A) The fact that 90% of the people know someone who is unemployed is not an indication that unemployment is abnormally high.
(B) The current level of unemployment is not moderate.
(C) If at least 5% of workers are unemployed, the result of questioning a representative group of people cannot be the percentage Roland cites.
(D) It is unlikely that the people whose statements Roland cites are giving accurate reports.
(E) If an unemployment figure is given as a certain percent, the actual percentage of those without jobs is even higher.
OE:
Argument Construction
Situation
Roland is alarmed that 90% of the population knows someone who is out of work. Sharon replies that a normal level
of unemployment is 5%, illustrating her point by saying that if a person knows 50 workers, at least one of them is
likely to be unemployed.
Reasoning
Sharon’s reply leads to what conclusion about unemployment? Sharon begins her reply with "but," indicating that she
is about to counter either Roland's statistic or his alarm; she accepts the statistic and addresses the alarm. If the
normal level of unemployment rate is 5% and if the average person knows 50 workers, then knowing one person out
of work is within the normal and expected range, not a cause for alarm. Sharon shows that it is possible for 90% of
the population to know someone unemployed and for unemployment to be a normal rate of 5% at the same time.

----------

Ron:
the first question is one of the less often encountered question types. here's what you have to realize about this sort of
question: if you understand the argument, then you basically already know the right answer.

what this means is that the only way they can really lead you astray is to give you tricky answer choices that sound like
they might be plausible, even if they have absolutely nothing to do with the goal of the argument.

here's how you can defend against those sorts of answer choices:
ALWAYS PREDICT THE ANSWER YOURSELF BEFORE YOU LOOK AT THE CHOICES for these kinds of
problems.

this is also the same advice to you would use for "main idea" problems on reading comprehension. basically, unless you
just don't understand the passage at all, you should already have a pretty good idea of what the main point is -- so get an
answer to the question in your head, and ONLY THEN look at the choices.

--

on the other hand, if you don't really see the point of the argument, then it's going to be very difficult to answer these
problems. unlike "draw the conclusion" problems, you are being asked about the gist of the argument; you are not being
asked to produce an answer that is necessarily 100% provable.

on "draw the conclusion", on the other hand, you are normally being asked to select an answer that MUST be true --
something that's still possible even without a firm grasp of the argument, just by ruling out answers that are out of
scope.

i guess what i'm basically trying to say here is that these problems are essentially going to be random guesses, unless
you can understand what the author of the argument is driving at. unlike a couple of the other genres of critical reasoning
(such as "draw the conclusion" and "find the assumption"), there's not really going to be a consistent way to reason these
out formally.
the point of sharon's argument is that the statistic that roland has cited as "alarming" is not, in fact, alarming.

this is sort of hard to do if you don't have an intuitive grasp of the argument, but i'll do my best to put forward a
technique with which you might solve this sort of thing: pay attention to the way in which the speakers' CLAIMS
are related.
in general, the speakers are not going to attack each other's facts; i've never actually seen a problem on which one
speaker actually disputes the factual accuracy of the statements made by the other.
so, it follows that the purpose of any sort of dialogue, in these questions, is related to claims that the speakers are
making, not to the facts underlying those claims.

this observation makes this problem much more accessible, since roland is only making one CLAIM: namely, that
the given statistic is "alarming". (remember, unless the gmat decides to violate all previous precedents, the response is
not going to dispute the factual accuracy of the 90% figure.)

sharon's argument starts with the word "but", so it's clear that sharon's purpose is to dispute roland's claim.
therefore, you should pick the choice that is closest to "sharon says that this statistic is not alarming". that would be (a).

-----------------------------

GMAT Critical Reasoning: Infer like a Master


Forum post by Stacey

Let’s start by talking about what you need to do for Inference questions in general. Then we’ll tackle the problem.

Inference arguments will provide you mostly with a series of premises and they’ll ask you to infer or to draw a
conclusion in some way.

Now, here’s the key: the test does not want you to infer or conclude in the way that you likely would in the real world.

(Did I emphasize that word strongly enough? ) In the real world, we conclude things that are likely to be true, given
certain information.

On GMAT Critical Reasoning, you’ll need to hold yourself to a higher standard: What must be true, based on the
information you were given? (Not just what is likely or reasonable to believe.)

Okay, let’s dive into this problem and see how this “what must be true?” concept works.

Step 1: Identify the Question


How do you know that this is an Inference question in the first place?

The question stem refers to the answer choices as possible conclusions:

“lead to which of the following [the answers] as a conclusion?”

GMAT Critical Reasoning (CR) questions may also ask you what can be inferred. (On Reading Comp and Integrated
Reasoning, you might see the words suggest or imply in the question stem.)

In all cases, the question is asking you what must be true based on the information you’ve been given so far.

Step 2: Deconstruct the Argument


Roland finds it alarming that 90% of people say they know someone who is unemployed.

Sharon’s response starts with the word but, so she doesn’t agree with something that Roland said. What?
According to Sharon, a normal, moderate level of unemployment is 5%. If you know 50 people who work, then 5% = 1
person.

So, if the unemployment rate is 5%, then you’d know 1 person who is unemployed. She seems to think that it’s not a big
deal that 90% of people know someone who is unemployed.

Do you know 50 people who work? You may not be close friends with 50 workers, but chances are you do know 50
people who work—your co-workers, your friends and family, acquaintances at the gym, the servers at your favorite
restaurant, etc.

Hmm. Sharon seems to think that it’s not a big deal that 90% of people know someone who is unemployed and maybe
she has a point! If I know 50 people who work, and if just one is unemployed right now, that reflects an unemployment
rate of just 5%—not a very high rate. So most people probably do know someone who is unemployed at the moment.

Note that Sharon doesn’t dispute Roland’s statistic. She just says that his figure isn’tsurprising. She thinks it’s pretty
normal and explains why.

Here’s what my notes looked like, taken as I read the argument:

The bracketed text at the end was not stated in the argument. This is my own conclusion, in my own words. I use the
brackets to signal to myself that I’m doing my own thinking here; I’m not writing what the argument actually stated.

Step 3: State the Goal


On Inference questions, the goal is to find the answer that must follow from the given information in the argument.

The most common trap on this question type is an answer that goes too far—what I call a real-world conclusion. A real-
world conclusion might be true, and even might be likelyto be true, but it doesn’t have to be true…so it’s not the right
answer on the GMAT.

Step 4: Work from Wrong to Right


All right, let’s dive in.

(A) The fact that 90% of the people know someone who is unemployed is not an indication that unemployment is
abnormally high.

OE : A. Correct. This statement properly identifies the conclusion to which the argument is leading.
Stacey : Roland is alarmed. Sharon isn’t and she has a reasonable explanation for why Roland’s statistic isn’t actually
alarming. This choice fits Sharon’s argument: she doesn’t think that Roland’s statistic indicates that unemployment is
unusually high.

(B) The current level of unemployment is not moderate.

OE :B. Sharon's argument is made in the abstract. No information is provided about the current level of unemployment.

Stacey : This one is a double trap. First, this goes along with what Roland is saying, but the question asks about
Sharon’s argument, not Roland’s. That’s trap #1.

Trap #2: We don’t technically know what the current level of unemployment is! The argument says only that 90% of
people currently know someone who is unemployed.

If anything, we could possibly real-world infer from Sharon’s argument that the current level is somewhere around 5%,
and she calls this level normal and moderate. So a choice that says it is not moderate does not match what Sharon is
saying.

(C) If at least 5% of workers are unemployed, the result of questioning a representative group of people cannot be the
percentage Roland cites.

OE : C. Sharon does not challenge Roland's statistics, and her argument is not designed to make a conclusion about
their accuracy, only their interpretation.

Stacey : This one is directly contradicted by the information given; if you follow Sharon’s math, she shows that a 5%
unemployment stat could easily lead to 90% of people knowing that someone is unemployed.

They’re trying to trap you into thinking that Sharon’s “but” means that she disputes Roland’s statistic in the first place.
She doesn’t; she just thinks that his interpretation of what the statistic means is wrong.

(D) It is unlikely that the people whose statements Roland cites are giving accurate reports.

OE : D. There is no information about the accuracy of Roland's reports, so no conclusion can be made about how likely
or unlikely they are to be accurate.

Stacey : This is another trap based on Sharon’s starting word, but. Sharon is not disputing Roland’s statistic, just how he
interprets it. He finds it alarming; she finds it normal.

(E) If an unemployment figure is given as a certain percent, the actual percentage of those without jobs is even higher.

OE : E. No information in Sharon's argument supports this conclusion.

Stacey : In the real world, this may very well be true. The method used to estimate the percentage may be missing
certain categories of people or something like that. The argument, though, doesn’t provide enough information to
conclude that this must be true. I’d call this one a real-world trap: it probably is true a lot of the time in the real world,
but it doesn’t have to be true.
The correct answer is (A). Sharon’s argument is set up to conclude that Roland’s 90% figure actually isn’t unusual and
so unemployment isn’t necessarily abnormally high. If anything, Sharon’s figures seem to show that the 90% figure goes
along with “normal, moderate” levels of unemployment, not unusually high levels.

What did you learn on this problem? Come up with your own takeaways before you read mine below.

Key Takeaways for Inference Problems


(1) Know how to identify the question type. On GMAT Critical Reasoning, this usually means some mention of the
word infer or some reference to a conclusion in the answers below (not in the argument above).

(2) Lay out the facts very clearly. Articulate to yourself what the facts do and don’t tell you. Don’t assume anything and
don’t try to draw any real-world conclusions!

(3) Look for an answer that must follow from the information you were already given. Be wary of “real-world
conclusion” traps: ones that might be true or are even pretty likely to be true but don’t have to be true. You’re not
looking for a “reasonable” conclusion. You’re looking for a “must be true” conclusion.

OG 10 / 11

Roland: The alarming fact is that 90 percent of the people in this country now report that they know someone who is unemployed.

Sharon: But a normal, moderate level of unemployment is 5 percent, with 1 out of 20 workers unemployed. So at any given time if a person
knows approximately 50 workers, 1 or more will very likely be unemployed.

Sharon's argument relies on the assumption that …..


(A) normal levels of unemployment are rarely exceeded
(B) unemployment is not normally concentrated in geographically isolated segments of the population
(C) the number of people who each know someone who is unemployed is always higher than 90% of the population
(D) Roland is not consciously distorting the statistics he presents
(E) knowledge that a personal acquaintance is unemployed generates more fear of losing one's job than does knowledge of unemployment
statistics

2nd question:

the correct answer to this problem should be (b).

* sharon cites a statistic that applies on a broad scale -- she's quoting an unemployment rate, which must apply over an extremely broad
geographical area (probably nationwide).
* she then moves to a statement about individual people, assuming that the same percentages that apply on a nationwide basis must
therefore apply to each individual.

if there are large variations in unemployment rates across different geographical areas, then this argument doesn't hold, since the number of
unemployed people known by different individuals in different areas will vary wildly.
OG13 CR 26

When a polygraph test is judged inconclusive, there is no reflection on the examinee. Rather, such a judgement means that the test has failed to
show whether the examinee was truthful or untruthful. Nevertheless, employers will sometimes refuse to hire a job applicant because of an
inconclusive polygraph test result.

Which of the following conclusions can most properly be drawn from the information above?

(A) Most examinees with inconclusive polygraph test results are in fact untruthful.

(B) Polygraph tests should not be used by employers in the consideration of job applicants.

(C) An inconclusive polygraph test result is sometimes unfairly held against the examinee.

(D) A polygraph test indicating that an examinee is untruthful can sometimes be mistaken.

(E) Some employers have refused to consider the results of polygraph tests when evaluating job applicants.

Following is the discussion article from Stacey :

Step 1: Identify the Question


First, we read the question stem:

Which of the following conclusions can most properly be drawn from the information above?•

The key identifying language is typical in this example. The language which of the following conclusions• indicates
that the answer choices contain conclusions, or inferences. This, coupled with the language can most properly be
drawn,• indicates that we have an Inference (or Draw a Conclusion) question.

The question language is generic in the sense that it doesn’t provide us with any details about the argument. We do know
several things, though. First, the argument we’re about to read will not contain any conclusion itself “ it will consist only
of premises, and those premises are usually fairly factual on this question type. Second, our task will be to find an
answer that must be true given some or all of those premises.

Step 2: Deconstruct the Argument


I already know that all of the sentences will contain premises “ but what will be the flow of the information? Will the
premises all be separate or will some pieces of information lead to others?

The first sentence indicates that (a) it’s possible for a polygraph test to be inconclusive• and (b) when this happens, it
doesn’t tell us anything about the examinee. Rather,• the second sentence says, this just means that the test has failed to
do what it was supposed to do. The test has failed, not the examinee. The third sentence begins nevertheless “ what
does that word mean? It means Despite what I’ve told you so far,• some contradictory thing comes next. Despite the
fact that an inconclusive• result only means that the test has failed, employers sometimes decide not to hire
someone because of that inconclusive result.

That’s ridiculous! It’s not my fault that the test failed! (You don’t really need to go this far in your thinking while
reading the argument, but sometimes putting ourselves into• the situation can help. : ) )

Your notes might look something like this (though there are lots of ways to write notes!):
Inconcl PT = test fail; don’t know about employee
BUT employer st won’t hire b/c of that
Note that I used abbreviations; you can use any you want as long as they make sense to you. My typical abbreviation for
sometimes or something is “st. Note that I didn’t label anything the conclusion “ because there isn’t one. Those are
all just facts.

Step 3: State the Goal


Our goal is to find an answer that must be true given some or all of the information presented in the argument. Note that
we do not need to use all of the information. There will be only one answer that must be true, given the information in
the argument.

The most common trap on this type is something that could be true and might even be likely to be true but does not
absolutely have to be true.

Step 4: Work from Wrong to Right


(A) Most examinees with inconclusive polygraph test results are in fact untruthful.•
I have no idea if this is true. The argument didn’t address whether the people who receive inconclusive polygraph results
are likely to be truthful or are not likely to be truthful. In fact, it specifically says that we can’t tell. Eliminate A.

OE : This statement makes a judgment that is explicitly contradicted in the passage, which states that an
inconclusive polygraph result is no reflection on the examinee.

(B) Polygraph tests should not be used by employers in the consideration of job applicants.•
I can believe that there are some people who would believe this or advocate for this, but the author of the argument
never comments on how the tests should or should not be used. This doesn’t have to be true.

Mitch : Too broad. Beware of answers that CHANGE THE SCOPE. The passage is only about polygraph tests whose results are
INCONCLUSIVE, not about ALL polygraphs tests. Eliminate B.

Chris , magoosh : As for why (B) doesn't work. The focus of the paragraph is polygraph test that yield inconclusive results. With (B) we
are taking into account ALL polygraph tests. Also, we are not discussing the evaluation of suspects but the use of inconclusive results in
court.

OE : Thissweeping conclusion is not as well supported by the passage as is answer choice (C). The passage
discusses only inconclusive polygraph results

(C) An inconclusive polygraph test result is sometimes unfairly held against the examinee.•
Let’s see. If an examinee were to get an inconclusive result and then not be hired specifically for that reason, then that
would be unfair, yes “ since the inconclusive result doesn’t actually mean that the person was being untruthful. Does this
ever happen? Yes, the last sentence says that employers will sometimes do this “ just as the answer choice says that
this sometimes happens.

In other words, it is possible to be truthful and get an inconclusive result, and it is possible to be untruthful and also get
an inconclusive result. Sometimes, people will not be hired because they have an inconclusive result, so it’s possible to
be truthful and yet not get hired. According to the argument, this answer choice is true; let’s keep it in and check the
remaining answers.

OE : Correct. Given the information in the passage, one can infer that inconclusive polygraph tests are
sometimes used unfairly against job applicants—if one makes the reasonable assumption that judging a job
applicant unsuitable is unfair if the judgment is based merely on the failure of a particular technique to provide
reliable evidence.
(D) A polygraph test indicating that an examinee is untruthful can sometimes be mistaken.•
I can believe that this is true in the real world but the argument never discusses a situation where a person gets an
untruthful• result from the test. The argument only discusses the case of the inconclusive result. So I can’t infer
anything for sure about an untruthful• result.

Mitch hunt : No. The passage is only about tests whose results are INCONCLUSIVE; no information is offered about a test indicating
that an examinee is UNTRUTHFUL. Eliminate D.

OE : The passage is concerned only with inconclusive tests, not cases when the polygraph test is mistaken.

(E) Some employers have refused to consider the results of polygraph tests when evaluating job applicants.•
Again, I can believe that this is true, but this circumstance doesn’t apply to the argument. The argument mentions
employers who do use the polygraph results, not those who do not.

OE : Information about employers who do not consider polygraph tests is irrelevant to the discussion.

The correct answer is C.

Answers A, B, D, and E all represent speculations “ things that we might brainstorm about such a situation, but that may
or may not be true. Maybe an inconclusive• result happens most often when someone knows how to beat• polygraph
tests, and so the person lies but gets away with it. If that were true, then answer A might be true but we were given no
actual evidence to support this idea.

Key Takeaways for Solving Inference CR Problems:


(1) Know how to recognize this type. The question stem will typically use some variation of the word conclusion or
infer. It will also mention something along the lines of most properly drawn• or best supported by.• Be careful about
one thing: Inference questions can sometimes include the words strongly supports• and this language also shows up in
Strengthen questions. If the question contains this language, check to see whether it refers to the answers as conclusions
or inferences “ if so, this is an Inference question, not a Strengthen question. (Strengthen questions will contain a
conclusion in the argument, not in the answer choices.)

(2) Know what to do with Inference questions. The argument will contain fact-based premises and no conclusion. Our
goal is to find an answer that must be true given some or all of the premises. We don’t need to use all of the information
in the argument.

(3) Watch out for traps! The answer choices usually contain Real World• inferences: things that might be or are even
likely to be true in the real world but they don’t absolutely have to be true. These are tricky because we’re used to
making reasonable assumptions in the real world when drawing a conclusion or inference “ but we can’t do that on the
GMAT.
==================================

Find the Discrepancy / Paradox

==================================

Take-aways for CR Discrepancy questions: (Stacey)


(1) The question stem will usually contain the words if true along with a request to explain something.

(2) Once you realize you have a Discrepancy question, look for the surprising event or disconnect in the argument. The
correct answer is going to hinge on this!

(3) Watch out for trap answers that do go along with the argument but don’t specifically explain the discrepancy (like
answer A in this problem). Also watch out for trap answers that seem to address the detail at issue, but instead just make
the surprising event even more surprising (like answer D in this problem).

All right, this is a Discrepancy question. The argument won’t contain a conclusion. It’ll contain some facts, at least one
of which is surprising in some way. It won’t fit with the rest of the information.
==================================

Strengthen the argument

==================================

OG13 CR 19

A major network news organization experienced a drop in viewership in the week following the airing of a controversial report on the economy.
The network also received a very large number of complaints regarding the report. The network, however, maintains that negative reactions to the
report had nothing to do with its loss of viewers.

Which of the following, if true, most strongly supports the network's position?

(A) The other major network news organizations reported similar reductions in viewership during the same week.

(B) The viewers who registered complaints with the network were regular viewers of the news organization's programs.

(C) Major network news organizations publicly attribute drops in viewership to their own reports only when they receive complaints about those
reports.

(D) This was not the first time that this network news organization has aired a controversial report on the economy that has inspired viewers to
complain to the network.

(E) Most network news viewers rely on network news broadcasts as their primary source of information regarding the economy.

Marty Murray , GMAT Coarch

Often on the GMAT critical reasoning answers are considered out of scope when they talk about something that happened or is happening
in a situation that is not the situation being discussed.
For instance, the focus of the argument in a CR question could be the effects of a tax increase in one country. Often the effects of a tax
increase in another country would be deemed out of scope.

In this case, however, there are multiple reasons why an answer choice that discusses situations that are not the specific situation being
discussed is the OA.

For one thing, as RBBmba@2104 mentioned, there is no other answer choice that fits at all. We are looking for the best answer here. So
one that could somehow be seen to be strengthening the argument will have to do. That answer is the best answer available.

At the same time, there is another specific, logical reason why answer choice A is maybe more valid than some other answer choices that
discuss situations other than that discussed in the argument.

Choice A discusses not just one possibly irrelevant situation, but rather an industry wide trend in which all of the networks experienced
reductions in viewership at the same time. This clearly is more relevant than something like, "Another network experienced a reduction in
viewership at the same time."

A parallel situation would be one in which a clothing manufacturer said that reduced sales in a particular month were not the result of a
change in its product line, and we find that that month there was a decline in clothing sales overall. Clearly that decline in overall clothing
sales is relevant.

Maybe one takeaway here is that as much as people might try to make scoring high on the GMAT a matter of following simple rules, the
truth is that while using a cookie cutter approach to answering GMAT questions works sometimes, it certainly does not work all of the
time. So be careful about seeking to impose some sort of rule driven framework on a test that is often too tricky or sophisticated to warrant
being handled in that way.

Regarding the other answer choices,

B does not provide any information that strengthens the argument. If anything it weakens it.

C discusses motivation or typical responses rather than any factors relevant to the argument.

D adds nothing to this discussion. The argument is about a lack of connection between complaints and reduced viewership, and the fact the
people have complained before does not prove anything related to the argument.

E Says nothing that is really relevant to the argument at hand.

So A it is.

_________________
Marty Murray

RBBmba@2014 : What if other major network news organizations reported similar reductions in viewership during the same
week because of airing of a controversial report on the economy -- doesn't that (re the RED part especially) WEAKENS the conclusion OR
it's an EXTREME case (and we should generally avoid such EXTREME cases on GMAT CR ?)

Marty Murray : What you put in red actually seems to maybe fit what you were originally talking about. Unless they all aired the same
report, the fact that other stations experienced a decline after airing controversial reports on the economy may not prove much.

At the same time, if we were looking for a way to weaken the argument and there were no other answer choice that weakens it at all, then
maybe what you wrote could be the best answer to a weakening question.

Regarding what you said about extreme cases, it sounds as if you are seeking to get CR questions right by using a simple but imperfect
rule.

Avoiding extreme cases sounds like something someone would do instead of actually seeing the logic of what is going on. If you really
want to get CR questions right consistently, you need to see the logic of what is going on.

That thing about avoiding extreme cases is not sure fire. It's a semi effective way of eliminating some answers if you don't really see what is
going on.
A better way to use extreme cases is as red flags. While maybe I would not eliminate an answer choice merely because it involves an
extreme statement, the fact that it does involve such a statement can be used as an alert that likely there is something wrong with it. Still, it
could be the best answer somehow, and the only way to really determine whether it is is via actually figuring out what is going on with the
argument and answer choices.

RBBmba@2014 : Other major network news organizations ALL aired the SAME REPORT -- this might be possible as the controversial
report is on the economy -- and as a result of it they had reductions in viewership during the same week... So, now will A still
STRENGTHEN the argument ?

Marty Murray : That is a cool twist. You are basically asking whether a possibly out of scope situation weakens a strengthen answer.

My answer is yes. If choice A were what you wrote above, it would no longer strengthen the argument. I mean if choice A said that all the
networks aired the story simultaneously and all of them simultaneously experienced declines in viewership, that answer choice would
clearly no longer support an argument that the decline is not related to airing the story, would it?

Mitch Hunt : to be CONTROVERSIAL means to express a view with which others DISAGREE.
If other networks air the SAME REPORT, then there is AGREEMENT among the networks -- implying that the report is NOT
controversial.
Contrary to your interpretation, the airing of a CONTROVERSIAL report serves to indicate that the report was NOT aired by other
networks.

Karishma : It is not given that anyone other than this network aired a controversial report. Every network has its own team of reporters and hence
its own report on the state of affairs. The report of this network was controversial and disliked - it doesn't mean everyone aired the same report.

RBBmba@2014 : Can't the report be from any THIRD PARTY, which ALL major networks decide to air same time as it's on ECONOMY
?

Mitch Hunt : You are suggesting that a third party did all of the following:
conducted research
assembled and edited film footage
added narration
sent to the networks the finished product.
Then each network broadcast the same finished product with no changes, with the result that all of the networks aired the SAME REPORT.
This suggestion defies common sense.

RBBmba@2014 :What I meant is that a CONTROVERSIAL report prepared by any THIRD PARTY, was aired by ALL networks same
week..But from your reply, I think, it's very UNLIKELY to see this in reality as NOT ALL NETWORK would air EXACT same
report without making any changes(re same finished product with no changes) to it.

Mitch Hunt : Your understanding seems correct.

It is important to understand what is meant by to air a report.

Last night the XYZ Network aired a controversial report about the economy.
Here, the portion in red conveys that the XYZ Network assembled ITS OWN REPORT and broadcast it.
It does NOT convey that the XYZ Network broadcast a report assembled by a third party.

If the XYZ Network were to discuss a controversial report assembled by a third party, the following wording would convey this meaning:
Last night the XYZ Network aired a story about a controversial new report.

In the CR passage at hand, the airing of a controversial report conveys that the network assembled ITS OWN REPORT and broadcast it.
Thus, it is not possible that other networks broadcast the same report.
Q. where do we account for the complaints?

Karishma : Complaints don't really have much to do with the question. Negative reaction to the report means people's dislike of the report. It isn't
only the complaints received. So the conclusion is saying that the report is not responsible for the loss of viewers.

Also, I agree that usually, when we try to strengthen something by taking an example from 'out of scope situations', it is not correct. For example, if
we are wondering whether a particular plan will succeed in country A, saying that another country has implemented it, doesn't strengthen the
probability of the plan succeeding in country A. But a lot depends on the given argument. Note here that we are saying that an internal factor (the
report) was not responsible for drop in viewership. So if we find that there was an external factor affecting all networks, then it does strengthen our
argument that the report was not the culprit. Also, you have to choose the best answer and (A) is certainly the best of the given lot.

Option C :

OE : Since the network did in fact receive complaints about the report, this statement is irrelevant.

souvik101990 : C) C does nothing because we are not concerned with the declaration of less viewership, but the reason for which it has
declined.

Option D

OE : The fact that the network has received complaints before about controversial reports on the economy that the
network’s news organization has aired tells us nothing about whether this recent report caused a subsequent drop in
viewership.

Karishma : Actually, (D) shouldn't be considered at all. Let me discuss why.

Argument:
A network aired a controversial report.
It received many complaints about the report.
It experienced a drop in viewership in the following week.
The network maintains that negative reactions to the report had nothing to do with its loss of viewers.

We need to strengthen that the report had nothing to do with loss of viewers.

(D) This was not the first time that this network news organization has aired a controversial report on the economy that has inspired viewers to
complain to the network.

So we know now that they have aired controversial reports before and got complaints. First of all, (D) doesn't tell us that there was no loss of
viewers when they aired controversial reports in the past. If there was loss of viewers in the past too, then the network's claim is not strengthened -
if anything, it is weakened a little. Even if there was no loss of viewers in the past, the network's claim still doesn't get strengthened much because
perhaps this time, the report was way beyond the tolerance level of people - we don't know. Remember, past doesn't predict the future accurately
and the future doesn't need to mirror the past. Hence, more often than not, past events will not provide much support to the future events. But we
only have the past as reference to what will happen in the future so sometimes we base our hypothesis on the past.

Option E :

OE : The fact that viewers turn to network news broadcasts as their primary source of information about the economy
tells us nothing about whether viewers might stop watching a particular network news organization’s programs as a
result of its airing a controversial report on the economy.

souvik101990 : E) E is largely irrelevant. Again we need some reason to support the network by saying that there was some other reason to
declined viewership.
OG13 CR 23

Male bowerbirds construct elaborately decorated nests, or bowers. Basing their judgment on the fact that different local populations of bowerbirds
of the same species build bowers that exhibit different building and decorative styles, researchers have concluded that the bowerbirds' building
styles are a culturally acquired, rather than a genetically transmitted, trait.

Which of the following, if true, would most strengthen the conclusion drawn by the researchers?

(A) There are more common characteristics than there are differences among the bower-building styles of the local bowerbird population that has
been studied most extensively.

(B) Young male bowerbirds are inept at bower-building and apparently spend years watching their elders before becoming accomplished in the
local bower style.

(C) The bowers of one species of bowerbird lack the towers and ornamentation characteristic of the bowers of most other species of bowerbird.

Option A

OE : The greater number of similarities than differences in style in one population could be attributed to either cultural
acquisition or genetic transmission, so the conclusion is not strengthened.

Ninja :

Notice that this choice only describes ONE local population (the one that has been studied most extensively). Within that group, there are more
common characteristics than there are differences among bower-building styles. Those similar characteristics could have been culturally acquired
within the group OR genetically transmitted. We can't tell either way, so choice (A) doesn't strengthen the conclusion. Eliminate this one.

Karishma : This tells us that there are more similarities than differences. This doesn't strengthen our argument at all. If anything, it weakens it by
implying that quite a bit could be genetically transmitted since there are more similarities.

Option B

Ninja : Young males have NO bower-building skills and must spend years watching their elders before becoming accomplished (highly skilled).
This suggests that the young males must LEARN how to build bowers from their elders. If the skills were genetic transmitted, then the males
would probably not need to watch and learn from their elders. Choice (B) suggests that the bowerbirds' building styles are culturally acquired
(learned), which supports the conclusion. Hang on to this one.

Karishma : If bower building were a genetically transmitted trait, young bowers would have known how to build. But they do not and instead they
acquire the skill by watching elders. This strengthens our argument that bower building skill is culturally acquired. Correct.

Option C
Ninja : Pay close attention to the details here. Choice (C) compares one species of bowerbird to most other species of bowerbirds.
The passage, on the other hand, is concerned with "different local populations of bowerbirds of the same species." Thus, choice (C)
has no bearing on the evidence in the passage or the conclusion. Eliminate (C).

Karishma : We are not comparing bowers of different species. We are talking about different local populations of the same species. Even if
bower building were genetically transmitted, the genes of one species would be different from those of another so genetic transmission would be
perfectly valid. Hence this option doesn't help us strengthen our argument that bower building is culturally acquired and not genetically transmitted.

Option D

Ceilidh : This is a CORRELATION vs. CAUSATION argument. Because building styles vary from location to location, the author
concludes that the building styles are culturally acquired. The assumption is that genetically transmitted traits are not location-specific for
this species of birds, and thus would not account for the difference.

Option D states that local populations seldom have contact with one another. This would imply that they do not share cultural

Ninja : We already know that different local populations of bowerbirds of the same species build bowers that exhibit different building and
decorative styles. Now we also know that those local populations probably don't have much contact with one another. This suggests that the groups
probably don't learn much from one another (little "cultural acquisition" between groups). If the groups had LOTS of contact, that
might weaken the conclusion--e.g. if the building styles are culturally acquired and the groups are in constant contact, then why would they have
different styles?

But choice (D) doesn't help us to understand the observed DIFFERENCES. Are those differences cultural acquired or is there some other
explanation (i.e. a genetic explanation)? This information does not help support (or weaken) the conclusion and should be eliminated.

DavidG : Imagine that you're observing bowerbirds in New Guinea. You're examining the distinctive style of the nests of Population A and
you're trying to settle a nature-nurture debate. Did these birds learn this style from other birds, or are they simply genetically programmed
to build this way?

If you already know that the nests in Population A are different from the nests in Population B, why would knowing that birds from A and
B rarely interact help you settle this debate?

If the nests are different, there was never any reason to suspect that the birds from Population A were learning anything from the birds from
Population B. So within A , there are still two possibilities.

Either some birds in A are learning their nest-making from other birds in A, and the trait is culturally acquired, or

the birds in A are somehow genetically distinct from the birds in B, and it's the genes that dictate the style of the nests. D doesn't tell us this.

Karishma : Whether they have contact or not doesn't matter. Whether they are in contact or no would have been useful to know if we were to
show why there are similarities in bower building styles (Close contact with other populations could have been the reason for similarities) or if we
were to prove that it is a genetically transmitted trait (far apart but similarities could have strengthened the case for genetic acquisition). The
building styles are different so it doesn't matter how close or far apart the populations are.

Option E

Ninja : Choice (E) does support the idea that learned (i.e. culturally acquired) traits DO exist among some songbirds. But are bowerbirds songbirds?
Even if they are, are they one of the songbirds whose song dialects are learned rather than genetically transmitted?

Choice (E) certainly doesn't hurt the conclusion. The conclusion is still possible given this new information, but, unlike choice (B), the new
information does not do much to strengthen the given argument.

OE : This statement provides an example of learned bird behavior, and so provides a little additional support for the
conclusion, but not as much additional support as does answer choice (B).
Karishma : Out of scope.

==================================

Evaluate the argument

==================================

In the past the country of Malvernia has relied heavily on imported oil. Malvernia recently implemented a program to convert
heating systems from oil to natural gas. Malvernia currently produces more natural gas each year than it uses, and oil
production in Malvernian oil fields is increasing at a steady pace. If these trends in fuel production and usage continue,
therefore, Malvernian reliance on foreign sources for fuel is likely to decline soon.

Which of the following would it be most useful to establish in evaluating the argument?

(A) When, if ever, will production of oil in Malvernia outstrip production of natural gas?

(B) Is Malvernia among the countries that rely most on imported oil?

(C) What proportion of Malvernia's total energy needs is met by hydroelectric, solar, and nuclear power?

(D) Is the amount of oil used each year in Malvernia for generating electricity and fuel for transportation increasing?

(E) Have any existing oil-burning heating systems in Malvernia already been converted to natural-gas-burning heating
systems?
---------------------------------------

A "most useful to evaluate" question is typically best approached from a weaken mindset. Look to establish the conclusion and
premises, decipher the gap in logic, and then think about how you would attack the gap. The correct answer will be focused
on one of the gaps in logic.

Here the argument states that while Malvernia relied heavily on imported oil in the past, the company has implemented a
program to convert heating systems from oil to natural gas, and the country is steadily increasing its oil production each year.
From these premises the argument concludes that, if these trends continue, Malvernia is likely to experience a reduced
dependency on foreign oil in the future. The primary gap in logic here is that this argument is focused on the supply of oil
increasing and then one demand of the oil potentially decreasing (heating systems). However, there could be many other oil
demands that increase drastically, potentially more than offsetting these factors. Answer D correctly addresses this, because if
the amount of oil used for generating electricity and transportation increases greatly, Malvernia may end up depending even
more on foreign oil, not less, and thus the conclusion would not hold.

Answer choice A is a pretty irrelevant comparison. Even though Malvernia currently produces more natural gas than it uses, it
may just use essentially zero natural gas. Answer choice B is completely out of scope - other countries don't matter. Answer
choice C is also out of scope - the argument is not discussing alternative energy sources. And answer choice E is irrelevant.
Knowing whether any systems have currently been converted (may 1 has, maybe 1,000,000 have) or not doesn't affect this
argument.

I hope this helps!


_________________

Brandon
Veritas Prep | GMAT Instructor

------------------

Malvernia

Step 1: Identify the Question

In the question stem, the words useful to establish in evaluating indicate that this is an Evaluate the Argument question.

Step 2: Deconstruct the Argument

Past: M lots I oil

Recent: heating oil → gas

Now: make > gas than use

oil prod incr steady

If trends cont → M rely less on foreign fuel

The conclusion is in the form of an if-then statement: If these trends continue, then something else will happen. It’s important
to note that the author’s argument is predicated on the idea that the trends will continue. Don’t fall into a trap based on the
possibility that the trends will not continue; the author hasn’t concluded anything about what might happen if the trends do not
continue. The author is only claiming that her conclusion is true if M continues to produce more gas each year than it uses and
if oil production keeps increasing at a steady pace.
What does this argument assume? M already has to import oil, but the argument assumes that increasing oil production at a
steady pace will allow the country to reduce its oil imports. What if M’s oil consumption is increasing at a much greater pace
than its oil production?

Step 3: Pause and State the Goal

On Evaluate questions, the answers will be in the form of a question or a “whether x is true” statement. The correct answer
will address an issue on which the argument hinges, depending on whether that statement is true: one way, the argument will
be strengthened; the other way, the argument will be weakened.

Step 4: Work from Wrong to Right

(A) The argument does not provide any detail about the relative amounts of oil vs. gas production. Whether M’s oil production
outstrips gas production next year or in 20 years, it is still unclear whether the country needs more oil than it produces.

(B) The argument does not compare M to other countries. It is focused solely on whether M will continue to need to import
foreign fuel.

(C) If 3% of M’s total needs are currently met by these other energy sources, nothing is changed about the current oil and gas
needs as stated in the argument. If 50% of M’s total needs are currently met by these other energy sources, it is still the case
that current oil and gas needs are the same as in the argument.

(D) CORRECT. If M needs an increasing amount of oil every year for other uses (electricity and transportation), then it may
be the case that M’s oil production is not increasing at a fast enough rate to keep pace with demand. In this case, the
argument is weakened. If, on the other hand, M does not need an increasing amount of oil every year for these other needs,
then it may be the case that M’s oil production is increasing at a fast enough rate to keep pace with demand, in which case
the argument is strengthened.

(E) If none have already been converted, the argument is not impacted because the conclusion is claimed to happen soon; the
argument does not say that the conclusion has already occurred. If 1,000 have already been converted, the argument is still
not impacted, because the conclusion is still set in the future.
_________________

Cheers!
-----------------------------
Please give kudos if you think it is worth it !

-----------------

VeritasPrepMark

Veritas Prep GMAT Instructor

Hi guys,

Looks like there's some confusion on this one, so let's give it a closer look. Thabk's emphasis on the conclusion is important
(and correct). When a CR question asks what would be most useful to evaluate "the argument", really it is asking you what
would help evaluate the conclusion. The conclusion here is clearly marked by the keyword "therefore": "Malvernian reliance on
foreign sources for fuel is likely to decline soon."

And what would help evaluate the conclusion? An answer choice that will (depending on what is established) either strengthen
or weaken that conclusion. So really, you can treat this kind of question as either Strengthen or Weaken, but in my experience
it's usually easiest to treat these "useful to evaluate" questions as Weaken. Think of it this way: The author's argument is
missing some information (what it would be useful to establish), and therefore there is a gap between his evidence and his
conclusion. The correct answer would expose that gap, just as it would in a standard Weaken question.

To the stimulus. On first reading, it seems like a pretty reasonable argument. The Malvernians have relied on imported oil in
the past, but they may be able to turn that around, for two reasons:

1. They're switching from oil to natural gas, of which they have a plentiful supply.
2. Domestic oil production is increasing.

If domestic oil supply is going up and demand is going down, it seems almost inevitable that their reliance on imported oil will
decrease. However, note the limited scope of the first premise: they aren't switching ALL oil usage to natural gas, only the fuel
used in their heating systems. We actually don't know anything about the overall demand for oil. That is the gap in the
author's reasoning. Choice D exploits this gap: if the amount of oil used for non-heating purposes (generating electricity and
transportation) is increasing, then the overall demand for oil might still be increasing too fast for domestic oil production to
keep pace (even though production is increasing as well).

E seems to be the most popular wrong answer choice, so let's talk about that one too. It references the heating systems
mentioned in the stimulus, so it's tempting. However, remember that we need to evaluate the conclusion. Will the fact that
some oil-burning systems have already been converted to natural gas affect whether Malvernian reliance on foreign sources
for fuel "is likely to decline soon"? Not really. Whether the conversions have already started or not, they will happen, so in
effect this answer neither adds to nor takes anything away from the argument. Now, you could get into all sorts of
complicated logic to convince yourself that E is the answer -- Like, maybe because the conversions have started, that means
that the all the conversions will happen faster, and maybe if the conversions happen faster it's more likely that reliance on
foreign oil will decline soon... Maybe, maybe, maybe. This kind of speculative thinking is never required on the GMAT (and in
fact can be disastrous). The correct answer on a Strengthen or Weaken question must DIRECTLY affect the conclusion -- not
after multiple steps. Don't overthink CR answer choices!

However, as this question shows, it is extremely important to read the stimulus carefully and note the specific wording
involved. Better to spend a little extra time up front to understand the stimulus rather than flounder around with the answer
choices, not knowing what you're looking for. Ideally on this question you would be able to predict, in vague from, what the
answer will look like before reading the answer choices. However, this isn't always possible. As long as you have a good
understanding of the stimulus you should be fine.

Hope that sheds some light on this question.

Happy studying...

You might also like